FM All Exercises

You might also like

Download as pdf or txt
Download as pdf or txt
You are on page 1of 127

UNSW Business School

School of Risk and Actuarial Studies

Financial Mathematics
Exercises
Contents

1 Time Value of Money and Cash Flow Valuation 2


1.1 Time Value of Money . . . . . . . . . . . . . . . . . . . . . . . . . . . 2

Exercise 1.1 [int1] . . . . . . . . . . . . . . . . . . . . . . . . . 2

Exercise 1.2 [int2] . . . . . . . . . . . . . . . . . . . . . . . . . 2

Exercise 1.3 [int3] . . . . . . . . . . . . . . . . . . . . . . . . . 2

Exercise 1.4 [int4] . . . . . . . . . . . . . . . . . . . . . . . . . 2

Exercise 1.5 [int5] . . . . . . . . . . . . . . . . . . . . . . . . . 2

Exercise 1.6 [int7] . . . . . . . . . . . . . . . . . . . . . . . . . 2

Exercise 1.7 [int8] . . . . . . . . . . . . . . . . . . . . . . . . . 2

Exercise 1.8 [int9] . . . . . . . . . . . . . . . . . . . . . . . . . 2

Exercise 1.9 [int10] . . . . . . . . . . . . . . . . . . . . . . . . 3

Exercise 1.10 [int11] . . . . . . . . . . . . . . . . . . . . . . . 3

Exercise 1.11 [int12] . . . . . . . . . . . . . . . . . . . . . . . 3

Exercise 1.12 [int13] . . . . . . . . . . . . . . . . . . . . . . . 3

Exercise 1.13 [int15] . . . . . . . . . . . . . . . . . . . . . . . 3

Exercise 1.14 [int19] . . . . . . . . . . . . . . . . . . . . . . . 4

Exercise 1.15 [int18] . . . . . . . . . . . . . . . . . . . . . . . 4

Exercise 1.16 [int20] . . . . . . . . . . . . . . . . . . . . . . . 5

Exercise 1.17 [int6] . . . . . . . . . . . . . . . . . . . . . . . . 5

Exercise 1.18 [int14] . . . . . . . . . . . . . . . . . . . . . . . 5

Exercise 1.19 [int24] . . . . . . . . . . . . . . . . . . . . . . . 5

Exercise 1.20 [int25] . . . . . . . . . . . . . . . . . . . . . . . 5

Exercise 1.21 [int21] . . . . . . . . . . . . . . . . . . . . . . . 5

Exercise 1.22 [int22] . . . . . . . . . . . . . . . . . . . . . . . 6

Exercise 1.23 [int23] . . . . . . . . . . . . . . . . . . . . . . . 6

Exercise 1.24 [new10] . . . . . . . . . . . . . . . . . . . . . . . 6

i
Financial Mathematics  Exercises School of Risk and Actuarial Studies  UNSW

1.2 Level Annuities . . . . . . . . . . . . . . . . . . . . . . . . . . . . . . 6

Exercise 1.25 [ann1] . . . . . . . . . . . . . . . . . . . . . . . . 6

Exercise 1.26 [ann2] . . . . . . . . . . . . . . . . . . . . . . . . 6

Exercise 1.27 [ann3] . . . . . . . . . . . . . . . . . . . . . . . . 7

Exercise 1.28 [annB1] . . . . . . . . . . . . . . . . . . . . . . . 7

Exercise 1.29 [annB2] . . . . . . . . . . . . . . . . . . . . . . . 7

Exercise 1.30 [annB3] . . . . . . . . . . . . . . . . . . . . . . . 7

Exercise 1.31 [annB4] . . . . . . . . . . . . . . . . . . . . . . . 7

Exercise 1.32 [annB5] . . . . . . . . . . . . . . . . . . . . . . . 7

Exercise 1.33 [annB6] . . . . . . . . . . . . . . . . . . . . . . . 7

Exercise 1.34 [annB7] . . . . . . . . . . . . . . . . . . . . . . . 7

Exercise 1.35 [ann4] . . . . . . . . . . . . . . . . . . . . . . . . 7

Exercise 1.36 [ann5] . . . . . . . . . . . . . . . . . . . . . . . . 7

Exercise 1.37 [ann6] . . . . . . . . . . . . . . . . . . . . . . . . 7

Exercise 1.38 [ann7] . . . . . . . . . . . . . . . . . . . . . . . . 7

Exercise 1.39 [annB8] . . . . . . . . . . . . . . . . . . . . . . . 7

Exercise 1.40 [annB9] . . . . . . . . . . . . . . . . . . . . . . . 7

Exercise 1.41 [ann13] . . . . . . . . . . . . . . . . . . . . . . . 8

1.3 Non-Level and Continuous Annuities . . . . . . . . . . . . . . . . . . 8

Exercise 1.42 [ann8] . . . . . . . . . . . . . . . . . . . . . . . . 8

Exercise 1.43 [ann9] . . . . . . . . . . . . . . . . . . . . . . . . 8

Exercise 1.44 [ann12] . . . . . . . . . . . . . . . . . . . . . . . 8

Exercise 1.45 [new8] . . . . . . . . . . . . . . . . . . . . . . . 8

Exercise 1.46 [ann14] . . . . . . . . . . . . . . . . . . . . . . . 8

Exercise 1.47 [ann15] . . . . . . . . . . . . . . . . . . . . . . . 8

Exercise 1.48 [new4] . . . . . . . . . . . . . . . . . . . . . . . 9

Exercise 1.49 [new1] . . . . . . . . . . . . . . . . . . . . . . . 9

Exercise 1.50 [annB10] . . . . . . . . . . . . . . . . . . . . . . 9

Exercise 1.51 [loaB2] . . . . . . . . . . . . . . . . . . . . . . . 9

Exercise 1.52 [annB11] . . . . . . . . . . . . . . . . . . . . . . 9

Exercise 1.53 [loaB1] . . . . . . . . . . . . . . . . . . . . . . . 9

Exercise 1.54 [annB12] . . . . . . . . . . . . . . . . . . . . . . 9

Exercise 1.55 [annB13] . . . . . . . . . . . . . . . . . . . . . . 9

Exercise 1.56 [annB14] . . . . . . . . . . . . . . . . . . . . . . 9

ii
Financial Mathematics  Exercises School of Risk and Actuarial Studies  UNSW

Exercise 1.57 [annB15] . . . . . . . . . . . . . . . . . . . . . . 9

Exercise 1.58 [new2] . . . . . . . . . . . . . . . . . . . . . . . 10

Exercise 1.59 [ann11] . . . . . . . . . . . . . . . . . . . . . . . 10

2 Loans and Investments 11


2.1 Loan Repayments . . . . . . . . . . . . . . . . . . . . . . . . . . . . . 11

Exercise 2.1 [loa1] . . . . . . . . . . . . . . . . . . . . . . . . . 11

Exercise 2.2 [loa2] . . . . . . . . . . . . . . . . . . . . . . . . . 11

Exercise 2.3 [loa3] . . . . . . . . . . . . . . . . . . . . . . . . . 12

Exercise 2.4 [loaB3] . . . . . . . . . . . . . . . . . . . . . . . . 12

Exercise 2.5 [loaB4] . . . . . . . . . . . . . . . . . . . . . . . . 12

Exercise 2.6 [loaB5] . . . . . . . . . . . . . . . . . . . . . . . . 12

Exercise 2.7 [loa5] . . . . . . . . . . . . . . . . . . . . . . . . . 12

Exercise 2.8 [loa6] . . . . . . . . . . . . . . . . . . . . . . . . . 12

Exercise 2.9 [loa7] . . . . . . . . . . . . . . . . . . . . . . . . . 13

Exercise 2.10 [loa8] . . . . . . . . . . . . . . . . . . . . . . . . 13

2.2 Investments . . . . . . . . . . . . . . . . . . . . . . . . . . . . . . . . 13

Exercise 2.11 [loa4] . . . . . . . . . . . . . . . . . . . . . . . . 13

Exercise 2.12 [loa9] . . . . . . . . . . . . . . . . . . . . . . . . 13

Exercise 2.13 [new13] . . . . . . . . . . . . . . . . . . . . . . . 14

Exercise 2.14 [loa10] . . . . . . . . . . . . . . . . . . . . . . . 14

Exercise 2.15 [loaB6] . . . . . . . . . . . . . . . . . . . . . . . 14

Exercise 2.16 [loaB7] . . . . . . . . . . . . . . . . . . . . . . . 14

Exercise 2.17 [loa11] . . . . . . . . . . . . . . . . . . . . . . . 14

Exercise 2.18 [ann16] . . . . . . . . . . . . . . . . . . . . . . . 14

Exercise 2.19 [loa12] . . . . . . . . . . . . . . . . . . . . . . . 15

Exercise 2.20 [loa13] . . . . . . . . . . . . . . . . . . . . . . . 15

Exercise 2.21 [loa14] . . . . . . . . . . . . . . . . . . . . . . . 15

3 Interest Rate Risk 16


3.1 Term Structure of Interest Rates . . . . . . . . . . . . . . . . . . . . 16

Exercise 3.1 [irr1] . . . . . . . . . . . . . . . . . . . . . . . . . 16

Exercise 3.2 [irr2] . . . . . . . . . . . . . . . . . . . . . . . . . 16

Exercise 3.3 [irr3] . . . . . . . . . . . . . . . . . . . . . . . . . 16

Exercise 3.4 [irr4] . . . . . . . . . . . . . . . . . . . . . . . . . 16

iii
Financial Mathematics  Exercises School of Risk and Actuarial Studies  UNSW

Exercise 3.5 [irr5] . . . . . . . . . . . . . . . . . . . . . . . . . 17

Exercise 3.6 [irr13] . . . . . . . . . . . . . . . . . . . . . . . . 17

Exercise 3.7 [irr14] . . . . . . . . . . . . . . . . . . . . . . . . 17

Exercise 3.8 [irr15] . . . . . . . . . . . . . . . . . . . . . . . . 18

Exercise 3.9 [irr16] . . . . . . . . . . . . . . . . . . . . . . . . 18

Exercise 3.10 [irr17] . . . . . . . . . . . . . . . . . . . . . . . . 18

Exercise 3.11 [irr18] . . . . . . . . . . . . . . . . . . . . . . . . 18

Exercise 3.12 [irr19] . . . . . . . . . . . . . . . . . . . . . . . . 19

Exercise 3.13 [irr20] . . . . . . . . . . . . . . . . . . . . . . . . 19

3.2 Price Sensitivity . . . . . . . . . . . . . . . . . . . . . . . . . . . . . . 19

Exercise 3.14 [irr6] . . . . . . . . . . . . . . . . . . . . . . . . 19

Exercise 3.15 [irr7] . . . . . . . . . . . . . . . . . . . . . . . . 19

Exercise 3.16 [new11] . . . . . . . . . . . . . . . . . . . . . . . 20

Exercise 3.17 [lif6] . . . . . . . . . . . . . . . . . . . . . . . . . 20

Exercise 3.18 [dur1] . . . . . . . . . . . . . . . . . . . . . . . . 20

Exercise 3.19 [dur2] . . . . . . . . . . . . . . . . . . . . . . . . 21

Exercise 3.20 [dur3] . . . . . . . . . . . . . . . . . . . . . . . . 21

Exercise 3.21 [dur4] . . . . . . . . . . . . . . . . . . . . . . . . 21

Exercise 3.22 [dur5] . . . . . . . . . . . . . . . . . . . . . . . . 22

Exercise 3.23 [dur6] . . . . . . . . . . . . . . . . . . . . . . . . 22

3.3 Immunisation . . . . . . . . . . . . . . . . . . . . . . . . . . . . . . . 23

Exercise 3.24 [irr8] . . . . . . . . . . . . . . . . . . . . . . . . 23

Exercise 3.25 [irr9] . . . . . . . . . . . . . . . . . . . . . . . . 23

Exercise 3.26 [irr10] . . . . . . . . . . . . . . . . . . . . . . . . 23

Exercise 3.27 [new12] . . . . . . . . . . . . . . . . . . . . . . . 24

Exercise 3.28 [irr11] . . . . . . . . . . . . . . . . . . . . . . . . 24

Exercise 3.29 [irr12] . . . . . . . . . . . . . . . . . . . . . . . . 25

Exercise 3.30 [imm1] . . . . . . . . . . . . . . . . . . . . . . . 25

Exercise 3.31 [imm2] . . . . . . . . . . . . . . . . . . . . . . . 25

Exercise 3.32 [imm3] . . . . . . . . . . . . . . . . . . . . . . . 26

4 Derivatives 27
4.1 Forwards, Futures and Swaps . . . . . . . . . . . . . . . . . . . . . . 27

Exercise 4.1 [der1] . . . . . . . . . . . . . . . . . . . . . . . . 27

Exercise 4.2 [der2] . . . . . . . . . . . . . . . . . . . . . . . . 27

iv
Financial Mathematics  Exercises School of Risk and Actuarial Studies  UNSW

Exercise 4.3 [der3] . . . . . . . . . . . . . . . . . . . . . . . . 27

Exercise 4.4 [der4] . . . . . . . . . . . . . . . . . . . . . . . . 27

Exercise 4.5 [der5] . . . . . . . . . . . . . . . . . . . . . . . . 27

Exercise 4.6 [der6] . . . . . . . . . . . . . . . . . . . . . . . . 28

Exercise 4.7 [der7] . . . . . . . . . . . . . . . . . . . . . . . . 28

Exercise 4.8 [der8] . . . . . . . . . . . . . . . . . . . . . . . . 28

Exercise 4.9 [der9] . . . . . . . . . . . . . . . . . . . . . . . . 28

Exercise 4.10 [der10] . . . . . . . . . . . . . . . . . . . . . . . 28

Exercise 4.11 [swp1] . . . . . . . . . . . . . . . . . . . . . . . 28

Exercise 4.12 [for1] . . . . . . . . . . . . . . . . . . . . . . . . 28

Exercise 4.13 [for2] . . . . . . . . . . . . . . . . . . . . . . . . 29

Exercise 4.14 [for3] . . . . . . . . . . . . . . . . . . . . . . . . 29

Exercise 4.15 [for4] . . . . . . . . . . . . . . . . . . . . . . . . 29

4.2 Options . . . . . . . . . . . . . . . . . . . . . . . . . . . . . . . . . . 29

Exercise 4.16 [der11] . . . . . . . . . . . . . . . . . . . . . . . 29

Exercise 4.17 [der12] . . . . . . . . . . . . . . . . . . . . . . . 30

Exercise 4.18 [der13] . . . . . . . . . . . . . . . . . . . . . . . 30

Exercise 4.19 [der14] . . . . . . . . . . . . . . . . . . . . . . . 30

Exercise 4.20 [der15] . . . . . . . . . . . . . . . . . . . . . . . 30

Exercise 4.21 [der16] . . . . . . . . . . . . . . . . . . . . . . . 30

Exercise 4.22 [der17] . . . . . . . . . . . . . . . . . . . . . . . 30

Exercise 4.23 [opt1] . . . . . . . . . . . . . . . . . . . . . . . . 30

Exercise 4.24 [opt2] . . . . . . . . . . . . . . . . . . . . . . . . 31

Exercise 4.25 [opt3] . . . . . . . . . . . . . . . . . . . . . . . . 31

Exercise 4.26 [opt4] . . . . . . . . . . . . . . . . . . . . . . . . 31

Exercise 4.27 [opt5] . . . . . . . . . . . . . . . . . . . . . . . . 31

Exercise 4.28 [opt6] . . . . . . . . . . . . . . . . . . . . . . . . 32

5 Stochastic Interest Rates 33


5.1 IID Returns . . . . . . . . . . . . . . . . . . . . . . . . . . . . . . . . 33

Exercise 5.1 [sto1] . . . . . . . . . . . . . . . . . . . . . . . . . 33

Exercise 5.2 [sto2] . . . . . . . . . . . . . . . . . . . . . . . . . 33

Exercise 5.3 [sto3] . . . . . . . . . . . . . . . . . . . . . . . . . 33

Exercise 5.4 [sto4] . . . . . . . . . . . . . . . . . . . . . . . . . 34

Exercise 5.5 [sto5] . . . . . . . . . . . . . . . . . . . . . . . . . 34

v
Financial Mathematics  Exercises School of Risk and Actuarial Studies  UNSW

Exercise 5.6 [iid1] . . . . . . . . . . . . . . . . . . . . . . . . . 34

Exercise 5.7 [iid2] . . . . . . . . . . . . . . . . . . . . . . . . . 34

Exercise 5.8 [iid3] . . . . . . . . . . . . . . . . . . . . . . . . . 35

5.2 Lognormal Model . . . . . . . . . . . . . . . . . . . . . . . . . . . . . 36

Exercise 5.9 [sto6] . . . . . . . . . . . . . . . . . . . . . . . . . 36

Exercise 5.10 [sto7] . . . . . . . . . . . . . . . . . . . . . . . . 36

Exercise 5.11 [sto8] . . . . . . . . . . . . . . . . . . . . . . . . 36

Exercise 5.12 [sto9] . . . . . . . . . . . . . . . . . . . . . . . . 37

Exercise 5.13 [sto10] . . . . . . . . . . . . . . . . . . . . . . . 37

Exercise 5.14 [log1] . . . . . . . . . . . . . . . . . . . . . . . . 37

Exercise 5.15 [log2] . . . . . . . . . . . . . . . . . . . . . . . . 37

Exercise 5.16 [log3] . . . . . . . . . . . . . . . . . . . . . . . . 38

Exercise 5.17 [log4] . . . . . . . . . . . . . . . . . . . . . . . . 38

5.3 Dependence and Further Concepts . . . . . . . . . . . . . . . . . . . 38

Exercise 5.18 [new3] . . . . . . . . . . . . . . . . . . . . . . . 38

Exercise 5.19 [new5] . . . . . . . . . . . . . . . . . . . . . . . 39

Exercise 5.20 [new6] . . . . . . . . . . . . . . . . . . . . . . . 39

Exercise 5.21 [new9] . . . . . . . . . . . . . . . . . . . . . . . 39

6 Solutions to Exercises 41
6.1 Module 1 . . . . . . . . . . . . . . . . . . . . . . . . . . . . . . . . . 41

6.2 Module 2 . . . . . . . . . . . . . . . . . . . . . . . . . . . . . . . . . 58

6.3 Module 3 . . . . . . . . . . . . . . . . . . . . . . . . . . . . . . . . . 69

6.4 Module 4 . . . . . . . . . . . . . . . . . . . . . . . . . . . . . . . . . 90

6.5 Module 5 . . . . . . . . . . . . . . . . . . . . . . . . . . . . . . . . . 103

1
Module 1

Time Value of Money and Cash Flow


Valuation

1.1 Time Value of Money

Exercise 1.1: [int1] Determine the interest earned during the 5th year by $100
invested today under compound interest with i = 0.05, and under simple interest
with i = 0.05.

Exercise 1.2: [int2] At what rate of compound interest will $200 grow to $275 in
5 years?

Exercise 1.3: [int3] How many years does it take $200 to accumulate to $275 at
an eective annual rate of 5%?

Exercise 1.4: [int4] With compound interest at i = 0.05, what is the present value
now of $275 in 5 years?

Exercise 1.5: [int5] If $150 grows to $240 in n years, what will $1000 grow to over
the same period?

Exercise 1.6: [int7] If funds invested today will earn 8% for the next 10 years and
at least 5% for the following 10 years, what is the most one must invest today to
accumulate $1 million in 20 years?

Exercise 1.7: [int8] What level rate of interest is equivalent to 8% for the next 10
years followed by 5% for the following 10 years?

Exercise 1.8: [int9] Assuming an eective rate of i = 0.10, nd the value of the
cash ows below at times t=0 and t = 3.

2
Financial Mathematics  Exercises School of Risk and Actuarial Studies  UNSW

Exercise 1.9: [int10] The following cash ows have a value of 7.7217 at time t=0
(assuming i = 0.05):

Find the value of the following cash ows at time 0:

Exercise 1.10: [int11] If $100 is deposited at time t=0 into an account earning
10% interest and $20 is withdrawn at t = 1 and 2, then how much can be withdrawn
at t = 3?

Exercise 1.11: [int12] At what rate of interest will $100 accumulate to $200 in 6
years?

Exercise 1.12: [int13] (SOA Course 2 May 2000, Question 1) Joe deposits $10
today and another $30 in ve years into a fund paying simple interest of 11% per
year. Tina will make the same two deposits, but the $10 will be deposited n years
from today and the $30 will be deposited 2n years from today. Tina's deposits earn
an annual eective rate of 9.15%. At the end of 10 years, the accumulated amount
of Tina's deposits equals the accumulated amount of Joe's deposits. What is the
value of n?

Exercise 1.13: [int15] (SOA Course 2 Nov 2000, Question 2) The following table
shows the annual eective interest rates being credited by an investment account,

3
Financial Mathematics  Exercises School of Risk and Actuarial Studies  UNSW

by calendar year of investment. The investment year method is applicable for the
rst 3 years, after which a portfolio rate is used.

Calendar Year of Investment Calendar Year of Portfolio


Investment Year Rates Portfolio Rates Rate
i1 i2 i3
1990 10% 10% x% 1993 8%
1991 12% 5% 10% 1994 (x − 1)%
1992 8% (x − 2)% 12% 1995 6%
1993 9% 11% 6% 1996 9%
1994 7% 7% 10% 1997 10%

An investment of $100 is made at the beginning of years 1990, 1991, and 1992. The
total amount of interest credited by the fund during the year 1993 is equal to 28.40.
What is the value of x?

Exercise 1.14: [int19] You wish to buy a new home theatre system and have two
potential payment options.

Option A. You pay $610 down (at t = 0), $475 next year (at t = 1) and $340 the
following year (at t = 2).
Option B. You pay $560 down (at t = 0), $580 next year (at t = 1) and $274 the
following year (at t = 2).

Assuming a compound interest accumulation function, determine the values of the


rate of interest r for which Option A is preferred to Option B.

Exercise 1.15: [int18] In return for a single payment of $1000, an investment bank
oers the following alternatives:

A. A lump sum of $1330 after three years

B. A lump sum of $1550 after ve years

C. Four annual payments, each of amount $425, the rst payment being made
after ve years

You wish to decide what alternative is the best.

(a) Write down an equation of value for each alternative and nd the yield for
each.

(b) Assume that an investor selects alternative A and that after three years she
invests the proceeds for a further two years at a xed rate of interest. How
large must this rate of interest be in order for her to receive at least $1550 at
the end of 5 years?

Why do we ask this question?

4
Financial Mathematics  Exercises School of Risk and Actuarial Studies  UNSW

(c) Assume that an investor selects alternative B and that after ve years he wants
to compare his proceeds with the value of alternative C. Determine the value
of alternative C in 5 years time. What interest rate should you use?

Exercise 1.16: [int20] Use an equation of value to determine the level annual pay-
ment (in arrears) equal in value to $1,000,000 (at time t = 0) at an interest rate of
13% p.a. eective, allowing for 5 payments.

Exercise 1.17: [int6] If v = 0.94, what are d and i?

Exercise 1.18: [int14] (SOA Course 2 May 2001, Question 12) Bruce and Robbie
each open up new bank accounts at time 0. Bruce deposits $100 into his bank
account and Robbie deposits $50 into his. Each account earns an annual eective
discount rate of d. The amount of interest earned in Bruce's account during the
11th year is equal to X. The amount of interest earned in Robbie's account during
the 17th year is also equal to X. What is the value of X?

Exercise 1.19: [int24] Tina issues a 2-year promissory note for a face value of
$6000 and receives $4843.30 in return (ie. she borrows $4843.30 and promises to
repay $6000 after 2 years). At the end of 6 months, 1 year, and 18 months, she
deposits $1000, $1000, and $2000 into her bank account and earns the same interest
rate as the implied rate on the promissory note. Assuming interest is compounded
semi-annually, determine how much extra money (in addition to the amount in her
bank account) she will need to redeem (repay) the note for its face value in 2 years
time.

Exercise 1.20: [int25] A trust account quotes a nominal annual interest rate of 6%.
Interest is credited quarterly, on the last day of each March, June, September and
December. Simple interest is paid for amounts on deposit for less than a quarter of
a year. In 2001, Maria made 4 deposits of $1000 into her trust account every 1st day
of March, June, September, December. By 31 December 2005, how much interest
will Maria have earned from these deposits?

Exercise 1.21: [int21] You are given the following interest options:

A. an eective rate of discount of 5% per annum

B. a nominal rate of interest of 5% per annum convertible semi-annually

C. a nominal rate of interest of 5% per annum convertible monthly

D. a nominal rate of discount of 5% per annum converted semi-annually

E. a nominal rate of discount of 5% per annum converted monthly

F. a force of interest of 5%

5
Financial Mathematics  Exercises School of Risk and Actuarial Studies  UNSW

(a) Discuss dierences between the above interest rates by expressing each option
as eective rates of interest.

(b) How much will an investment of $10,000 accumulate to in 4 12 years based on


each of the above interest options?

Exercise 1.22: [int22] Assume that the force of interest is δ(t) = 0.04(1 + t)−1 ,
with t measured in years. Using an accumulation function a(t):

(a) Calculate the equivalent eective rate of interest for the period t = 1 to t = 2.

(b) Calculate the equivalent eective rate of interest for the period t = 2 to t = 3.

(c) Find the value at time t=2 of an investment that accumulates to $200,000
at time t = 4.

Exercise 1.23: [int23] A fund credits simple interest with i = 10% from time zero
to time k. After time k, the fund accumulates at a constant force of interest of 8%.

(a) Find the value of k that maximises a(4).

(b) Using the value of k from (a), nd the force of interest as function of time t
(for 0 ≤ t ≤ 4).

Exercise 1.24: [new10] Suppose the force of interest is δ = 0.05. Using Excel, plot
(on the same graph) (i) i(m) against m and (ii) d(m) against m, for 0.5 < m < 50

1.2 Level Annuities

Exercise 1.25: [ann1] You are given a combined annuity-immediate payable monthly
such that payments are $1000 p.a. for the rst 6 years and $400 p.a. for the next
4 years together with a lump sum of $2000 at the end of the 10 years. An interest
rate of 12% p.a convertible monthly is assumed.

(a) Find the present value of this annuity.

(b) Calculate the amount of the level annuity-immediate payable for 10 years
having the same present value as the payments in (a).

Exercise 1.26: [ann2] Bill leaves an inheritance to four charities, Faith Foundation
(F), Hope Institution (H), Love Trust (L) and Peace, Inc. (P). The total inheritance
is a series of level payments at the end of each year forever. During the rst 20 years,
F, H, and L share each payment equally. All payments after 20 years revert to P.
The present values of the shares of the four charities are known to be all equal.
What is the implied eective rate of interest?

6
Financial Mathematics  Exercises School of Risk and Actuarial Studies  UNSW

Exercise 1.27: [ann3] Cathy must pay o a loan with ve annual payments of
$15,000 each. The rst loan payment is due 10 years from now. In order to accumu-
late the funds, she plans on making ten annual deposits of C into an account paying
eective annual interest of 6%. Having computed the least possible amount C (and
assuming she succeeded in her nancial mathematics course and thus it took her a
negligible amount of time), she immediately makes the rst deposit. Calculate C.

Exercise 1.28: [annB1] Exercise 2.1.11 from Broverman 5th Ed (2.1.11 in 4th Ed).

Exercise 1.29: [annB2] Exercise 2.1.19S from Broverman 5th Ed (2.1.19S in 4th
Ed).

Exercise 1.30: [annB3] Exercise 2.1.25 from Broverman 5th Ed (2.1.25 in 4th Ed).

Exercise 1.31: [annB4] Exercise 2.1.28 from Broverman 5th Ed (2.1.28 in 4th Ed).

Exercise 1.32: [annB5] Exercise 2.1.31 from Broverman 5th Ed (2.1.31 in 4th Ed).

Exercise 1.33: [annB6] Exercise 2.2.20 from Broverman 5th Ed (2.2.20 in 4th Ed).

Exercise 1.34: [annB7] Exercise 2.2.26 from Broverman 5th Ed (2.2.26 in 4th Ed).

Exercise 1.35: [ann4] To settle a $100,000 death benet, Tim, the primary ben-
eciary, opted to take an annuity-immediate payable monthly for 25 years. The
monthly payment was calculated using an eective annual interest rate of 3%. Af-
ter making payments for 10 years, the insurance company decides to increase the
monthly payments for the remaining 15 years by changing the eective annual in-
terest rate to 5%. By how much will the monthly payment increase?

Exercise 1.36: [ann5] Find the present value of a set of cash ows which pay $100
at the end of year 1, $200 at the end of year 2, $100 at end of year 3, $200 at the end
of year 4, and so on ($100 at odd years, $200 at even), with the nal payment being
at the end of the 20th year. The interest rate is 5% p.a. semi-annual compounding.

Exercise 1.37: [ann6] Bob has inherited an annuity-due on which there remain 12
payments of $18,000 per year at an eective discount rate of 10%; the rst payment
is due immediately. He wishes to convert this to a 20-year annuity-immediate at
the same eective rate of discount. What will be the size of the payments under the
new annuity?

Exercise 1.38: [ann7] Broverman 5th Ed: 2.2.13 (2.2.13 in 4th Ed). Also solve for
the case if Smith repays the loan over 5 years (monthly payments).

Exercise 1.39: [annB8] Exercise 2.2.1 from Broverman 5th Ed (2.2.1 in 4th Ed).

7
Financial Mathematics  Exercises School of Risk and Actuarial Studies  UNSW

Exercise 1.40: [annB9] Exercise 2.2.18 from Broverman 5th Ed (2.2.18 in 4th Ed).

Exercise 1.41: [ann13] Given that δ(t) = 1


20−t
, t ≥ 0, nd s10 .

1.3 Non-Level and Continuous Annuities

Exercise 1.42: [ann8] A loan of $4000 is being repaid by a 30-year increasing


annuity-immediate where payments increase each year and payments are in arrears.
The initial payment is P, each subsequent payment is P larger than the preceding
payment. The annual eective interest rate is 4%. Calculate the value of the future
payments (ie. the loan principal outstanding) after the ninth payment. Compare
your result with the initial loan amount and explain it. Is such a payment pattern
likely to exist in reality? Why?

Exercise 1.43: [ann9] Nicole, a UNSW Business part-time student, expects an


increasing amount of income as she advances through her program but will need to
borrow to cover her university costs. Accordingly, she plans to borrow a decreasing
annual amount from a student credit loan during her 5 years at university, and to
repay the loan with increasing amounts for 15 years after graduation. She borrows
amounts 5X, 4X, 3X, 2X and X at the beginning of each of 5 years, where the last
payment is paid at the beginning of her nal year. At the end of the rst year after
graduation she pays $500, then increases the amount by $200 each year until a nal
payment of $3300. If the eective annual interest rate assumed is 5%, determine X.

Exercise 1.44: [ann12] Paulo is saving madly to buy his rst home ten years from
now. He deposits to a fund each January 1 and July 1 for the years 2004 through
2014. The deposit he makes on each July 1 will be 10.25% greater than the one on
the immediately preceding January 1. The amount he deposits on each January 1
(except for January 1, 2004) will be the same amount as the deposit made on the
immediately preceding July 1. The fund will be credited with interest at a nominal
annual rate of 10%, compounded quarterly. On December 31, 2014, the fund will
have a balance of $110,000, an amount Paulo considers is enough for a home deposit
and other miscellaneous expenses. Determine Paulo's initial deposit to the fund.

Exercise 1.45: [new8] Using Excel, nd the present value of a 30-year annuity
immediate which pays t3 + ln(10t + 12) at the end of year t, assuming an eective
interest rate of 5%.

Exercise 1.46: [ann14] Value the following set of cashows at a rate of 10% p.a.:
1 2 1 2 1
$10 at time , $20 at times and time 1 , $30 at times 1,1 and 2 , $40 at times
3 3 3 3 3
2, 2 32 and 3 13 , $50 at times 3 and 3 23 , and $60 at time 4.

Exercise 1.47: [ann15] Mary purchases an increasing annuity-immediate for $50,000


that makes twenty annual payments as follows:

8
Financial Mathematics  Exercises School of Risk and Actuarial Studies  UNSW

(i) P, 2P, . . . , 10P in years 1 through 10; and

(ii) 10(1.05)P, 10(1.05)2 P, . . . , 10(1.05)10 P in years 11 through 20.

The annual eective interest rate is 7% for the rst 10 years and 5% thereafter.
Calculate P.

Exercise 1.48: [new4] Suppose the interest rate is a constant 5% p.a. eective, and
the ination rate is a constant 3% p.a. Determine the (initial) annual payment from
a 20 year annuity-immediate which is purchased at the fair price with $10,000 in
the case of:

(a) a xed annuity (level payments)

(b) an ination-indexed annuity

(c) Show that (a) and (b) are equally fair. Explain your calculations.

Exercise 1.49: [new1] A perpetuity has annual payments (in arrears) of 1, 3, 6,


10, 15, etc. For a constant force of interest of δ = 0.05:

(a) Find the present value of the perpetuity analytically.

(b) Verify your answer by nding the approximate present value using Excel by
considering the 500 payments only. Plot the present value of each payment
against time t. Does this shape remind you of something?

Exercise 1.50: [annB10] Exercise 2.3.7 from Broverman 5th Ed (2.3.7 in 4th Ed).

Exercise 1.51: [loaB2] Exercise 2.3.8 from Broverman 5th Ed (2.3.8 in 4th Ed).

Exercise 1.52: [annB11] Exercise 2.3.15 from Broverman 5th Ed (2.3.15 in 4th
Ed).

Exercise 1.53: [loaB1] Exercise 2.3.19 from Broverman 5th Ed (2.3.19 in 4th Ed).

Exercise 1.54: [annB12] Exercise 2.3.20 from Broverman 5th Ed (2.3.20 in 4th
Ed).

Exercise 1.55: [annB13] Exercise 2.3.22 from Broverman 5th Ed (2.3.22 in 4th
Ed).

Exercise 1.56: [annB14] Exercise 2.3.23 from Broverman 5th Ed (2.3.23 in 4th
Ed).

Exercise 1.57: [annB15] Exercise 2.3.24 from Broverman 5th Ed (2.3.24 in 4th
Ed).

9
Financial Mathematics  Exercises School of Risk and Actuarial Studies  UNSW

Exercise 1.58: [new2] Find the present value of a 10-year increasing annuity that
pays at an annual rate of 100, 200, . . . , 1000, given that the annual eective interest
rate is 5% and:

(a) payments are made annually in arrears

(b) payments are made monthly in arrears

(c) payments are made continuously

Exercise 1.59: [ann11] A one-year deferred continuous varying annuity is payable


2
for 13 years. The rate of payment at time t is t − 1 per annum, and the force of
1
interest at time t is . Find the present value of the annuity.
1+t

10
Module 2

Loans and Investments

2.1 Loan Repayments

Exercise 2.1: [loa1] A bank decides to lend a company $15000 at a rate of interest
of 5% p.a. to be repaid by annual instalments over 5 years (in arrears).

(a) Calculate the annual payment.

(b) Calculate the loan outstanding at the end of the second payment using the
retrospective method.

(c) Calculate the loan outstanding at the end of the second payment using the
prospective method.

(d) At the end of the second year the bank tells you that from that time onwards
the rate of interest charged is going to be increased to 7.5% p.a. If you still
want to payo the loan by the end of the fth year what must your annual
payment change to?

(e) Having reviewed your company's free cash ows you decide that the amount
calculated in (d) is not aordable. You renegotiate with the bank and they
oer to extend your loan so that you can pay o the loan in an additional 4
years (instead of 3 years), at 7.6% p.a. What is the new annual repayment?

(f ) Using Excel, setup a loan schedule for part (e) above.

Exercise 2.2: [loa2] A loan of $20000 is to be repaid by 6 annual payments begin-


ning one year after the loan is made. The lender wants annual payments of interest
only at a rate of 7% and repayments of the principal in a single lump sum at the
end of 6 years. The borrower can accumulate the principal in a sinking fund earning
an annual interest rate of 5%, and decides to do this by means of 6 level deposits
starting one year after the loan is made.

(a) What should the annual payment be?

11
Financial Mathematics  Exercises School of Risk and Actuarial Studies  UNSW

(b) What if the sinking fund interest rate was 7%?

(c) Suppose you can decide whether you can setup a sinking fund arrangement
or to have a standard loan arrangement (ie. repay both capital and interest
with each payment) to repay the loan. If the sinking fund rate was 5% which
method would you prefer?
(Hint: you should not need to do any extra calculations to decide this  al-
though you can use it to check your answer if you wish)

(d) Model the cash ows of this sinking fund arrangement in a spreadsheet (for
the 5% case).

Exercise 2.3: [loa3] An individual borrows $5000 to buy a plasma TV. The sum
borrowed is repayable by 24 monthly instalments in arrears, which are calculated
on the basis of a at rate of interest of 10% p.a.

(a) Calculate the monthly repayment and the true (eective) annual rate of inter-
est being charged. Do this by hand using Newton-Raphson with 5 iterations
(starting at 10%), then using Excel with 10 iterations.

(b) Just after making the 12th repayment, the outstanding loan is to be repaid.
What is the outstanding balance which must be repaid at this time?

Exercise 2.4: [loaB3] Exercise 3.1.7 from Broverman 5th Ed (3.1.7 in 4th Ed).

Exercise 2.5: [loaB4] Exercise 3.1.8 from Broverman 5th Ed (3.1.8 in 4th Ed).

Exercise 2.6: [loaB5] Exercise 3.1.11 from Broverman 5th Ed (3.1.11 in 4th Ed).

Exercise 2.7: [loa5] A loan of $20000 is being repaid by monthly instalments of


1
principal and interest (18% p.a. nominal) over 8 years. Provide a schedule in
3
Excel showing the principal and interest contained in each of the last four monthly
instalments.

Exercise 2.8: [loa6] A householder is paying o four debts by monthly payments


all at an eective rate of 1% per month (12% p.a. nominal). The monthly payments
and respective terms to run are:

Monthly Payment ($) Terms to Run (Months)


4.36 11
17.20 15
35.00 12
20.24 18

The householder arranges a consolidation of these debts, with the total (sum) pay-
ments under the consolidated loan being equal to the total remaining payments
under the existing loans.

12
Financial Mathematics  Exercises School of Risk and Actuarial Studies  UNSW

Calculate the monthly instalment and the term to run of the consolidated loan so
that the eective rate of interest involved will be unchanged. Note that a nal
repayment may be required to ensure the loan is fully repaid. For this exercise, this
nal repayment is assumed to be made one month after the last monthly instalment.
You may nd Excel helpful in speeding up algebraic computation.

Exercise 2.9: [loa7] Paul takes out a loan of $47,500 to purchase a new car. The
interest applicable is 12% p.a. (monthly compounding). Instead of paying o the
loan using level instalments, he decides to pay it o using monthly payments over
3 years. The payments within each year is the same, but the payments in the 2nd
year are 10% higher than the payments in the 1st year, and the payments in the 3rd
year are 10% higher than the payments in the 2nd year. Set out the loan schedule
for this loan in Excel.

Exercise 2.10: [loa8] A recently married couple have decided to buy a new house
in Sydney. After an investigation of their nancial situation they nd that they will
need to borrow $600,000 from the bank. The rate of interest charged is 6.75% p.a
eective.

(a) If they want to pay o the loan in 10 years using annual payments, how much
would they have to pay in total over the 10 years?

(b) If they want to pay o the loan in 10 years using monthly payments, how much
would they have to pay in total over the 10 years?

(c) Suppose they choose to follow (a). At the end of year 5 (just after the payment
at time 5), interest rates increase to 7.25% p.a. eective. How much do they
need to pay to settle the loan at that time?

2.2 Investments

Exercise 2.11: [loa4] (McCutcheon & Scott, 1986, p. 158) A loan of $75,000 is to
be issued bearing interest at the rate of 8% per annum payable quarterly in arrears.
The loan will be repaid at par (ie. 100 per 100 face value) in 15 equal annual
instalments, with the rst instalment being repaid ve years after the issue date.

Find the price to be paid on the issue date by a purchaser of the whole loan who
wishes to realise a yield of (a) 10% per annum eective, and (b) 10% per annum
convertible half-yearly.

Exercise 2.12: [loa9] (McCutcheon & Scott, 1986, p. 197) A loan of nominal
amount $500,000 was issued bearing interest of 8% per annum payable quarterly in
arrears. The loan principal will be repaid at $105% by 20 annual instalments, each
of nominal amount $25,000, the rst repayment being ten years after the issue date.

An investor, liable to both income tax and capital gains tax, purchased the entire
loan on the issue date at a price to obtain a net eective annual yield of 6%. Assume

13
Financial Mathematics  Exercises School of Risk and Actuarial Studies  UNSW

that capital losses cannot oset capital gains for tax purposes. Find the price paid,
given that his rates of taxation for income and capital gains are:

(a) 40% and 30% respectively

(b) 20% and 30% respectively

Do this question in both Excel and R.

Exercise 2.13: [new13] Consider a 10-year bond of face value $100 and annual
coupons at a rate of 6%.

(a) Write down an equation of value given the price of the bond is $80.

(b) Using the Newton's Ralphson method in Excel, nd the yield to maturity of
the bond using a precision of 0.01%. Use any appropriate initial estimate.

Exercise 2.14: [loa10] (McCutcheon & Scott, 1986, p. 206) Two bonds (100 face
value) each have an outstanding term of four years. Redemption will be at par for
both bonds. Interest is payable annually in arrears at the annual rate of 15% for
the rst bond and 8% for the second bond. Interest payments have just been made
and the prices of the bonds are $105.80 and $85.34 respectively.

(a) Verify that an investor, liable for income tax at the rate of 35% and capital
gains tax at the rate of 50% who purchases either of these bonds (but not
both) will obtain a net yield on his transaction of 8% per annum.

(b) Assume now that the investor is allowed to oset capital gains by capital
losses. Show that, if the proportion of his available funds invested in the 8%
bond is such that the overall capital gain is zero, he will achieve a net yield of
combined transaction of 8.46% per annum.

Exercise 2.15: [loaB6] Exercise 4.3.4 from Broverman 5th Ed (4.3.4 in 4th Ed).

Exercise 2.16: [loaB7] Exercise 4.3.1 from Broverman 5th Ed (4.3.1 in 4th Ed).

Exercise 2.17: [loa11] An investor purchased an Australian Government bond on


11 June 2006 paying a coupon 5.75% p.a with a maturity of 15 June 2011. The bond
is ex-interest within 7 days prior to the coupon payment. Explain what is meant
by ex-interest for an Australian government bond and describe the payments that
the buyer will receive on an ex-interest Australian government bond. Determine
the price paid for the Australian Government bond at a yield of 4.75% p.a on 11
June 2006.

Exercise 2.18: [ann16] Outline the payments made on ination indexed bonds and
give an example of an investor who would invest in these nancial instruments.

14
Financial Mathematics  Exercises School of Risk and Actuarial Studies  UNSW

Exercise 2.19: [loa12] A loan of nominal amount $500,000 was issued bearing in-
terest of 8% per annum payable quarterly in arrears. The loan will be repaid at
$110% by 10 annual instalments, each of nominal amount $50,000, the rst repay-
ment being ten years after the issue date. An investor, liable to both income tax and
capital gains tax, purchased the entire loan on the issue date at a price to obtain a
net eective annual yield of 7%. Find the price paid, given that his rates of taxation
for income and capital gains are both 15%. Do this question in Excel.

Exercise 2.20: [loa13] A loan, on which interest is payable half-yearly, was issued
on 1 January 1974. The loan was to be redeemed with deferred annual payments
(always on 1 January) in accordance with the following schedule:

Amount redeemed Redemption


in each year rate
1 Jan 1984 to 1 Jan 1992 (inclusive) $150 000 105%
1 Jan 1993 to 1 Jan 2003 (inclusive) $250 000 110%
1 Jan 2004 $300 000 112%

Interest is payable at the rate of 7% p.a. until the payment on 1 July 2000 has been
made and thereafter at 8% p.a. What was the issue price if a purchaser of the whole
loan secured a yield of 6.5% p.a. eective on his or her investment? Do this question
using Excel.

Exercise 2.21: [loa14] A loan of nominal amount $1,200 was issued bearing interest
of 10% per annum payable annually in arrears. The loan will be repaid by 3 annual
nominal payments of equal value, the rst repayment being two years after the issue
date. The actual repayment will be at $100% for the rst two instalments, and
$120% for the nal instalment. An investor, liable to both income tax and capital
gains tax at 20%, purchased the entire loan on the issue date at a price to obtain a
net eective annual yield of 8%. Find the price paid, given that it is greater than
$1,200.

15
Module 3

Interest Rate Risk

3.1 Term Structure of Interest Rates

Exercise 3.1: [irr1]

Consider the following spot interest rates that are quoted on a nominal p.a. basis
(2)
assuming interest compounds semi-annually (ie. they are i interest rates).

Term (Years) % p.a.


0.5 4.875180
1.0 5.031182
1.5 5.234408
2.0 5.448436

(a) Use these spot rates to calculate the value of a 6.75% bond paying semi-annual
coupons maturing in two years time with a face value of $100.

(b) Calculate the yield to maturity on this bond for the price calculated above.

(c) Determine the par yield, as a semi-annual compounding yield, for one year
and two year maturity bonds corresponding to the above rates. Interpret your
result.

(d) Determine the 6 month forward rates corresponding to these spot rates.

Exercise 3.2: [irr2] Consider two 5 year bonds. One has a 9% coupon and sells for
101.00; the other has a 7% coupon and sells for 93.20. Find the price of a 5 year
zero coupon bond.

Exercise 3.3: [irr3] Let s(t), 0 ≤ t ≤ ∞ denote a spot rate curve, that is, the
present value of a dollar to be received at time t is e−s(t)t . Show explicitly that if the
spot rate curve is at and that s(t) = r, then all forward rates must be the same.

16
Financial Mathematics  Exercises School of Risk and Actuarial Studies  UNSW

Exercise 3.4: [irr4] The half year forward rates are as follows (semi-annual com-
pounding):

Time Period % p.a


0  0.5 5.00
0.5  1 5.50
1  1.5 6.00
1.5  2 6.10
2  2.5 6.25
2.5  3 7.00

Calculate the 1 year forward rates for time periods 0  1, 0.5  1.5, 1  2, 1.5  2.5,
2  3.

Exercise 3.5: [irr5]

Consider the following spot rates (semi-annual compounding):

Term (Years) % p.a.


0.5 4.5000
1.0 5.2500

and forward rates:

Time Period % p.a.


1-1.5 7.5082
1.5-2 2.0290

Calculate the value of a bond paying semi-annual coupons of 8% p.a., maturing in


2 years time.

Exercise 3.6: [irr13] (Institute and Faculty of Actuaries CT1 April 2015) A 9-
month forward contract is issued on 1 April 2015 on a stock with a price of $6 per
share on that date. Dividends are assumed to be received continuously and the
dividend yield is 3.5% per annum.
Calculate the theoretical forward price per share of the contract, assuming no arbi-
trage and a risk-free force of interest of 9% per annum.

Exercise 3.7: [irr14] (Institute and Faculty of Actuaries CT1 Sep 2015) An investor
wishes to obtain a rate of interest of 3% per annum eective from a 91-day treasury
bill.
Calculate:

(a) the price that the investor must pay per $100 nominal.

(b) the annual simple rate of discount from the treasury bill.

17
Financial Mathematics  Exercises School of Risk and Actuarial Studies  UNSW

Exercise 3.8: [irr15] (Institute and Faculty of Actuaries CT1 Sep 2015) Three
bonds, each paying annual coupons in arrear of 3% and redeemable at $100 per
$100 nominal, reach their redemption dates in exactly one, two and three years'
time, respectively. Each of these bonds is currently selling at $101.

(a) Calculate the one-year, two-year and three-year spot rates of interest implied
by the information given.

(b) Calculate the one-year forward rate starting from the end of the second year,
f2,3

Exercise 3.9: [irr16] (Institute and Faculty of Actuaries CT1 Apr 2016) At time
t = 0, the one-year zero-coupon yield is 4% per annum eective and the one-year
forward rate per annum eective at time t (t = 1, 2, . . .) is given by:

ft,t+1 = (4 + t)%

(a) Determine the issue price per $100 nominal of a three-year 4% coupon bond
issued at time t = 0, paying coupons annually in arrear and redeemable at
105%.

(b) Determine the three-year par yield at time t = 0.

Exercise 3.10: [irr17] (Institute and Faculty of Actuaries CT1 Sep 2016) A zero-
coupon bond was issued on 1 January 1975 with a redemption date of 1 January
2015. An investor bought the bond to provide a yield to maturity of 5% per annum
convertible half yearly. On a particular date the borrower defaulted, repaying 80%
of the capital to all bondholders. The investor obtained a rate of return until the
date of default which was equivalent to a force of interest of 4.8% per annum.
Determine the date on which the borrower defaulted.

Exercise 3.11: [irr18] (Institute and Faculty of Actuaries CT1 Sep 2016) At the
beginning of 2015 a 182 day commercial bill, redeemable at $100, was purchased for
$96 at the time of issue and later sold to a second investor for $97.50.
The initial purchaser obtained a simple rate of interest of 3.5% per annum before
selling the bill.

(a) Calculate the annual simple rate of return which the initial purchaser would
have received if they had held the bill to maturity.

(b) Calculate the length of time in days for which the initial purchaser held the
bill.

(c) The second investor held the bill to maturity. Calculate the annual eective
rate of return achieved by the second investor.

18
Financial Mathematics  Exercises School of Risk and Actuarial Studies  UNSW

Exercise 3.12: [irr19] (Institute and Faculty of Actuaries CT1 Apr 2011) The
n − year spot rate of interest yn is given by:

n
yn = 0.03 + for n = 1, 2, 3, 4
1000

(a) Calculate the implied one-year and two-year forward rates applicable at time
t=2

(b) Calculate, assuming no arbitrage

1. The price at time t=0 per $100 nominal of a bond which pays annual
coupons of 4% in arrear and is redeemed at 115% after 3 years.

2. The 3-year par yield

Exercise 3.13: [irr20] (Institute and Faculty of Actuaries CT1 Apr 2008) The n-
year spot rate of interest, in , is given by:

in = a − bn

for n = 1, 2, 3 where a and b are constants.

The one-year forward rates applicable at time 0 and at time 1 are 6.1% per an-
num eective and 6.5% per annum eective respectively. The 4-year par yield is 7%
per annum.

Stating any assumptions:

(a) calculate the values of a and b

(b) calculate the price per $1 nominal at time 0 of a bond which pays annual
coupons of 5% in arrear and is redeemed at 103% after 4 years.

3.2 Price Sensitivity

Exercise 3.14: D(δ) be the duration, at a constant force of interest


[irr6] Let
(continuous compounding) δ p.a., of a xed-interest security with interest payable
continuously at the annual rate D per unit nominal and redeemable at R per unit
nominal in n years time. Let g = D/R. Show that:

¯ n + nv n
g(Iā)
D(δ) =
gān + v n

Exercise 3.15: [irr7] Consider a xed-interest security bearing interest of 5% p.a.


payable continuously and redeemable at par in n years time, where n is not neces-
sarily an integer. Assuming a constant continuously compounding force of interest
of 7% p.a:

19
Financial Mathematics  Exercises School of Risk and Actuarial Studies  UNSW

(a) Determine the duration of the security for n = 20 and n = 60.

(b) Note that the duration of the security, on the basis of a specied constant
force of interest per annum δ , may be considered as a function of n. Assuming
δ = 0.07, show that the duration is maximised when the following equation is
satised:
0.07
0.07ān + 0.05(n − ān ) =
0.07 − 0.05
Hence (or otherwise) nd the maximum duration and the corresponding value
of n at which the duration is maximised.

Exercise 3.16: [new11] Consider a level 10-year annuity-immediate paying $1 at


the end of each year.

(a) Write down the expressions which relate the modied duration and convexity
with derivatives of the price (present value).

(b) For i = 5%, nd the modied duration and the convexity.

(c) Using Excel and a Taylor's approximation, nd an approximate modied du-
ration and convexity. Compare this with the answer in (b).

Exercise 3.17: [lif6] Show that the Macaulay Duration of äx is equal to:


X
Dx = wk k
k=0

where wk is given by:


vk k p
wk = P∞ xl
l=0 v l px

Exercise 3.18: [dur1] (Institute and Faculty of Actuaries CT1 Sep 2015) An insur-
ance company has sold a pension product to an individual. Under the arrangement,
the individual is to receive an immediate annuity of $500 per year annually in arrear
for 12 years. The insurance company has invested the premium it has received in
a xed-interest bond that pays coupons annually in arrear at the rate of 5% per
annum and which is redeemable at par in exactly eight years.

(a) Calculate the duration of the annuity at an interest rate of 4% per annum
eective.

(b) Calculate the duration of the bond at an interest rate of 4% per annum eec-
tive.

(c) State with reasons whether the insurance company will make a prot or a loss
if there is a small increase in interest rates at all terms.

20
Financial Mathematics  Exercises School of Risk and Actuarial Studies  UNSW

Exercise 3.19: [dur2] (Institute and Faculty of Actuaries CT1 Sep 2016) The gov-
ernment of a heavily indebted country has a range of bonds currently in issue. These
include bonds with nominal amounts outstanding of $4bn and $5bn with terms to
redemption of exactly three years and ten years respectively from the current time.
Both bonds pay annual coupons in arrear of 4%.
The government is negotiating a restructuring of its debt portfolio and proposes to
transform the three and ten year bonds into perpetuities paying an annual coupon
of 5% in arrear. The yield curve is currently at with gross redemption yields at 6%
per annum eective.

(a) Calculate, showing all workings, the duration of the current portfolio of three
year and ten-year bonds.

(b) Calculate, showing all workings, the duration of the proposed portfolio of
bonds.

(c) The government's objective is that the present value of the proposed portfolio
of bonds will be 80% of the present value of the current portfolio of three-year
and ten year bonds.
Determine the nominal amount of the new bonds that the government will
have to issue to achieve the objective.

Exercise 3.20: [dur3] (Institute and Faculty of Actuaries CT1 Apr 2010) A pension
fund has to pay out benets at the end of each of the next 40 years. The benets
payable at the end of the rst year total $1 million. Thereafter, the benets are
expected to increase at a xed rate of 3.8835% per annum compound.
Calculate the discounted mean term of the liabilities using a rate of interest of 7%
per annum eective.

Exercise 3.21: [dur4] (Institute and Faculty of Actuaries CT1 Apr 2012) A com-
pany has the following liabilities:

• annuity payments of $200,000 per annum to be paid annually in arrear for the
next 20 years

• a lump sum of $300,000 to be paid in 15 years

The company wishes to invest in two xed-interest securities in order to immunise


its liabilities.

Security A has a coupon rate of 9% per annum and a term to redemption of 12


years.
Security B has a coupon rate of 4% per annum and a term to redemption of 30 years.

Both securities are redeemable at par and pay coupons annually in arrear. The
rate of interest is 8% per annum eective.

21
Financial Mathematics  Exercises School of Risk and Actuarial Studies  UNSW

(a) Calculate the present value of the liabilities.

(b) Calculate the duration of the liabilities.

(c) Calculate the nominal amount of each security that should be purchased so
that Redington's rst two conditions for immunisation against small changes
in the rate of interest are satised for this company.

(d) Describe the further calculations that will be necessary to determine whether
the company is immunised against small changes in the rate of interest.

Exercise 3.22: [dur5] (Institute and Faculty of Actuaries CT1 Sep 2013) A pension
fund has liabilities to meet annuities payable in arrear for 40 years at a rate of $10
million per annum.
The fund is invested in two xed-interest securities. The rst security pays annual
coupons of 5% and is redeemed at par in exactly ten years' time. The second
securitypays annual coupons of 10% and is redeemed at par in exactly ve years'
time. The present value of the assets in the pension fund is equal to the present
value of the liabilities of the fund and exactly half the assets are invested in each
security. All assets and liabilities are valued at a rate of interest of 4% per annum
eective.

(a) Calculate the present value of the liabilities of the fund.

(b) Calculate the nominal amount held of each security purchased by the pension
fund.

(c) Calculate the duration of the liabilities of the pension fund.

(d) Calculate the duration of the assets of the pension fund.

(e) Without further calculations, explain whether the pension fund will make a
prot or loss if interest rates fall uniformly by 1.5% per annum eective.

Exercise 3.23: [dur6] (Institute and Faculty of Actuaries CT1 Sep 2009) A member
of a pensions savings scheme invests $1,200 per annum in monthly instalments, in
advance, for 20 years from his 25th birthday. From the age of 45, the member
increases his investment to $2,400 per annum. At each birthday thereafter the
annual rate of investment is further increased by $100 per annum. The investments
continue to be made monthly in advance for 20 years until the individual's 65th
birthday.
At the age of 65, the scheme member uses his accumulated investment to purchase
an annuity with a term of 20 years to be paid half-yearly in arrear. At this time the
interest rate is 5% per annum convertible half-yearly.

(a) Calculate the accumulation of the investment at the age of 65 using a rate of
interest of 6% per annum eective.

(b) Calculate the annual rate of payment of the annuity.

(c) Calculate the duration of the annuity.

22
Financial Mathematics  Exercises School of Risk and Actuarial Studies  UNSW

3.3 Immunisation

Exercise 3.24: [irr8] (Boyle, 1992) Suppose the term structure of spot rates is level
for all maturities and equal to 8% p.a. Suppose that in the next instant, the term
structure of interest rates will be either 9% p.a. for all maturities or 7% p.a. for all
maturities. Consider the following strategy. An investor goes short a zero coupon
bond with a 10-year maturity and a face value of 1000. Simultaneously, she uses
the proceeds to purchase a 5-year zero coupon bond with maturity value M5 and a
15-year maturity bond with maturity value M15 .

(a) Give expressions for the value of assets VA (i) and the value of liabilities VL (i).

(b) What is meant by an arbitrage opportunity?

(c) By suitable choice of M5 and M15 such that VA (.08) − VL (.08) is zero, demon-
strate that arbitrage prots are possible with parallel shifts to a at yield
curve.

Exercise 3.25: [irr9] Consider 3 coupon paying bonds x, y and z. You have
calculated their Price to be (Px , Py , Pz ), and their duration and convexity to be
(Dx , Dy , Dz ) and (Cx , Cy , Cz ) respectively. Consider a portfolio by buying 1 unit
of each bond. Derive a formula for the duration and convexity for the portfolio in
terms of the price, duration, and convexity of the individual bonds.

Exercise 3.26: [irr10] Consider a portfolio of insurance liabilities. Your best esti-
mate of the future outgoes (claims) are as follows

Time Outgo
1 3m
2 4m
3 3m
4 2m

Assume that the spot rate term structure is at and equal to 4.5%. Assume that
the insurer can only invest in 2 ZCBs. One matures in 0.5 years while the other
matures in 5 years. Find an immunisation strategy using the two bonds.

Using Excel, analyse:

(a) What happens to the surplus if the yield shifts in a parallel fashion to 6.5%?

(b) What happens to the surplus if the yield shifts in a parallel fashion to 2.5%?

(c) What happens to the surplus if the yield curve twists and you are faced with
the following spot rate curve?

23
Financial Mathematics  Exercises School of Risk and Actuarial Studies  UNSW

Time Rate
0.5 3%
1 3.5%
1.5 4%
2 4.5%
2.5 5%
3 5.5%
3.5 6%
4 6.5%
4.5 7%
5 7.5%

(d) Determine the surplus if we are faced with a term structure where the t-year
spot rate is given by:

1 − e−λt 1 − e−λt
   
st = α + β +γ − e−λt
λt λt

where α = 0.06, β = 0.01, γ = −0.08, and λ = 0.6 are known parameters.

Exercise 3.27: [new12] The Nelson-Siegel class of term structure models are com-
monly used in practice to model the yield curve through time. One example is the
model used in Exercise 3.26(e). Another simple example is the following yield curve,
which describes the zero coupon yield of maturity τ (ie. τ -year spot rate) as:

1 − e−λτ
 
s τ = α + βt
λτ

where α and λ are constant parameters and βt is a time-varying parameter. It is


assumed that α = 0.06 and λ = 0.6, while the values of βt for each time t are
2
independent and identically distributed with βt ∼ N (−0.01, 0.002 ).

(a) Using Excel, plot the spot curve (for maturities 0 < τ < 20) for the given
parameters and when βt is equal to its mean of -0.01.

(b) Why is the use of Fisher-Weil duration more realistic?

(c) Consider the portfolio of insurance liabilities in Exercise 3.26, and again as-
sume that the insurer can only invest in 2 ZCBs of maturity 0.5 and 5 years
respectively. Find an immunisation strategy if the current value of βt is -0.01.

(d) Simulate 1000 outcomes for the yield curve in the next moment and determine
the surplus in each case. Also plot a histogram for the surplus. Is your portfolio
fully immunised? Why?

Exercise 3.28: [irr11] Consider a portfolio of insurance liabilities. Your best esti-
mate of the future outgoes (claims) are as follows

24
Financial Mathematics  Exercises School of Risk and Actuarial Studies  UNSW

Time Outgo
1 3m
2 4m
3 3m
4 2m

Assume that the spot rate is at and is equal to 4%. Suppose the insurer has
available for investment two coupon bonds. One is a 4% coupon bond with 0.75
years till maturity. The second bond is a 8% coupon bond with 8 years till maturity.
Find an immunisation strategy using the two bonds. (Derive your solutions without
using Excel.)

Exercise 3.29: [irr12] Suppose you have a liability of $2 million due at time 3. As-
sume that the spot rate is at and is equal to 6%. You have available for investment
ve ZCBs, with maturities at 1, 2, 3, 4 and 5 respectively.

(a) Suppose you wish to use an immunisation strategy using 2 bonds. Derive the
portfolio.

(b) Explain the term cash ow matching. Derive the portfolio (using 1, 2, 3, 4 or
all 5 bonds) that corresponds to a cash ow matching strategy.

(c) Suppose the spot rate moves to 7% at. What happens to your surplus for
the strategies in (a) and (b)?

(d) Suppose the spot rate curve such that the spot rate for maturity T is equal to
(3 + T )%. What happens to your surplus for the strategies in (a) and (b)?

Exercise 3.30: [imm1] (Institute and Faculty of Actuaries CT1 Apr 2016) An
insurance company has liabilities of $6 million due in exactly 8 years' time and a
further $11 million due in exactly 15 years' time. Its assets are:

• a 5-year zero-coupon bond of nominal amount $5.5088 million

• a 20-year zero-coupon bond of nominal amount $13.7969 million

The current rate of interest is 8% per annum eective at all durations.

(a) Show that the rst two conditions of Redington's theory for immunisation
against small changes in the rate of interest are satised.

(b) Explain, without doing any further calculations, whether the insurance com-
pany will be immunised against small changes in the rate of interest.

Exercise 3.31: [imm2] (Institute and Faculty of Actuaries CT1 Sep 2016) A pen-
sion fund has liabilities of $10m to meet at the end of each of the next ten years. It
is able to invest in two zero-coupon bonds with a term to redemption of three years
and 12 years respectively. The rate of interest is 4% per annum eective.

Calculate:

25
Financial Mathematics  Exercises School of Risk and Actuarial Studies  UNSW

(a) the present value of the liabilities of the pension fund

(b) the duration of the liabilities of the pension fund

(c) the nominal amount that should be invested in the zero-coupon bonds to
ensure that the present values and durations of the assets and liabilities is the
same

Exercise 3.32: [imm3]

(a) State the three conditions that are necessary for a fund to be immunised from
small, uniform changes in the rate of interest.

(b) Comment on the practical usefulness of the theory of immunisation

26
Module 4

Derivatives

4.1 Forwards, Futures and Swaps

Exercise 4.1: [der1] On 12 May 1987, the closing value of the S&P 500 Index was
293.3 and the December 1987 S&P 500 futures closing index, with delivery in 210
days, was 299.0. Calculate the theoretical futures index assuming transactions and
storage costs are negligible, a constant annual continuously compounding interest
rate of 7%, and that the S&P 500 portfolio pays dividends continuously at an annual
rate of 3.5% of its market value on 12 May 1987. You may also assume that interest
rates are deterministic so that the futures price is equal to the theoretical forward
price.

Exercise 4.2: [der2] Consider a forward contract to buy 6.5% coupon 6 year Trea-
sury bonds in 2 years time (immediately after the coupon then due has been paid).
These bonds are assumed to be currently available as 6.5% 8 year Treasury bonds
at a yield of 6.96% p.a. (semi-annual). Funding costs for the rst year are 6.5% p.a.
(monthly compounding) and 7% p.a. (monthly compounding) for the second year.
Determine the forward price and forward yield in two years time.

Exercise 4.3: [der3] An investor holds a short position in a forward contract on


gold for delivery in 90 days at $450 an ounce. The current spot price of gold is $420
an ounce and insurance and storage cost for gold are 2.5% p.a of the spot price, paid
on delivery. Ninety day (simple) interest rates are 9.75% p.a. What is the value of
this forward contract?

Exercise 4.4: [der4] Consider a forward contract on 10000 shares, deliverable in 6


months time. The share is currently trading at $10.00. Assume that there will be a
dividend payment of 0.40 per share in 3 months time. Funding costs for 6 months
are 6% p.a. (monthly compounding). Transaction costs are 2% of the value of the
shares purchased. Determine the forward price for sale of the shares in 6 months at
which all net funding and other costs will be covered.

27
Financial Mathematics  Exercises School of Risk and Actuarial Studies  UNSW

Exercise 4.5: [der5] Explain the cost of carry formula:

Ft,T = St er(T −t) − der(T −t1 )

There are no storage costs but there is a payment to the holder of the spot asset
(eg. dividend) of d at time t1 .

Exercise 4.6: [der6] Suppose the current spot and forward rates are as given in
Table 6.4 of Sherris (1996, p. 109). Calculate the implied repo rate (the risk-free rate
implied by current spot and forward prices) associated with each forward contract
(corresponding to the two 90-day forward rates). Also, for each forward contract,
outline an arbitrage strategy you could use to realise your arbitrage prot now. Give
the amount of the prot in both instances. Use simple interest as the contracts are
for terms less than one year.

Exercise 4.7: [der7] Review Example 7.1 of Sherris (p. 133)

Exercise 4.8: [der8] Review Example 7.4 of Sherris (p. 138)

Exercise 4.9: [der9] Review Example 7.10 of Sherris (p. 149)

Exercise 4.10: [der10] Consider an agreement where party A receives the spot
price for N units of a commodity each period while paying a xed amount X per
unit for N units. If the agreement is made for M periods (ie. at times t1 , t2 , .., tM ),
derive a formula that can be used to determine the swap price X on inception of the
swap contract. Assume that no storage costs or dividends occur during the period,
and the risk free interest rate is r (continuous compounding).

Exercise 4.11: [swp1](Oh Kang Kwon, 2004) Companies X and Y have been of-
fered the following rates per annum on a $10 million 5-year investment:

Fixed Floating
X 5.0% LIBOR +0.1%
Y 6.4% LIBOR +0.6%

If X wants a oating rate investment and Y wants a xed rate investment, then
design a swap that will net a nancial intermediary 0.1% per annum and appear
equally attractive to both companies.

Exercise 4.12: [for1] (Institute and Faculty of Actuaries CT1 Sep 2015) A nine-
month forward contract was issued on 1 October 2015 on a share with a price at
that date of $10. Dividends of 50 cents per share are expected on 1 November 2015
and 1 May 2016. The risk-free force of interest is 5% per annum.

(a) Calculate the forward price at issue, stating any further assumptions made
and showing all workings.

28
Financial Mathematics  Exercises School of Risk and Actuarial Studies  UNSW

(b) Explain why the expected price of the share nine months after issue does not
have to be taken into account when pricing the forward.

Exercise 4.13: [for2] (Institute and Faculty of Actuaries CT1 Apr 2015) A 9-month
forward contract is issued on 1 April 2015 on a stock with a price of $6 per share
on that date. Dividends are assumed to be received continuously and the dividend
yield is 3.5% per annum. The actual forward price per share of the contract is $6.30
and the risk-free force of interest is 9% per annum.

(a) Describe what is meant by the no arbitrage assumption in nancial mathe-
matics.

(b) Calculate the theoretical forward price per share of the contract, assuming no
arbitrage

(c) Outline how an investor could make an arbitrage prot

Exercise 4.14: [for3] (Institute and Faculty of Actuaries CT1 Sep 2010) A six-
month forward contract was issued on 1 April 2009 on a share with a price of $700
at that date. It was known that a dividend of $20 per share would be paid on 1 May
2009. The one-month spot, risk-free rate of interest at the time of issue was 5% per
annum eective and the forward rate of interest from 1 May to 30 September was
3% per annum eective.

(a) Calculate the forward price at issue, assuming no arbitrage, explaining your
working.

(b) It has been suggested that the forward price cannot be calculated without
making a judgement about the expected price of the share when the forward
contract matures. Explain why this statement is not correct.

(c) Comment on whether the method used in part (a) would still be valid if it was
not known with certainty that the dividend due on 1 May 2009 would be paid.

Exercise 4.15: [for4] (Institute and Faculty of Actuaries CT1 Apr 2011) A one-year
forward contract on a stock is entered into on 1 January 2011 when the stock price
is $68 and the risk-free force of interest is 14% per annum. The stock is expected to
pay an annual dividend of $2.50 with the next dividend due in eight months' time.
On 1 April 2011, the price of the stock is $71 and the risk-free force of interest is
12% per annum. The dividend expectation is unchanged.
Calculate the value of the contract to the holder of the long forward position on 1
April 2011.

4.2 Options

Exercise 4.16: [der11] In a one-period binomial model, it is assumed that the


current share price of 260 will either increase to 285 or decrease to 250 at the end

29
Financial Mathematics  Exercises School of Risk and Actuarial Studies  UNSW

of one year. The annual risk-free interest rate is 5% compounded continuously and
assume that this share pays no dividends.

(a) Calculate the price of a one-year European call option with a strike price of
275 by replicating the payo with a portfolio of shares and bonds.

(b) Calculate the price of a one-year European put option with a strike price of
275 by replicating the payo with a portfolio of shares and bonds.

(c) Verify numerically that the put-call parity relationship holds in this case.

Exercise 4.17: [der12] Assume that the stock price is currently $50, and will in-
crease or decrease by 10% at the end of the month. The interest rate is 5% p.a.
(simple). Find the price of a call option with a strike price of $50.

Exercise 4.18: [der13] In the pricing of forwards, futures, swaps and options, the
expected value of the underlying asset for these contracts has not appeared in the
valuation. Explain why this is the case. (Do you nd it surprising?)

Exercise 4.19: [der14] In a one-period binomial model, it is assumed that the


current share price of 260 will either increase to 285 or decrease to 250 at the end
of one year. The annual risk-free interest rate is 4% compounded continuously and
assume that this share pays no dividends. Calculate the price of an option that pays
the cash dierence between the square of the share price at the end of the year and
70225, provided that the dierence is positive (ie. otherwise it pays nothing).

Exercise 4.20: [der15] The current stock price is $20, and the risk free rate (simple)
is 5% p.a. One year call and put options with strike price $22 are priced at $1.2245
and $2.5000 respectively. Verify that there is an arbitrage opportunity, and identify
the transactions required.

Exercise 4.21: [der16] Suppose we want to price a call option on a share using
a binomial model of the share price. Consider a portfolio of the share and bond.
Suppose however that the stock pays a xed dividend of $D on the maturity date
of the option (ie at time T) and that the owner of the option will not receive the
share dividend. Derive a formula for the number of stocks and bond that need to
be held to replicate this option payo.

Exercise 4.22: [der17] Consider call and put (European) options (with the same
strike price) on gold. The spot price of gold is G(0) at time 0, and the maturity of
the option is at time T. Storage costs of $c per unit of gold are payable at time T.
By equating the cost at time 0 of two portfolios that have the same payo at time
T, nd an updated version of the put-call parity that takes into account the storage
costs.

Exercise 4.23: [opt1] Identify the prot or loss, ignoring any trading costs, in each
of the following scenarios:

30
Financial Mathematics  Exercises School of Risk and Actuarial Studies  UNSW

(a) A call option with an exercise price of 480 is bought for a premium of 37. The
price of the underlying share is 495 at the expiry date.

(b) A put option with an exercise price of 180 is bought for a premium of 12. The
price of the underlying share is 150 at the expiry date.

(c) A put option with an exercise price of 250 is written for a premium of 22. The
price of the underlying share is 272 at the expiry date.

Exercise 4.24: [opt2] A European call option and a European put option each have
a strike price of $105.

(a) Suppose share price at expiry is assumed to be uniformly distributed between


90 and 110. Calculate the expected payos for both these options.

(b) Suppose share price at expiry is assumed to be uniformly distributed between


80 and 120. Calculate the expected payos for both these options

(c) Comment on the dierence between the results above.

(d) Calculate the dierence between the expected payo of the put and the ex-
pected payo of the call in (a) and (b) above. Comment on your answers.

Exercise 4.25: [opt3] A three-month European call option with an exercise price
of 500 on a share whose current price is 480 is currently priced at 42.

(a) What would you expect the price to be for a 3-month put option with the same
exercise price if the risk-free interest rate is 6% compounded continuously and
no dividends are payable during the life of the option?

(b) What assumptions did you have to make to come up with the answer above?

Exercise 4.26: [opt4] How would you expect the price of a European put option
on a non-dividend paying share to change (justify your answers) if:

(a) the current share price fell

(b) there was a sudden increase in the risk free interest rate.

Exercise 4.27: [opt5] (Institute exam CT8 April 2015) Consider a non-dividend
paying share with price St at time t. Two options written have the following char-
acteristics:

• a European call option maturing in two years, strike price $10.15, option price
$3.87

• a European put option maturing in two years, strike price $10.15, option price
$0.44

31
Financial Mathematics  Exercises School of Risk and Actuarial Studies  UNSW

The continuously compounded risk-free rate of interest is 4% p.a.

(a) State and prove the put-call parity relationship for this share.

(b) Calculate the share price implied by the option prices.

Exercise 4.28: [opt6] How might you construct a:

(a) A bull spread with call options

(b) A bull spread with put options

(c) A bear spread with call options

(d) A bear spread with put options

(e) A straddle

32
Module 5

Stochastic Interest Rates

5.1 IID Returns

Exercise 5.1: [sto1] (Institute of Actuaries Examination April 2003) $1000 is in-
vested for 10 years. In any year, the yield on the investment will be 4% with prob-
ability 0.4, 6% with probability 0.2, 8% with probability 0.4, and is independent of
the yield in any other year.

(a) Calculate the mean accumulation at the end of 10 years.

(b) Calculate the standard deviation of the accumulation at the end of 10 years.

(c) Without carrying out any further calculations, explain how your answers to
(a) and (b) would change (if at all) if:

(i) the yields had been 5%, 6% and 7% instead of 4%, 6%, and 8% p.a.
respectively.

(ii) the investment had been made for 12 years instead of 10 years.

Exercise 5.2: [sto2] (Institute of Actuaries Examination September 2003) In any


year t, the yield on a fund of investments has mean jt and standard deviation st . In
any year, the yield is independent of the yield in any other year. The accumulated
value, after n years, of a unit sum of money invested at time 0 is Sn .

(a) Derive formulae for the mean and variance of Sn if jt = j and st = s for all
years t.

(b) Calculate the expected value of S8 if j = 0.06.

(c) Calculate the standard deviation of S8 if j = 0.06 and s = 0.08.

Exercise 5.3: [sto3] (Institute of Actuaries Examination September 2002) $10,000


is invested in a bank account which pays interest at the end of each year. The rate
of interest is xed randomly at the beginning of each year and remains unchanged

33
Financial Mathematics  Exercises School of Risk and Actuarial Studies  UNSW

until the beginning of the next year. The rate of interest applicable in any one year
is independent of the rate applicable in any other year.

During the rst year, the rate of interest per annum eective will be one of 3%,
4% or 6% with equal probability. During the second year, the rate of interest per
annum eective will be either 5% with probability 0.7, or 4% with probability 0.3.

(a) Assuming that interest is always reinvested in the account, calculate the ex-
pected accumulated amount in the bank account at the end of two years.

(b) Calculate the variance of the accumulated account in the bank account at the
end of two years.

Exercise 5.4: [sto4] (Institute of Actuaries Examination April 2005) In any year,
the interest rate per annum eective on monies invested with a given bank is equally
likely to be i1 or i2 (i1 > i2 ), and is independent of the interest rates in all previous
years.

(a) Express the mean and variance of the eective rate in a particular year in
terms of i1 and i2 .

(b) The accumulated value at time t = 25 years of $1 million invested with the
bank at time t = 0 has expected value $5.5 million and standard deviation
$0.5 million. Find i1 and i2 .

Exercise 5.5: [sto5] (Institute of Actuaries Examination September 2000) An in-


surance company calculates the single premium for a contract paying $10,000 in ten
years time as the present value of the benet payable at the expected rate of interest
it will earn on its funds. The annual eective rate of interest over the whole of the
next ten years will be 7%, 8% or 10% with probabilities 0.3, 0.5 and 0.2 respectively.

(a) Calculate the single premium.

(b) Calculate the expected prot at the end of the term of the contract.

Exercise 5.6: [iid1] (Institute and Faculty of Actuaries CT1 Sep 2010) The annual
rates of return from an asset are independently and identically distributed.
The expected accumulation after 20 years of $1 invested in this asset is $2 and the
standard deviation of the accumulation is $0.60.

(a) Calculate the expected eective rate of return per annum from the asset, show-
ing all the steps in your working.

(b) Calculate the variance of the eective rate of return per annum.

Exercise 5.7: [iid2] (Institute and Faculty of Actuaries CT1 Apr 2017) An individ-
ual aged exactly 65 intends to retire in ve years' time and receive an annuity-certain.
The annuity will be payable monthly in advance and will cease after 20 years. The

34
Financial Mathematics  Exercises School of Risk and Actuarial Studies  UNSW

annuity will increase at each anniversary of the commencement of payment at the


rate of 3% per annum.
The individual would like the initial level of annuity to be $20,000 per annum. The
price of the annuity will be the present value of the payments on the date it com-
mences using an interest rate of 7% per annum eective.
In order to purchase the annuity, the individual invests $200,000 on his 65th birth-
day in a particular fund.
The investment return on the fund in any given year is independent of returns in all
other years and he annual return is:

• 4% with a probability of 60%

• 7% with a probability of 40%

(a) Calculate the price of the annuity.

(b) Calculate, showing all workings, the expected accumulation of the investment
at the time of retirement.

(c) Calculate, showing all workings, the standard deviation of the investment at
the time of retirement.

(d) Determine the probability that the individual will have sucient funds to
purchase the annuity.

Exercise 5.8: [iid3] (Institute and Faculty of Actuaries CT1 Sep 2009) A life in-
surance company is issuing a single premium policy which will pay out $20,000 in
twenty years time. The interest rate the company will earn on the invested funds
over the rst ten years of the policy will be 4% per annum with a probability of
0.3 and 6% per annum with a probability of 0.7. Over the second ten years the
interest rate earned will be 5% per annum with probability 0.5 and 6% per annum
with probability 0.5.

(a) Calculate the premium that the company would charge if it calculates the
premium using the expected annual rate of interest in each ten year period.

(b) Calculate the expected prot to the company if the premium is calculated as
above. The rate of interest in the second ten year period is independent of
that in the rst ten year period.

(c) Explain why, despite the company using the expected rate of interest to cal-
culate the premium, there is a positive expected prot.

(d) By considering each possible outcome in (b):

1. Find the range of possible prots.

2. Calculate the standard deviation of the prot to the company.

35
Financial Mathematics  Exercises School of Risk and Actuarial Studies  UNSW

5.2 Lognormal Model

Exercise 5.9: [sto6] (Institute of Actuaries Examination September 2005) An in-


surance company has just written contracts that require it to make payments to
policyholders of $1,000,000 in ve years time. The total premiums paid by policy-
holders amounted to $850,000. The insurance company is to invest half the premium
income in xed interest securities that provide a return of 3% per annum eective.
The other half of the premium income is to be invested in assets that have an uncer-
tain return. The return from these assets in year t, it , has a mean value of 3.5% per
annum eective and a standard deviation of 3% per annum eective. The random
variables (1 + it ) (for t = 1, 2, . . .) are independent and lognormally distributed.

(a) Deriving all necessary formulae, calculate the mean and standard deviation of
the accumulation of the premiums over the ve-year period.

(b) A director of the company suggests that investing all the premiums in the
assets with an uncertain return would be preferable because the expected
accumulation of the premiums would be greater than the payments due to the
policyholders.

Explain why this still may be a more risky investment policy.

Exercise 5.10: [sto7] (Institute of Actuaries Examination April 2002) A company


is adopting a particular investment strategy such that the expected annual eective
rate of return from investments is 7% and the standard deviation of annual returns
is 9%. Annual returns are independent and (1 + it ) is lognormally distributed where
it is the return in the tth year. The company has received a premium of $1,000 and
will pay the policyholder $1,400 after 10 years.

(a) Calculate the expected value and standard deviation of an investment of $1,000
over 10 years, deriving all formulae that you use.

(b) Calculate the probability that the accumulation of the investment will be less
than 50% of its expected value in ten years time.

(c) The company has invested $1,200 to meet its liability in 10 years time. Cal-
culate the probability that it will have insucient funds to meet its liability.

Exercise 5.11: [sto8] (Institute of Actuaries Examination September 2007) The


expected eective annual rate of return from a bank's investment portfolio is 6%
and the standard deviation of annual eective returns is 8%. The annual eective
returns are independent and (1+it ) is lognormally distributed, where it is the return
in year t.
Deriving any necessary formulae:

(a) Calculate the expected value of an investment of $2 million after ten years.

36
Financial Mathematics  Exercises School of Risk and Actuarial Studies  UNSW

(b) Calculate the probability that the accumulation of the investment will be less
than 80% of the expected value.

Exercise 5.12: [sto9] (Institute of Actuaries Examination September 2000) An


investment bank models the expected performance of its assets over a ve-day period.
Over that period, the return on the bank's portfolio, i, has a mean value of 0.1%
and standard deviation of 0.2%. (1 + i) is lognormally distributed.

Calculate the value of j such that the probability that i is less than or equal to j is
0.05.

Exercise 5.13: [sto10] (Institute of Actuaries Examination September 2004) The


expected annual eective rate of return from an insurance company's investments is
6% and the standard deviation of annual returns is 8%. The annual eective returns
are independent and (1 + it ) is lognormally distributed, where it is the return in the
tth year.

(a) Calculate the expected value of an investment of $1 million after ten years.

(b) Calculate the probability that the accumulated of the investment will be less
than 90% of the expected value.

Exercise 5.14: [log1] (Institute and Faculty of Actuaries CT1 April 2010) The
annual returns, i, on a fund are independent and identically distributed. Each year,
2
the distribution of 1 + i is lognormal with parameters µ = 0.05 and σ = 0.004,
where i denotes the annual return on the fund.

(a) Calculate the expected accumulation in 25 years' time if $3,000 is invested in


the fund at the beginning of each of the next 25 years.

(b) Calculate the probability that the accumulation of a single investment of $1


will be greater than its expected value 20 years later.

Exercise 5.15: [log2] (Institute and Faculty of Actuaries CT1 April 2011) The an-
nual rates of return from a particular investment, Investment A, are independently
and identically distributed. Each year, the distribution of (1 + it ), where it is the
2
rate of interest earned in year t, is log-normal with parameters µ and σ . The mean
and standard deviation of it are 0.06 and 0.03 respectively.

An insurance company has liabilities of $15m to meet in one year's time. It currently
has assets of $14m. Assets can either be invested in Investment A, described above,
or in Investment B which has a guaranteed return of 4% per annum eective.

(a) Calculate µ and σ2.

(b) Calculate, to two decimal places, the probability that the insurance company
will be unable to meet its liabilities if:

37
Financial Mathematics  Exercises School of Risk and Actuarial Studies  UNSW

1. All assets are invested in Investment B.

2. 75% of assets are invested in Investment A and 25% of assets are invested
in Investment B.

(c) Calculate the variance of return from each of the portfolios from the above
scenarios.

Exercise 5.16: [log3] (Institute and Faculty of Actuaries CT1 April 2012) The
annual yields from a fund are independent and identically distributed. Each year,
2
the distribution of 1 + it is log-normal with parameters µ = 0.05 and σ = 0.004,
where i denotes the annual yield on the fund.

(a) Calculate the expected accumulation in 20 years' time of an annual investment


in the fund of $5,000 at the beginning of each of the next 20 years.

(b) Calculate the probability that the accumulation of a single investment of $1


made now will be greater than its expected value in 20 years' time.

Exercise 5.17: [log4] (Institute and Faculty of Actuaries CT1 April 2014) An in-
vestor is considering investing $18,000 for a period of 12 years. Let it be the eective
rate of interest in the tth year, t ≤ 12. Assume, for t ≤ 12, that it has mean value of
0.08 and standard deviation 0.05 and that 1 + it is independently and lognormally
distributed.
At the end of the 12 years the investor intends to use the accumulated amount of
the investment to purchase a 12-year annuity certain paying:

• $4,000 per annum monthly in advance during the rst four years;

• $5,000 per annum quarterly in advance during the second four years;

• $6,000 per annum continuously during the nal four years.

The eective rate of interest will be 7% per annum in years 13 to 18 and 9% per
annum in years 19 to 24 where the years are counted from the start of the initial
investment.

(a) Determine the distribution of S12 where St is the accumulation of $1 over t


years.

(b) Calculate the probability that the investor will meet the objective.

5.3 Dependence and Further Concepts

Exercise 5.18: [new3] Company A and Company B currently have a swap contract
where A pays (annually) a oating interest rate on a principal of $1 million to B
in exchange for a xed rate of 5% from B to A. The oating rate it in year t is
assumed to be independent for each year, and (1 + it ) is lognormally distributed
with it having mean 4% and standard deviation 2%.

38
Financial Mathematics  Exercises School of Risk and Actuarial Studies  UNSW

(a) Find the expected value of the accumulated value after 10 years of the net
receipts (from the perspective of A) from the swap.

(b) Using Excel, simulate (1000 times) the interest rate for the next 10 years
and verify your answer in (a). In addition, nd the variance and plot the
distribution of the accumulated value.

(c) In what situation may a company wish to enter into a xed-for-oating swap?

Exercise 5.19: [new5] Suppose that the interest rate yt (for year t) follows a mean
reverting process dened by:

yt = µ + β(yt−1 − µ) + σεt

where εt ∼ N (0, 1) iid for each time t = 0, 1, . . ..


It is also known that µ = 0.05, β = 0.4, and σ = 0.01. Denote the accumulated
value of $1 in one years time as S1 . Find:

(a) E(S1 )

(b) Var(S1 )

(c) Pr(S1 < 1.04)

if the interest rate last year was (i) 4% and (ii) 6%.

Exercise 5.20: [new6] Suppose that the interest rate yt (for year t) follows a mean
reverting process dened by:

yt = µ + β(yt−1 − µ) + σεt

It is also known that µ = 0.05, β = 0.4, and σ = 0.01. Denote the accumulated
value of $1 in ten years time as S10 . Using 1000 simulations in Excel, estimate:

(a) E(S10 )

(b) Var(S10 )

(c) Pr(S10 < 1.55)

(d) Plot the histogram of S10 .

if the interest rate last year was (i) 4% and (ii) 6%.

Exercise 5.21: [new9] Suppose that the interest rates in each year are independent
2
and identically distributed, with (1 + it ) ∼ LN (µ, σ ) and it having mean 4% and
standard deviation 2%. Denote as s̈30 the expected value and variance of the 30
year accumulated value of an annual payment of $1 in advance.

39
Financial Mathematics  Exercises School of Risk and Actuarial Studies  UNSW

(a) Derive recursive formulae which can be used to nd E(s̈30 ) and Var(s̈30 ).

(b) Using Excel and the formulae in (a), nd E(s̈30 ) and Var(s̈30 ).

(c) Estimate E(s̈30 ) and Var(s̈30 ) using simulation in Excel, and compare your
answers with those in (b).

40
Module 6

Solutions to Exercises

6.1 Module 1

Exercise 1.1 [int1]

Under compound interest:


Accumulated amount at t = 4: 100(1.05)4 = 121.5506
Accumulated amount at t = 5: 100(1.05)5 = 127.6282
Interest earned = 6.08

Under simple interest:


Accumulated amount at t = 4: 100(1 + 4 · 0.05) = 120
Accumulated amount at t = 5: 100(1 + 5 · 0.05) = 125
Interest earned = 5.00

Exercise 1.2 [int2]

275 1/5

200(1 + i)5 = 275 ⇒ i = 200
− 1 = 0.06576

Exercise 1.3 [int3]


ln(275/200)
200(1.05)t = 275 ⇒ t = ln 1.05
= 6.527 years

Exercise 1.4 [int4]


5 1
275v = 215.47, where v= 1+i

Exercise 1.5 [int5]


n
150(1 + i) = 240
∴ 1000(1 + i)n = 1000
150
150(1 + i)n = 1000
150
· 240 = 1600

Exercise 1.6 [int7]

The most one requires will be the amount X which is sucient even when the
interest rate is always at its minimum of 5% in the latter 10 years (as this minimises

41
Financial Mathematics  Exercises School of Risk and Actuarial Studies  UNSW

the interest earned). Therefore:


10 10
X(1.08) (1.05) = 1000000
∴ X = 284360

Exercise 1.7 [int8]

The accumulation of $1 under the eective rate and the 8% and 5% rates should be
equivalent:
(1 + i) = (1.08)10 (1.05)10
20

∴ i = 0.06489

Exercise 1.8 [int9]

t = 0:
Value at time
2 3
5 + 3v + v = 8.2307
Value at time t = 3:
3
8.2307(1 + i) = 10.9550
Alternatively: 5(1 + i)3 + 3(1 + i) + 2 = 10.9550

Exercise 1.9 [int10]

The 2nd set of cash ows contains two of the 1st set, one starting at t = 0 and
another starting att = 1. Thus, the value is given by:
7.7217 + v7.7217 = 15.0757

Exercise 1.10 [int11]

100 (1.10)3 − 20 (1.10)2 − 20 (1.10) = 86.90

Exercise 1.11 [int12]


6
100(1 + i) = 200
∴ i = 0.12246

Exercise 1.12 [int13]

(SOA Course 2 May 2000, Question 1)

Accumulated value of 1st account after 10 years:

10(1 + 0.11 · 10) + 30(1 + 0.11 · 5) = 67.50

Therefore, accumulated value of 2nd account after 10 years must also be 67.50:

10(1.0915)10−n + 30(1.0915)10−2n = 67.50


∴ 30(1.0915)10 v 2n + 10(1.0915)10 v n − 67.50 = 0

42
Financial Mathematics  Exercises School of Risk and Actuarial Studies  UNSW

p
n −10(1.0915)10 ± 100(1.0915)20 + 120(1.0915)10 · 67.50
∴v =
60(1.0915)10
= 0.8158 or − 1.1491
ln 0.8158
Since v n > 0, we have v n = 0.8158 ⇒ n = ln v
= 2.325

Exercise 1.13 [int15]

(SOA Course 2 Nov 2000, Question 2)

For the rst investment (the $100 at beginning of 1990), the interest credited in
1993 is 8% of the accumulated value at the beginning of 1993 (ie. after 3 years):

100(1.10)(1.10)(1 + 0.01x)(0.08) = 9.68 + 0.0968x

Similarly, for the second investment, the interest credited is:

100(1.12)(1.05)(0.10) = 11.76

and for the third investment:

100(1.08)(0.01(x − 2)) = 1.08x − 2.16

Therefore, the total interest credited in 1993 is:

19.28 + 1.1768x = 28.40

∴ x = 7.7498

Exercise 1.14 [int19]


1
Using v= 1+r
, we have the following present values for the two payment options:

P VA = 610 + 475v + 340v 2

P VB = 560 + 580v + 274v 2

Option A is preferred when P VB − P VA = −50 + 105v − 66v 2 > 0. Note that the
2
quadratic −50 + 105v − 66v = 0 has no real roots. The quadratic is negative at
v = 1, since −50 + 105 − 66 = −11 < 0. This implies that P VB < P VA for all values
of r , so Option B is always preferred to A.

Exercise 1.15 [int18]


(a) The equation of value for each of the alternative is given as follows with the
corresponding yield (rate of interest) r:
Alternative A:
1000(1 + r)3 = 1330
r = 9.9724%

43
Financial Mathematics  Exercises School of Risk and Actuarial Studies  UNSW

Alternative B:
1000(1 + r)5 = 1550
r = 9.1607%

Alternative C:
 2  3 !
1 1 1
1000(1 + r)5 = 425 1 + + +
1+r 1+r 1+r

r = 8.5761%
Note: for C, the yield must be found numerically (eg. using Solver in Excel - See
Spreadsheet int18.xlsx)

(b) We need the accumulated value of $1330 at a rate r to be at least $1550:

1330(1 + r)2 ≥ 1550


∴ 1330r2 + 2660r − 220 ≥ 0
For equality, we have:

−2660 ± 26602 + 4 · 1330 · 220
r= = 0.079543 or − 2.0795
2660
∴ r ≥ 7.9543%
We ask this question to compare A and B. If we chose A, then after 3 years, we
would need to invest the $1330 at some interest rate (eg. by putting the money in
the bank) for 2 years, after which it can be compared with the $1550 from B (at 5
years). Thus, we must be able to earn at least 7.95% during this 2 year period for
A to be a better choice than B.

(c) The value of alternative C is:


 2  3 !
1 1 1
425 1 + + + = 1508.97
1+r 1+r 1+r

which is less than the value of B ($1550). Note that we have used the interest rate
from alternative C.

Exercise 1.16 [int20]

Equation of value:
P (v + v 2 + v 3 + v 4 + v 5 ) = 1000000
1
where v= 1.13
. Therefore, we obtain P = 284314.54.

Exercise 1.17 [int6]

d = 1 − v = 0.06
1
v = 1+i ⇒ i = v1 − 1 = 0.06383
Also: i = d/v = 0.06383

44
Financial Mathematics  Exercises School of Risk and Actuarial Studies  UNSW

Exercise 1.18 [int14]

(SOA Course 2 May 2001, Question 12)

Note the relationship between accumulating using eective interest and discount
rates:
1 + i = (1 − d)−1
Thus:
X = 100[(1 − d)−11 − (1 − d)−10 ] = 50[(1 − d)−17 − (1 − d)−16 ]
For clarity, denote R = (1 − d)−1 .

100R10 (R − 1) = 50R16 (R − 1)
2R10 = R16
R6 = 2
R = 21/6

∴ X = 100(R11 − R10 ) = 38.8793

Exercise 1.19 [int24]

An equation of value to solve for the eective semi-annual interest rate i is

4843.30(1 + i)4 = 6000

whose only positive solution is i = 0.055. The total value of the three payments at
the time the note matures is

1000(1.055)3 + 1000(1.055)2 + 2000(1.055) = 4397.27

so she will need a top up of $1602.73 to redeem the note for $6000.

Exercise 1.20 [int25]

Each deposit accumulates under simple interest for 1 month, then under compound
interest for the remaining quarters. Therefore, by the end of December 2005:

March deposit accumulates to:

  19
1 0.06
1000 1 + 0.06 1+ = 1333.59
12 4

June deposit accumulates to:

  18
1 0.06
1000 1 + 0.06 1+ = 1313.88
12 4

September deposit accumulates to

  17
1 0.06
1000 1 + 0.06 1+ = 1294.46
12 4

45
Financial Mathematics  Exercises School of Risk and Actuarial Studies  UNSW

December deposit accumulates to

  16
1 0.06
1000 1 + 0.06 1+ = 1275.33
12 4

Therefore, the total interest would be:

1333.59 + 1313.88 + 1294.46 + 1275.33 − 4000 = 1217.25

Exercise 1.21 [int21]

Using the relations for equivalent rates

m −m
i(m) d(m)
 
−1
1+i= 1+ = (1 − d) = 1 − = eδ
m m

we have:

Given rate Equivalent eective rate i A(4.5) = 10000(1 + i)4.5


d = 0.05 0.0526316 12596.32
i(2) = 0.05 0.0506250 12488.63
i(12) = 0.05 0.0511619 12517.38
d(2) = 0.05 0.0519395 12559.10
d(12) = 0.05 0.0513809 12529.11
δ = 0.05 0.0512711 12523.23

Exercise 1.22 [int22]

Note that the accumulation function is given by:

Z t  Z t 
0.04
a(t) = exp δ(s)ds = exp ds = (1 + t)0.04
0 0 1+s

(a)
a(2) − a(1) (3)0.04 − (2)0.04
= = 0.01635084
a(1) (2)0.04

(b)
a(3) − a(2) (4)0.04 − (3)0.04
= = 0.01157375
a(2) (3)0.04

(c) Since A(t) = A(0)(1 + t)0.04 , then A(0) = 200000


(5)0.04
= 187530.19. Therefore:

200000 0.04
A(2) = (3) = 195954.86
(5)0.04

46
Financial Mathematics  Exercises School of Risk and Actuarial Studies  UNSW

Exercise 1.23 [int23]

We have (
1 + 0.1t t≤k
a(t) =
(1 + 0.1k) exp [0.08(t − k)] t>k
Hence (
(1 + 0.1k) exp [0.08(4 − k)] k<4
a(4) =
1.4 k≥4

(a) To maximize a(4), dierentiate w.r.t. k (assuming a(4) = (1 + 0.1k)e0.08(4−k) ,


ie. k < 4; will need to check this)

d
a(4) = 0.1 exp [0.08(4 − k)] − 0.08 (1 + 0.1k) exp [0.08(4 − k)]
dk
and set to be zero. Solving for k (and noting that the exponential is positive) we
have:
0.1 = 0.08 (1 + 0.1k)
∴ k = 2.5
Check: at k = 2.5, a(4) > 1.4, so assumption of k < 4 is OK. Note that if the as-
sumption of k < 4 was violated, then a(4) = 1.4 as opposed to our earlier expression
0.08(4−k)
of (1 + 0.1k)e , and a(4) will have a maximum of 1.4 (at any k ≥ 4).

(b)
(
1 + 0.1t t ≤ 2.5
a(t) =
1.25e0.08(t−2.5) t > 2.5
(
ln(1 + 0.1t) t ≤ 2.5
ln a(t) =
ln 1.25 + 0.08(t − 2.5) t > 2.5
(
0.1
d t ≤ 2.5
∴ δ(t) = ln a(t) = 1+0.1t
dt 0.08 t > 2.5

Exercise 1.24 [new10]

We use the result:

m −m
i(m) d(m)
 
−1
1+i= 1+ = (1 − d) = 1 − = eδ
m m
Therefore:

i(m) = m eδ/m − 1


d(m) = −m e−δ/m − 1


See spreadsheet new10.xlsx for plotting i(m) and d(m) . The plot should be similar to
the one below.

47
Financial Mathematics  Exercises School of Risk and Actuarial Studies  UNSW

Nominal Interest/Discount Rates

i(m)

0.054
d(m)

0.052
Nominal Rate

0.050
0.048
0.046

0 10 20 30 40 50

Compounding Frequency

Note that δ = 0.05 and all the points on this plot yield the same eective rate of
interest!

Exercise 1.25 [ann1]


1
(a) Each monthly payment is of the nominal amount. The eective monthly
12
rate i is given by:
0.12
i= = 0.01
12
Therefore:
 72  120
1000 1 400 1
PV = a + a + 2000
12 72 0.01 1.01 12 48 0.01 1.01
1 72  72 1 48  120
1000 1 − ( 1.01 400 1 − ( 1.01
   
) 1 ) 1
= + + 2000
12 0.01 1.01 12 0.01 1.01
= 4262.50 + 618.34 + 605.99
= 5486.80

Alternatively, we can obtain the annual eective rate j:

j = 1.0112 − 1 = 0.1268

48
Financial Mathematics  Exercises School of Risk and Actuarial Studies  UNSW

Then, using annuities payable monthly:


 6  10
(12) 1 (12) 1
PV = 1000a6 + 400a4 + 2000
1+j 1+j
6 10
1 − vj6 1 − vj4
     
1 1
= 1000 + 400 + 2000
0.12 1+j 0.12 1+j
= 5486.80

(b) Assume level annuity payments payable monthly, then


1 120 
1 − ( 1.01

)
X = 5486.80
0.01
∴ X = 78.720
The quoted annual payment is then 78.720 × 12 = 944.64.

Exercise 1.26 [ann2]

The equation of value is based on the present value of the amount received by each
charity (which are known to be equal):

P
a = v 20 P a∞
3 20
P 1 − v 20
 
P
∴ = v 20
3 i i
20 20
1 − v = 3v
v 20 = 0.25
∴ i = (0.25)−1/20 − 1 = 0.07177

Exercise 1.27 [ann3]

The accumulated value of the deposits 10 years after the rst deposit is:

AV = C s̈10 0.06
1
10 !
1−
= C(1.06)10 1.06
× 1.06
0.06
= 13.9716C
The present value of the loan payments as at the end of 10 years is:

P V = 15000ä5 0.06
1
5 !
1 − 1.06
= 15000 × 1.06
0.06
= 66976.5842
where we have used a rate of 6% because that is the amount paid by the account
(so the remaining funds in the bank will be earning 6% between years 10 and 15).

Equating these values and solving for C, we obtain C = 4793.75.

49
Financial Mathematics  Exercises School of Risk and Actuarial Studies  UNSW

Exercises 1.281.34
[annB1annB7] See the Mathematics of Investment and Credit solutions manual.

Exercise 1.35 [ann4]

The original and nal monthly eective interest rates i and j are given by:

(1 + i)12 = 1.03 ⇒ i = 0.00246627


(1 + j)12 = 1.05 ⇒ j = 0.00407412
The original monthly payment P is given by:

100000
P a300 i = 100000 ⇒ P = = 472.1087
a300 i
After 10 years, the remaining value of the annuity is:

P a180 i
This will also be the present value of the new annuity (with annual payments of P 0 ),
which is valued at the new interest rate j:
P a180 i = P 0 a180 j
Therefore, the new payment is:
a180 i
P0 = P = 538.1869
a180 j
Thus, the payment increase is:

P 0 − P = 66.08

Exercise 1.36 [ann5]

The 1-year eective rate i and 2-year eective rate j are given by:

i = 1.0252 − 1 = 0.050625
j = 1.0254 − 1 = 0.103813
By drawing a cash ow diagram, it can be seen that the cash ow stream is a 20-year
annuity with annual payments of $100, plus an additional $100 every 2nd year (ie.
an additional 20-year annuity with biannual payments of $100). Thus:

P V = 100a20 i + 100a10 j = 1844.16

Exercise 1.37 [ann6]

A discount rate of 10% p.a. is equivalent to an interest rate of i = 11.11% p.a.


Therefore, the current cash ows are worth:

18000ä12 i = 18000 (1 + a11 i ) = 129162.10


We want annual payments of X where:

129162.10 = Xa20 i
∴ X = 16337.69

50
Financial Mathematics  Exercises School of Risk and Actuarial Studies  UNSW

Exercise 1.38 [ann7]

See solutions manual. For 5 years, monthly payment required is 296.94.

Exercises 1.391.40
[annB8annB9] See the solutions in the Broverman text and solutions manuals.

Exercise 1.41 [ann13]

s10 is the accumulated value (at time 10) of $1 paid at each time t = 1, . . . , 10. The
accumulated value (at time 10) of a single $1 paid at time n is given by:
Z 10 
1
exp dt = exp [− ln(20 − 10) + ln(20 − n)]
n 20 − t
  
20 − n
= exp ln
10
20 − n
=
10
Therefore:
10
X 20 − n 19 + 18 + . . . + 10 1 10(10 + 1)
s10 = = = 20 − = 14.5
n=1
10 10 10 2

Exercise 1.42 [ann8]

The loan repayments are an increasing annuity with payments of P, 2P, . . . , 30P at
times t = 1, 2, . . . , 30. To determine the value of P, we use an equation of value (at
time t = 0):
ä30 0.04 − 30v 30
 
4000 = P (Ia)30 0.04 =
0.04
∴ P = 18.32
Immediately after the ninth payment, the outstanding loan is found by decomposing
the remaining payments into a level annuity and an increasing annuity:

10P v + 11P v 2 + 12P v 3 + ... + 30P v 21 = 9P a21 0.04 + P (Ia)21 0.04


= 4774.80
This is greater than the original loan amount, and is because the earlier payments
are (very) small relative to the latter payments. Therefore, the earlier payments are
insucient to pay o the interest, let alone pay o part of the principal. Hence, this
is also unlikely to exist in reality.

Exercise 1.43 [ann9]

The borrowings are a decreasing annuity, whereas the repayments can be decom-
posed into a level annuity of $300 and an increasing annuity starting at $200. An
equation of value (at the end of year when nal loan amount is received) is:

X(Ds)5 0.05 (1 + i) = 300a15 0.05 + 200(Ia)15 0.05

51
Financial Mathematics  Exercises School of Risk and Actuarial Studies  UNSW

Noting that:

a15 0.05 = 10.379658


ä − 15v 15
(Ia)15 0.05 = 15 0.05 = 73.667689
0.05
(Ds)5 0.05 = (1 + i)5 (Da)5 0.05
= (1 + i)5 [6a5 0.05 − (Ia)5 0.05 ]
ä5 0.05 − 5v 5
 
5
= (1 + i) 6a5 0.05 −
0.05
= 17.115531

We obtain:
300a15 0.05 + 200(Ia)15 0.05
X= = 993.11
(Ds)5 0.05 (1 + i)

Exercise 1.44 [ann12]

The annual eective rate i is given by:

1 + i = 1.0254 ⇒ i = 0.103813

For convenience, we will determine the initial deposit by discounting all cash ows
to time t=0 (1/1/2004).

The entire series of deposits can be decomposed into two series:

A. Deposits of X, 1.1025X, . . . , (1.1025)10 X at times t = 0, 1, . . . , 10

B. Deposits of 1.1025X, (1.1025)2 X, . . . , (1.1025)11 X at times t = 21 , 1 12 , . . . , 10 21

where time t is measured in years.

These two series are (geometrically) increasing annuities, and can be present valued
as geometric progressions:

P VA = X + 1.1025Xv + . . . + (1.1025)10 Xv 10
1 − (1.1025v)11
 
=X
1 − 1.1025v
= 10.934815X
P VB = v 1/2 (1.1025) X + 1.1025Xv + . . . + (1.1025)10 Xv 10
 

= v 1/2 (1.1025) P VA
= 11.474726X

Therefore, the initial deposit X is found as follows:

P VA + P VB = 110000v 11

∴ X = 1656.19

52
Financial Mathematics  Exercises School of Risk and Actuarial Studies  UNSW

Exercise 1.45 [new8]

The present value is $67989.65.

Refer to Spreadsheet new8.xlsx.

Exercise 1.46 [ann14]

There are a number of ways to decompose the payments into annuity streams, which
allow the present value to be determined more easily.

A simple method is to consider 3 cash ow streams, each containing payments 1


year apart (which also means we will use the annual eective rate of 0.10):

A. 10, 20, 30, 40 at times t = 31 , 1 13 , 2 13 , 3 13

B. 20, 30, 40, 50 at times t = 23 , 1 23 , 2 23 , 3 23

C. 30, 40, 50, 60 at times t = 1, 2, 3, 4

Each of these streams is the combination of an increasing annuity and a level annuity.
Therefore:

P VA = 10v 1/3 (Iä)4


P VB = 10v 2/3 (Iä)4 + 10v 2/3 ä4
P VC = 10v(Iä)4 + 20vä4

The annuity factors are given by:

(Iä)4 = 1 + 2v + 3v 2 + 4v 3
= 8.302780
1 − v4
 
ä4 = (1.1)
0.1
= 3.486852

Therefore, we obtain the present value:

P VA + P VB + P VC = 329.95

A more elegant method involves decomposing the original payments into:

X. 10,20,30,. . . ,10,20,30 at all times (t = 13 , 23 , 1, . . . , 4)

Y. 0,0,0,10,10,10,. . . ,30,30,30 at all times (t = 13 , 23 , 1, . . . , 4)

The payments of X can be grouped by year, resulting in four payments of (10,20,30).


Thus, X is a 4-year level annuity-due, with each payment being an increasing annu-
ity:
P VX = 10(Ia)3 j ä4 i = 194.3179

53
Financial Mathematics  Exercises School of Risk and Actuarial Studies  UNSW

1
where i = 0.1 is the annual eective rate, and j = (1.1)1/3 − 1 is the
3
-year eective
rate.

Conversely, the payments of Y can are a 3-year increasing annuity, with each pay-
ment being a level annuity (omitting the rst three payments of 0):

P VY = 10a3 j (Ia)3 i = 135.6289

Summing these up, we obtain the same present value:

P VX + P VY = 329.95

Exercise 1.47 [ann15]

The present value (at time t = 0) of the rst ten payments is:

P (Ia)10 0.07

The present value (at time t = 10) of the last ten payments is:

10(10P ) = 100P

since each payment is discounted at the same rate as the payment growth rate (each
n n n
payment is of amount 10(1.05) P which is worth 10(1.05) P v at time t = 10, and
v n = (1.05)−n cancels with the factor (1.05)n ).
Therefore, the present value of all payments at time t=0 is:

P (Ia)10 0.07 + 100P (1.07)−10 = 50000

Since (Ia)10 0.07 = 34.739133, we get:

P = 584.29

Exercise 1.48 [new4]

(a) Let the annual payment be X. Therefore:

Xa20 0.05 = 10000

∴ X = 802.4259

(b) Let the rst annual payment be X. Therefore:

(1.03)Xv + (1.03)2 Xv 2 + . . . + (1.03)20 Xv 20 = 10000

1 − (1.03v)20
 
∴ 1.03Xv = 10000
1 − 1.03v
∴ X = 608.1346

54
Financial Mathematics  Exercises School of Risk and Actuarial Studies  UNSW

(c) To be fair, we need the present value of both payment streams to be equal (to
10000).

In nominal terms:

20
X 20
X
608.13(1.03)k v0.05
k
= k
802.43v0.05 = 10000
k=1 k=1

or in real terms:

20
X 20
X
608.13vrk = 802.43(1.03)−k vrk = 10000
k=1 k=1

1.05
where r= 1.03
− 1.

Exercise 1.49 [new1]

(a) The annual eective interest rate is i = eδ − 1 = 0.05127.


We note that the payment at time n is 1+2+. . .+n. Therefore, the perpetuity
can be decomposed into a level perpetuity of 1 (1st payment at time 1), plus a
level perpetuity of 2 (1st payment at time 2), etc. This can be interpreted as
an increasing perpetuity, with each regular payment being a level perpetuity
itself:

P V = vä∞ + 2v 2 ä∞ + 3v 3 ä∞ + . . .


= ä∞ v + 2v 2 + 3v 3 + . . .


= ä∞ (Ia)∞

Therefore:
  
1 ä∞
PV =
d i
1
= 2
di
= 8200

where d = iv = 0.04877.

(b) See spreadsheet new1.xlsx. The plot should be similar to the one below.

55
Financial Mathematics  Exercises School of Risk and Actuarial Studies  UNSW

120



●●




● ●

● ●

● ●
● ●
● ● ●
● ●

● ●
● ●

100
● ●
● ●

● ●

● ●

● ●

● ●

● ●

80
● ●


● ●

● ●
PV of payment



● ●


● ●
60



● ●


● ●


● ●




● ●●
40



● ●●



● ●


●●
● ●

●●

● ●
●●


20

● ●●


●●


● ●●


●●



● ●


●●



● ●


●●


●●


● ●●


●●


●●


●●


● ●●


●●


●●



● ●


●●


●●


●●


●●


●●


●●


●●


●●


●●


●●


●●


●●


●●


●●


●●


●●


●●


●●


●●


●●


●●


●●


●●


●●


●●


●●


●●


●●


●●


●●


●●


●●


●●


●●


●●


●●


●●


●●


●●


●●


●●


●●


●●


●●


●●


●●


●●


●●


●●


●●


●●


●●


●●


●●


●●


●●


●●


●●


●●


●●


●●


●●


●●


●●


●●


●●


●●


●●


●●


●●


●●


●●


●●


●●


●●


●●


●●


●●



0

0 100 200 300 400 500

Time

Aside: The graph is actually related to the Gamma function. The present value of
the nth payment is:

1 2 n 1 n 1 2 −δn 1 −δn
n v + nv = n e + ne
2 2 2 2
Thus, we require:

X ∞
X
2 −δn
ne and ne−δn
n=1 n=1

These can be approximated using the Gamma function by noting that:

Z ∞ ∞
X
Γ(k) = (k − 1)! = xk−1 e−x dx ≈ xk−1 e−x
0 x=1

Therefore, the present value is:

∞ ∞
!
1 X X
PV = n2 e−δn + ne−δn
2
Zn=1∞ Z ∞  n=1
1 2 −δx −δx
≈ x e dx + xe dx
2 0 0
1 1 ∞ 2 −u 1 ∞ −u
 Z Z 
= u e du + 2 ue du where u = δx
2 δ3 0 δ 0
 
1 2! 1!
= +
2 δ3 δ2

56
Financial Mathematics  Exercises School of Risk and Actuarial Studies  UNSW

The force of interest is δ = 0.05, which results in:

P V ≈ 8200

Exercises 1.501.57
[annB10annB15] See the solutions in the Broverman text and solutions manuals.

Exercise 1.58 [new2]


(12)
We require 100(Ia)10 , 100(Ia)10 , and 100(Iā)10 . The monthly eective rate is:

j = (1 + i)1/12 − 1 = 0.004074

and therefore the nominal rate (payable monthly) is:

i(12) = 12j = 0.04889

The force of interest is:


δ = ln(1 + i) = 0.04879
Therefore:

ä10 − 10v 10
100(Ia)10 =
i
= 3937.38
(12) ä − 10v 10
100(Ia)10 = 10 (12)
i
= 4026.81
ä − 10v 10
100(Iā)10 = 10
δ
= 4035.01

Exercise 1.59 [ann11]

The present value of the annuity is given by:

Z 14
PV = (t2 − 1)v(t)dt
1

where the discount factor v(t) varies (continuously) with time t:


 Z t 
1 1 1
v(t) = = exp − ds = exp (− ln(1 + t)) =
a(t) 0 1+s 1+t

Therefore, we have:

14 2 14
t −1
Z Z
PV = dt = (t − 1)dt = 84.50
1 t+1 1

57
Financial Mathematics  Exercises School of Risk and Actuarial Studies  UNSW

6.2 Module 2

Exercise 2.1 [loa1]

(a) The repayments involve annual payments (in arrears) of X, ie. an annuity-
immediate:

Xa5 = 15000
X = 3464.62

(b) The retrospective method considers the accumulated value of past cash ows:

OB2 = 15000(1.05)2 − Xs2


= 9435.02

(c) The prospective method considers the present value of future cash ows:

OB2 = Xa3
= 9435.02

(d) Let the new annual payment be Y. The new future repayments must be able
to repay the outstanding balance calculated in (b) and (c). Therefore:

Y a3 0.075 = OB2 = 9435.02


Y = 3628.12

(e) Let the renegotiated payment be Z. We have:

Za4 0.076 = OB2 = 9435.02


Z = 2823.31

(f ) See the tutorial solution spreadsheet (loa1.xls).

Exercise 2.2 [loa2]

(a) The annual payment consists of two components: the interest repayment, and
the payment into the sinking fund. The interest payment is simply 7% of the
loan:
20000(0.07) = 1400
The sinking fund payment is an amount X such that the sinking fund will
accumulate to the $20000 principal after 6 years:

Xs6 0.05 = 20000

∴ X = 2940.35
Therefore, the annual repayment is:

1400 + 2940.35 = 4340.35

58
Financial Mathematics  Exercises School of Risk and Actuarial Studies  UNSW

(b) Similarly to (a), we have:


Xs6 0.07 = 20000
∴ X = 2795.92
Therefore, the annual repayment is 4195.92.

(c) The standard loan arrangement would require annual payments of $4195.92 as
the two methods are equivalent when the sinking fund and loan interest rates
are the same. Hence, we would prefer the standard loan arrangement, as it
involves lower annual repayments. In general, the standard loan is preferable
if the sinking fund rate is below the loan interest rate (which is usually true 
loan interest rate higher than savings interest rate).

(d) See the tutorial solution spreadsheet (loa2.xls).

Exercise 2.3 [loa3]

(a) Total interest to be repaid:

I = Lf n = 5000 · 0.10 · 2 = 1000


Monthly loan repayments are:

L+I 6000
R= = = 250
N 24
The monthly eective rate j is the solution to the following equation of value:

Ra24 j = 5000
Using Newton-Raphson, we dene the function f as follows:

1 − (1 + j)−24
 
f (j) = Ra24 j − 5000 = 250 − 5000
j
The derivative is:

24j(1 + j)−25 − (1 − (1 + j)−24 )


 
0
f (j) = 250
j2
Starting with j0 = 0.10:
f (j0 )
j1 = j0 − = −0.062716
f 0 (j0 )
f (j1 )
j2 = j1 − = −0.022528
f 0 (j1 )
f (j2 )
j3 = j2 − = 0.004878
f 0 (j2 )
f (j3 )
j4 = j3 − = 0.014291
f 0 (j3 )
f (j4 )
j5 = j4 − = 0.015125
f 0 (j4 )

59
Financial Mathematics  Exercises School of Risk and Actuarial Studies  UNSW

This results in an annual eective rate i of:

i = (1 + j5 )12 − 1 = 19.74%

Using spreadsheet loa3.xlsx for 10 iterations of Newton Ralphson. The solution


is j = 0.015131. Therefore, the eective annual rate i is:

i = (1 + j)12 − 1 = 19.75%

(b) The outstanding balance (using the prospective method) is:

250a12 j = 2724.66

Exercises 2.42.6
[loaB3loaB5] See the Mathematics of Investment and Credit solutions manual.

Exercise 2.7 [loa5]


0.18
The eective monthly rate is
12
= 0.015. Therefore, the monthly repayment X is
given by:
Xa100 0.015 = 20000 ⇒ X = 387.41
As we are only required to provide the last four instalments (ie. 97th100th pay-
ments), we can begin the loan schedule at time t = 96 (months). The outstanding
balance at time t = 96 is:

OB96 = Xa4 0.015 = 1493.23

The loan schedule for times t = 97, . . . , 100 can then be found using the following
recursive equations:

It = OBt−1 · 0.015
P Rt = X − It
OBt = OBt−1 − P Rt

Therefore, we obtain:

Time Interest Principal Repaid Outstanding Balance


(t) (It ) (P Rt ) (OBt )
96 ? ? 1493.23
97 22.40 365.01 1128.22
98 16.92 370.49 757.73
99 11.37 376.05 381.69
100 5.73 381.69 0

The full loan schedule is shown in the tutorial spreadsheet provided (loa5.xls).

60
Financial Mathematics  Exercises School of Risk and Actuarial Studies  UNSW

Exercise 2.8 [loa6]

The present value of the original four loans is:

P V = 4.36a11 0.01 + 17.2a15 0.01 + 35a12 0.01 + 20.24a18 0.01


= 1009.51
The total (sum) payments is:

11 (4.36) + 15 (17.2) + 35 (12) + 18 (20.24) = 1090.28


For the new consolidated loan, denote the term to run (in months) as n. Since
n must be an integer, the consolidated loan cannot have level repayments with a
further restriction that these repayments must sum to the original total (1090.28).
Therefore, the loan will be assumed to involve monthly repayments of X, with an
additional nal payment of Y to account for any remaining outstanding balance.

We have two equations to describe the two constraints (repayments must sum to
original total; eective rate must remain the same):

nX + Y = 1090.28
Xan 0.01 + Y v n+1 = 1009.51
Here, we have three variables and two equations. Therefore, we must x one variable,
after which we can determine the other two as a function of the rst variable. A
logical choice would be to set n to dierent values (as it has to be an integer) and
get X and Y by solving the simultaneous equations (noting that the coecients
n, v n+1 , an 0.01 are all known once n is known). Here are the results for various
choices of n:

n X Y
11 69.25 328.56
12 72.45 220.86
13 74.02 128.05
14 74.51 47.13
15 74.29 -24.11
16 73.60 -87.40
17 72.61 -144.05
18 71.41 -195.13

Intuitively, the nal payment should not be negative and should be as small as
possible as it represents an extra `top-up' payment to ensure the loan is fully repaid
to the nearest cent. The purpose of the nal payment is not to repay a signicant
portion of the loan. Hence, the solution where n = 14 is the most appropriate,
and so we choose n = 14 with X = 74.51 and Y = 47.13. In other words, the
consolidated annuity is of $74.51 for 14 months, with a nal payment of $47.13 at
the end of 15th month.

Note: the necessity of the nal payment Y can be veried by observing (from the
table) that there is no solution where n is an integer and Y = 0.
The table can be found in spreadsheet loa6.xlsx.

61
Financial Mathematics  Exercises School of Risk and Actuarial Studies  UNSW

Exercise 2.9 [loa7]

The initial payment X can be found as follows, noting the monthly eective rate is
0.12
12
= 0.01:

Xa12 0.01 + (1.1)Xv 12 a12 0.01 + (1.1)2 Xv 24 a12 0.01 = 47500

∴ X = 1440.80
The loan schedule is shown in the tutorial spreadsheet provided (loa7.xls).

Exercise 2.10 [loa8]

(a) Let X be the annual payment. We have:

Xa10 0.0675 = 600000 ⇒ X = 84441.97

Hence total payment is 10X = 844419.69.

(b) Let Y be the monthly payment. The monthly eective rate i is given by:

1 + i = (1.0675)1/12 ⇒ i = 0.005458

Therefore:
Y a120 i = 600000 ⇒ Y = 6828.08
Hence the total payment is 120Y = 819369.39, which is 25050.30 less than the
total payment in (a).

(c) The amount to settle the loan is the outstanding balance at time t = 5. This
will not change when the interest rate changes, as the outstanding balance
is determined from the past (ie. using the recursive relationships of a loan
schedule). Therefore, the amount will be OB5 under the old interest rate of
6.75%:
OB5 = Xa5 0.0675 = 348558.74
Note that this implies the prospective method (based on the future) becomes
incorrect when interest rates change, while the retrospective method (based
on the past) remains correct. If we used the prospective method, we would
have calculated an outstanding balance of:

Xa5 0.0725 = 343923.44

This is the present value of future repayments (as payments now have to be
discounted at the new rate), but it is not the outstanding balance.

Exercise 2.11 [loa4]

Note that we cannot nd a solution analytically due to the complexity of the loan
(dierent period for interest and principal (re)payments; non-equal actual pay-
ments). Therefore, we will model the loan using a spreadsheet (loa4.xls).

62
Financial Mathematics  Exercises School of Risk and Actuarial Studies  UNSW

The time step will be one quarter, ie. t = 0, 41 , . . . , 19. The interest (It ), outstanding
balance (OBt ), and nominal/actual payments (N P Rt , AP Rt ) are described using a
set of recursive relationships:

It = OBt− 1 · 0.02
4

OBt = OBt− 1 − N P Rt
4
75000
N P Rt = = 5000 (t = 5, 6, . . . , 19)
15
AP Rt = 5000 (t = 5, 6, . . . , 19)
0.08
where we note that the quarterly eective interest rate is
4
= 0.02, and the
nominal and actual payments are equal since the loan is repaid at par.

The net cash ows received by the purchaser are the payments of interest (It ) and
principal (AP Rt ). The price for a required eective yield i is found by discounting
these cash ows at the rate i. Therefore:

X  1 t
P = (It + AP Rt )
t
1+i

where the summation is taken over t = 0, 14 , . . . , 19.


For i = 0.10, theprice is P = 66636.60.
2
For i = 1 + 0.10
2
− 1 = 0.1025, the price is P = 65565.63.

Exercise 2.12 [loa9]

This question requires a spreadsheet model due to the complexity of the capital
gains tax. A spreadsheet model can be setup using the standard recursive formulae,
along with the following formulae for tax purposes:

ITt = τI · It
 +
+ N P Rt
CGTt = (τCG · CGt ) = τCG AP Rt − P
500000

The capital gain is determined as the actual repayment less the purchase price. The
N P Rt
portion of the loan repaid by AP Rt is of the entire loan, and this portion of
500000
N P Rt
the loan was purchased at a price of
500000
P.
The price is the present value of the net cash ows at the eective yield of 6%:

X X
P = v t CFt = v t (AP Rt + It − ITt − CGTt )
t t

The spreadsheet model can be found in loa9.xls. The solution is (a) $439,608.30 and
(b) $538,334.94.

63
Financial Mathematics  Exercises School of Risk and Actuarial Studies  UNSW

Note that each component of the price can be expressed as follows (where i = 0.06):
X
v t AP Rt = 25000(1.05)v 10 ä10
t
X X
t
v (It − ITt ) = (1 − τI ) (0.08) v t OBt− 1
4
t t
 +
X
t 25000
v CGTt = 0.3 25000(1.05) − P v 10 ä10
t
500000

where:
X (4) (4) (4)
v t OBt− 1 = 500000a10 + 475000v 10 a1 + . . . + 25000v 28 a1
4
t
(4) (4) (4)
= 500000a10 + 25000v 10 a1 (Da)19

Exercise 2.13 [new13]

(a) The equation of value is:

80 = 6a10 + 100v 10

(b) Newton-Raphson nds the root of f (x) = 0 recursively as follows:

f (xn−1 )
xn = xn−1 −
f 0 (xn−1 )

where xn is the root obtained from the nth iteration.

The equation we need to solve is:

1 − v 10
 
0=6 + 100v 10 − 80
i
6 6(1 + i)−10
= − + 100(1 + i)−10 − 80
i i
Thus, dene:

6 6(1 + x)−10
f (x) = − + 100(1 + x)−10 − 80
x x
See Spreadsheet new13.xlsx for the iterations in Excel. The solution is i =
9.1349%.

Exercise 2.14 [loa10]

You can (and should) do this question both by hand and by spreadsheet.

(a) The capital gain is already known as the price is given and the bond only has
one capital repayment at the end.

64
Financial Mathematics  Exercises School of Risk and Actuarial Studies  UNSW

For the 15% bond on its own, the capital gain is:

100 − 105.80 < 0

Therefore there will be no CGT. The equation of value is then:

105.80 = 15 (1 − 0.35) a4 j + 100 (1 + j)−4

where j is the annual eective yield. The solution j = 8.00% can easily be
veried by substitution.

For the 8% bond, the capital gain is:

100 − 85.34 = 14.66 > 0

Therefore there will be CGT. The equation of value is then:

85.34 = 8(1 − 0.35)a4 j + 100 − 14.66(0.5) (1 + j)−4




The solution j = 8.00% can easily be veried by substitution.

Therefore both bonds (on their own) give an eective yield of 8% p.a.

The solution by spreadsheet is provided in loa10.xls. The equations to be used


are:

N P Rt= 100 (t = 4)
AP Rt = N P Rt
It= i · OBt−1
OBt = OBt−1 − N P Rt
ITt= (0.35)It
 
N P Rt
CGTt = (0.50) AP Rt − P
100
X
P = v t (AP Rt + It − ITt − CGTt )
t

where i is the interest rate (15% or 8%) and j is the annual eective yield,
and v = (1 + j)−1 .
We require the yield such that P is (a) 105.80 or (b) 85.34. These both result
in a solution of j = 8.00%.

(b) Instead of putting $100 in one single bond, we now put $x into the 15% bond
and $ (100 − x) into the 8% bond. This means that we buy:

x
of the 15% bond with face value 100
105.8
and
100 − x
of the 8% bond with face value 100
85.34

65
Financial Mathematics  Exercises School of Risk and Actuarial Studies  UNSW

We want to nd the value x such that the net capital gain is 0. Note that
this would minimise our net CGT liability, as we have minimised the capital
gain and we do not get a refund from capital losses (therefore no advantage in
having a negative CG).

x 100 − x
CG = (100 − 105.8) + (100 − 85.34) = 0
105.8 85.34
100(14.66)
85.34
∴x= 5.8 = 75.81
105.8
+ 14.66
85.34
The equation of value is then:

x  100 − x
15(1 − 0.35)a4 j + 100vj−4 + 8(1 − 0.35)a4 j + 100vj−4

100 =
105.8 85.34
The solution j = 8.46% can be veried by substitution.

For the spreadsheet model, we note that the combined portfolio can be con-
sidered as a single bond (since interest payments are still level, and face value
is still $100). The price is $100, since we have zero capital gain (ie. purchasing
a par bond). The interest rate becomes:

i = 0.15α + 0.08(1 − α)
x 100−x
where α= 105.8
is the proportion of the rst bond purchased, and 1−α = 85.34
is the proportion of the second bond purchased.

The rest of the spreadsheet remains the same. The yield can be solved numer-
ically to obtain j = 8.46%.

Exercises 2.152.16
[loaB6loaB7] See the Mathematics of Investment and Credit solutions manual.

Exercise 2.17 [loa11]

When an Australian government bond is ex-interest, the owner of the bond at the
date of the bond going ex-interest receives the next coupon payment. The buyer
does not receive the next coupon payment. The buyer receives the maturity face
value and all coupons except the next payment.

Price is:
f
v d (C + Gan + 100v n )
0.0475
where the interest rate used is the semi-annual eective rate i= 2
= 0.02375,
and:

f = number of days to next coupon date (11 Jun 2006 to 15 Jun 2006)
d = number of days between coupon dates (15 Dec 2005 to 15 Jun 2006)
C = next coupon payment
G = regular semi-annual coupon payment
n = is the number of coupons to maturity

66
Financial Mathematics  Exercises School of Risk and Actuarial Studies  UNSW

5.75
Hence f = 4, d = 182, C = 0, G = 2
= 2.875, and n = 10. Price on 11 June 2006
is (next coupon is not received):

4
P = v 182 2.875a10 + 100v 10


= 0.999484 (2.875 × 8.808866 + 100 × 0.790789)


= 0.999484 × 104.404433
= 104.351

Exercise 2.18 [ann16]

See Sherris (p. 4043). Users that may be interested include superannuation funds
and anyone with long term liabilities that rise in line with ination (such as ination-
linked life annuities, or some general insurance liabilities). See also:
http://en.wikipedia.org/wiki/Ination-indexed_bond

Exercise 2.19 [loa12]

Using a spreadsheet model similar to Exercise 2.12, we obtain a price of $516,099.04.


The spreadsheet is provided in loa12.xls.

Exercise 2.20 [loa13]

A spreadsheet model can be used with the following equations (where time t =
0, 21 , . . . , 30 is in years):
(
0.035 t ≤ 26.5
it =
0.04 t > 26.5

150000 t = 10, 11, . . . , 18

N P Rt = 250000 t = 19, 20, . . . , 29

300000 t = 30


1.05 t = 10, 11, . . . , 18

Rt = 1.10 t = 19, 20, . . . , 29

1.12 t = 30

AP Rt = N P Rt × Rt
It = it · OBt− 1
2

OBt = OBt− 1 − N P Rt
X 2
P = vjt (AP Rt + It )
t

where j = 0.065 is the required


P eective yield. Also note that the original outstand-
ing balance (face value) is t N P Rt = 4, 400, 000.

The spreadsheet is provided in loa13.xls. The price is $4,797,748.66.

67
Financial Mathematics  Exercises School of Risk and Actuarial Studies  UNSW

Exercise 2.21 [loa14]

The price P is the present value of all cash ows at the net eective yield of j = 0.08.
This question is not too complex, and can be solved by hand. The complexity
arises from capital gains tax. However, note that the capital gain for the rst two
repayments (at times t = 2, 3) is:

400
400 − P <0
1200
since we are given P > 1200. Therefore, the only possibility of a capital gain is with
the 3rd repayment (at time t = 4). For the 3rd repayment, the capital gain is:

400 1
400(1.2) − P = 480 − P
1200 3
which will only be positive if P < 1440. Therefore, the capital gains tax for the 3rd
repayment is: (
0.20 480 − 13 P

P < 1440
CGT4 =
0 P ≥ 1440
Thus, we can solve this question by considering two cases: P < 1440 and P ≥ 1440.

(i) For P < 1440, there is CGT, and the price is given by:

P = (1 − 0.20) 0.10 1200v + 1200v 2 + 800v 3 + 400v 4


 
 
1
+ 400(v + v + 1.2v ) − 0.20 480 − P v 4
2 3 4
3

which can be solved to obtain P = 1249.47. This is consistent with the


assumption of P < 1440, so it is a valid solution.

(ii) For P ≥ 1440, there is no CGT, and the price is given by:

P = (1 − 0.20) 0.10 1200v + 1200v 2 + 800v 3 + 400v 4 + 400(v 2 + v 3 + 1.2v 4 )


 

= 1258.80

This is not consistent with the assumption of P ≥ 1440, so it is not a valid


solution.

Therefore, the price of the loan is $1249.47.

68
Financial Mathematics  Exercises School of Risk and Actuarial Studies  UNSW

6.3 Module 3

Exercise 3.1 [irr1]

(a) From the spot rates we can work out the discount factors:

 −1
0.04875180
v(0.5) = 1+ = 0.976204
2
 −2
0.05031182
v(1) = 1+ = 0.951525
2
 −3
0.05234408
v(1.5) = 1+ = 0.925421
2
 −4
0.05448436
v(2) = 1+ = 0.898067
2

and hence the value of the bond is:

6.75
P = [v(0.5) + v(1) + v(1.5) + v(2)] + 100v(2) = 102.467
2

(b) To determine the yield (eective yearly), we need to solve:

(2)
6.75a2 i + 100vi2 = 102.467

This can be done numerically to obtain i = 5.50537% (using the function


written earlier in Exercise 2.13). The equivalent semi-annual eective yield is
j = (1 + i)1/2 − 1 = 2.7158% (ie. i(2) = 5.43161%).

(c) The par yield of the 1 year coupon bond can be determined by solving:

100i [v(0.5) + v(1)] + 100v(1) = 100

or equivalently:
i[v(0.5) + v(1)] + v(1) = 1
1 − v(1)
∴i= = 0.025146
v(0.5) + v(1)
The equivalent nominal rate (payable semi-annually) is i(2) = 5.0292%.
Similarly, the par yield of the 2 year coupon bond is:

1 − v(2)
= 0.027173
v(0.5) + v(1) + v(1.5) + v(2)

which is equivalent to a nominal yield (payable semi-annually) of i(2) = 5.4346%.


This means that if these bonds have coupon rates equal to these par yields,
then their price will be 100 using the spot rates of this question.

69
Financial Mathematics  Exercises School of Risk and Actuarial Studies  UNSW

(d) Recall the following relationship between spot rates and forward rates, which
holds due to no arbitrage (ie. accumulation of $1 with certainty must be the
same under spot and forward rates):

(1 + st )t = (1 + st−1 )t−1 (1 + ft−1,t )


Using semi-annual time steps and nominal rates, we have

(2) (2)
s f0,0.5
1 + 0.5 = 1 +
2 2
(2) 2
! ! (2)
!
(2)
s1 s0.5 f0.5,1
1+ = 1+ 1+
2 2 2
!3 !2 (2)
!
(2) (2)
s1.5 s1 f1,1.5
1+ = 1+ 1+
2 2 2
(2) 4 (2) 3
! ! (2)
!
s2 s1.5 f1.5,2
1+ = 1+ 1+
2 2 2

(2) (2) (2) (2)


With the spot rates s0.5 , s1 , s1.5 and s2 given in the table, the forward rates
(2) (2) (2) (2)
f0,0.5 , f0.5,1 , f1,1.5 and f1.5,2 can be sequentially obtained as 4.8752%, 5.1873%,
5.6415%, and 6.0919%.

Exercise 3.2 [irr2]

A zero coupon bond can be constructed by purchasing a combination of the two


bonds (ie. x 9% bonds and y 7% bonds).

To have a net coupon each period of zero, we require:

9x + 7y = 0
To have a face value of $100 on maturity, we require:

x+y =1
Solving these simultaneously, we obtain x = −3.5 and y = 4.5. The price of the
ZCB is therefore:

P = −3.5 × 101.00 + 4.5 × 93.20 = 65.9

Exercise 3.3 [irr3]

For all t, by equating the accumulation of $1 with certainty, we obtain:

exp ((t − 1)s(t − 1)) exp (ft−1,t ) = exp (ts(t))


∴ (t − 1)s(t − 1) + ft−1,t = ts(t)
∴ ft−1,t = tr − (t − 1)r = r
which is a at forward rate curve.

70
Financial Mathematics  Exercises School of Risk and Actuarial Studies  UNSW

Exercise 3.4 [irr4]


(2)
Denote ft−1,t as the nominal forward rate (semi-annual compounding) for time period
(t − 1, t). The forward rates can be found by equating the accumulation of $1 over
the corresponding time periods:

(2)
!2 (2)
! (2)
!
f0,1 f0,0.5 f0.5,1
1+ = 1+ 1+ = (1.025)(1.0275) = 1.053188
2 2 2
(2)
!2 (2)
! (2)
!
f0.5,1.5 f0.5,1 f1,1.5
1+ = 1+ 1+ = (1.0275)(1.03) = 1.058325
2 2 2
(2)
!2 (2)
! (2)
!
f1,2 f1,1.5 f1.5,2
1+ = 1+ 1+ = (1.03)(1.0305) = 1.061415
2 2 2
(2)
!2 (2)
! (2)
!
f1.5,2.5 f1.5,2 f2,2.5
1+ = 1+ 1+ = (1.0305)(1.03125) = 1.062703
2 2 2
2  (2)
!
f2.5,3
 
f2,3 f2,2.5
1+ = 1+ 1+ = (1.03125)(1.035) = 1.067344
2 2 2

These can be solved to obtain:

(2)
f0,1 = 5.2498%
(2)
f0.5,1.5 = 5.7498%
(2)
f1,2 = 6.0500%
(2)
f1.5,2.5 = 6.1750%
(2)
f2,3 = 6.6247%

Exercise 3.5 [irr5]

The value of the bond is given by:

P = 4 [v(0.5) + v(1) + v(1.5) + v(2)] + 100v(2)

where v(t) is the discount factor associated with the t-year (semi-annual compound-
ing) spot rate:
1
v(t) =  (2) 2t
st
1+ 2

71
Financial Mathematics  Exercises School of Risk and Actuarial Studies  UNSW

v(0.5) = 0.977995 and v(1) = 0.949497 are calculated by substituting the given spot
rates, whereas v(1.5) and v(2) are determined using spot and forward rates:

1 1
v(1.5) =  3 =  2  
(2) (2) (2)
s1.5 s1 f1,1.5
1+ 2
1+ 2
1+ 2

1
=
0.0525 2 0.075082
 
1+ 2
1+ 2
= 0.915142
1 1
v(2) =  4 =  2   
(2) (2) (2) (2)
s2 s1 f1,1.5 f1.5,2
1+ 2 1+ 2
1+ 2
1+ 2

1
=
0.0525 2 0.075082 0.020290
  
1+ 2
1+ 2
1+ 2
= 0.905951

The resulting price is P = 105.591.

Exercise 3.6 [irr13]


9
The theoretical price is 6e(0.09−0.035)× 12 = $6.2527

Exercise 3.7 [irr14]

(a) P = 100(1.03)−91/365 = $99.266

(b) Let annual simple rate of discount = d


99.266 91
=1− d
100 365
d = 0.02945

Exercise 3.8 [irr15]

(a) Let in = spot rate for term n.

One year bond gives:

101 = 103vi1
101
v i1 = = 0.98058
103
103
i1 = − 1 = 1.980%
101

72
Financial Mathematics  Exercises School of Risk and Actuarial Studies  UNSW

Two year bond gives:

101 = 3vi1 + 103vi22


101 − 3 101
vi22 = 103
= 0.95202
103
i2 = 2.489%
Three year bond gives:

101 = 3vi1 + 3vi22 + 103vi33


101 − 3 × 0.98058 − 3 × 0.95202
vi33 = = 0.92429
103
i3 = 2.659%

(b) Forward rate f2,3 where:

(1 + i3 )3
1 + f2,3 = = 1.03
(1 + i2 )2
f2,3 = 3%

Exercise 3.9 [irr16]

(a) Issue price (per $100 nominal) is

4 4 4 105
+ 2
+ 3
+
1 + i1 (1 + i2 ) (1 + i3 ) (1 + i3 )3
where it is the t-year zero coupon rate at time t=0 and we have that:

(1 + it−1 )t−1 × (1 + ft−1,t ) = (1 + it )t


where ft−1,t is the one year forward rate at time t−1
1 + i1 = 1.04 (i1 is given)
(1 + i2 )2 = (1 + i1 )(1 + f1,2 )
= 1.04 × 1.05
(1 + i3 ) = (1 + i2 )2 (1 + f2,3 )
3

= 1.04 × 1.05 × 1.06


4 4 4 + 105
Issue Price = + +
1.04 1.04 × 1.05 1.04 × 1.05 × 1.06
= $101.68

(b) let yc3 be the 3 year par yield. Then yc3 is given by
 
1 1 1 100
100 = yc3 + + +
1 + i1 (1 + i2 )2 (1 + i3 )3 (1 + i3 )3
 
1 1 1 100
= yc3 + + +
1.04 1.04 × 1.05 1.04 × 1.05 × 1.06 1.04 × 1.05 × 1.06
= yc3 × 2.741205336 + 86.39159583
yc3 = 4.9644%

73
Financial Mathematics  Exercises School of Risk and Actuarial Studies  UNSW

Exercise 3.10 [irr17]

Let the oringinal price of the zero coupon bond =P

P = 100v 80 at 2.5%

= 13.87

Equation of value for the purchaser:

13.87eδt = 80 where t is the time of default from date of issue

ln(80/13.87)
t=
δ
ln(5.7678)
=
0.048
= 36.506 years

The default happened 36 years and 185 days since 1 January 1975. There are 181
days until the end of June, so default occurs on 4 July 2011.

Exercise 3.11 [irr18]

(a) Simple rate of return is (100 − 96)/96 = 0.041666


Expressed as an annual rate, this is:

365
0.041666 × = 8.3562%
182

(b) Let the time in years

97.5 − 96
= 0.035t
96
97.5 − 96
t=
0.035 × 96
= 0.44643 years
= 163 days

(c) Equation of value for the second investor:

97.5(1 + i)(182−163)/365 = 100


100
(1 + i)19/365 =
97.5
i = 62.640%

74
Financial Mathematics  Exercises School of Risk and Actuarial Studies  UNSW

Exercise 3.12 [irr19]

(a) We can nd forward rates f2,3 and f2,4 from:

(1 + y3 )3 = (1 + y2 )2 (1 + f2,3 ) and

(1 + y4 )4 = (1 + y2 )2 (1 + f2,4 )2

⇒ (1.033)3 = (1.032)2 (1 + f2,3 )


⇒ f2,3 = 3.50029%
and

(1.034)4 = (1.032)2 (1 + f2,4 )2


⇒ f2,4 = 3.60039%

(b) 1. Price per $100 nominal

2 3 3

4 v@3.1% + v@3.2% + v@3.3% + 115v@3.3%
= 4(0.969932 + 0.938946 + 0.907192) + 115 × 0.907192

2. Let yc3 = 3-year par yield

2 3 3

1 = yc3 v@3.1% + v@3.2% + v@3.3% + v@3.3%
= yc3 (0.969932 + 0.938946 + 0.907192) + 0.907192
⇒ yc3 = 0.032957
i.e. 3.2957%

Exercise 3.13 [irr20]

(a) Assuming no arbitrage,

i1 = f0,1 and (1 + i2 )2 = (1 + i1 )(1 + f1,2 )


Hence a − b = 0.061

⇒ a = b + 0.061
(1 + a − 2b)2 = 1.061 × 1.065

⇒ 1 + a − 2b = 1.061 × 1.065
⇒ b = −0.002
⇒ a = 0.059

(b) Firstly, nd the 4-year spot rate. Consider $1 nominal:

1 = 0.07(vi1 + vi22 + vi33 + vi44 ) + vi44


= 0.07(1.061−1 + 1.063−2 + 1.065−3 ) + 1.07 × vi44
⇒ (1 + i4 )4 = 1.31429212
⇒ i4 = 7.0713% p.a.

75
Financial Mathematics  Exercises School of Risk and Actuarial Studies  UNSW

Let bond price be P. Then

P = 0.05(vi1 + vi22 + vi33 + vi44 ) + 1.03 × vi44


= 0.05(1.061−1 + 1.063−2 + 1.065−3 ) + 1.08 × 1.0707134
= 0.9545

Exercise 3.14 [irr6]

We have:

P = D · ān + Re−δn
∂P ∂
= D ān − nRe−δn
∂δ ∂δ
where:
Z n
∂ ∂
ān = e−δt dt
∂δ ∂δ 0
Z n
∂ −δt
= e dt
0 ∂δ
Z n
=− te−δt dt
0
¯ n
= −(Iā)

Therefore, the duration is given by:

1 ∂P
D(δ) = − ·
P ∂δ
¯ n − nRe−δn
−D(Iā)
=−
D · ān + Re−δn
¯ n + nv n
g(Iā)
=
gān + v n

where g = D/R.

Exercise 3.15 [irr7]

(a) Consider a nominal amount of $1 of bond. The coupon income is g = 0.05 p.a.,
payable continually to redemption at time n. The duration is (from Exercise
3.14):
¯ n + nv n
g(Iā)
D(δ) =
gān + v n
at δ = 0.07, which results in 11.592 for n = 20 and 14.345 for n = 60.

(b) To nd the maximum duration, we need to dierentiate D(δ) w.r.t. n (and set

76
Financial Mathematics  Exercises School of Risk and Actuarial Studies  UNSW

this to zero). To do so, it is useful to note the following intermediate results:

∂ n
v = v n ln v = −δv n
∂n

nv n = nv n ln v + v n = (1 − δn) v n
∂n
∂ 1 ∂ 1
ān = (1 − v n ) = (δv n ) = v n
∂n δ ∂n δ
∂ ¯  1 ∂ 1
Iā n = (ān − nv n ) = (v n − (1 − δn)v n ) = nv n
∂n δ ∂n δ
Therefore:

¯ n + nv n (gv n − δv n )

∂ (gān + v n ) (gnv n + (1 − δn)v n ) − g(Iā)
D(δ) =
∂n (gān + v n )2

Setting this to zero:

¯ n + nv n (gv n − δv n )
0 = (gān + v n ) gnv n + (1 − δn)v n − g(Iā)
 

¯ n + nv n (g − δ) v n
= (gān + v n ) 1 + (g − δ)n v n − g(Iā)
 

¯ n − nv n v n
= (gān + v n ) + (g − δ) gnān + nv n − g(Iā)
  

¯ n vn
= (gān + v n ) + (g − δ) gnān − g(Iā)
  

Noting that we require an equation in terms of ān (ie. without ¯ n ):


v n and (Iā)
h g i
0 = (gān + 1 − δān ) + (g − δ) gnān − (ān − nv n )
δ
h g i
= 1 + (g − δ) ān + gnān − (ān − n + nδān )
δ i
h g
= 1 + (g − δ) ān − (ān − n)
δ
h g i
∴ (δ − g) ān − (ān − n) = 1
δ
g 1
∴ ān + (n − ān ) =
δ δ−g
δ
∴ δān + g(n − ān ) =
δ−g
as required (g = 0.05, δ = 0.07). This can then be solved numerically to
obtain n = 64.349 and a corresponding duration of 14.349.

Exercise 3.16 [new11]

(a) The modied duration and convexity are:

1 ∂P
MD = −
P ∂i
1 ∂ 2P
C=
P ∂i2
77
Financial Mathematics  Exercises School of Risk and Actuarial Studies  UNSW

(b) For a 10 year annuity, we note that:

1 − (1 + i)−10
P (i) =
i
∂P 10i(1 + i)−11 + (1 + i)−10 − 1
=
∂i i2
2
∂ P 2(−55i (1 + i)−12 − 10i(1 + i)−11 − (1 + i)−10 + 1)
2
=
∂i2 i3
For i = 0.05 we see that:
P (0.05) = 7.7217
1 ∂P
MD = −
P ∂i
= 4.856
1 ∂ 2P
C=
P ∂i2
= 35.602

(c) The rst and second derivatives of the price are approximated by:

∂P P (i + h) − P (i − h)
=
∂i 2h
∂ 2P P (i + h) − 2P (i) + P (i − h)
=
∂i2 h2
where h is a small increment in the interest rate. For this question, we shall
take h = 0.005. See Excel spreadsheet new11.xlsx. For h = 0.005, modied
duration is 4.8576 and convexity is 35.6093.

Exercise 3.17 [lif6]

Recall that:

X ∞
X
äx = k
v k px = (1 + i)−k k px
k=0 k=0
To nd the duration, we dierentiate w.r.t. i:
∞ ∞
d X X
äx = − k(1 + i)−k−1 k px = − kv k+1 k px
di k=0 k=0

Therefore the modied duration of äx is:


P∞ k
k=0 kv k px
MDx = v P ∞ l
l=0 v l px
and the Macaulay Duration is:
P∞ k ∞
MDx k=0 kv k px
X v k k px
Dx = = P ∞ l
= wk k, where wk = ∞ l .
P
v l=0 v l px k=0 l=0 v l px

Again, this is the weighted average of the payment maturities, where the weights
take into account both the time value of money and the probabilities of survival
(since the payments are contingent to the survival of (x)).

78
Financial Mathematics  Exercises School of Risk and Actuarial Studies  UNSW

Exercise 3.18 [dur1]

(a) Duration of the annuity payment is

(Ia)12 56.6328
= = 6.0343 years
a12 9.3851

(b) Duration of the bond is:

5(Ia)8 + 800v 8
5a8 + 100v 8
5 × 28.9133 + 800 × 0.73069
=
5 × 6.7327 + 100 × 0.73069
729.119
=
106.733
= 6.8313 years

(c) The duration of the assets (the bond) is greater than the duration of the
liabilities (pension payments). If there is a rise in interest rates, the present
value of the assets will fall by more than the present value of the liabilities and
the insurance company will make a loss.

Exercise 3.19 [dur2]

(a)

tCt v t
P
Duration = P
Ct v t
P3 t
 P10 t

t=1 4 × 0.04 tv + t=1 5 × 0.04 tv + 4 × 1 × 3v 3 + 5 × 1 × 10v 10
= P3  P 10 
t + t + 4 × 1 × v 3 + 5 × 1 × v 10
t=1 4 × 0.04 v t=1 5 × 0.04 v
0.16(Ia)3 + 0.20(Ia)10 + 12v 3 + 50v 10
=
0.16a3 + 0.20a10 + 4v 3 + 5v 10
0.16 × 5.2422 + 0.20 × 36.9624 + 12 × 0.83962 + 50 × 0.55839
=
0.16 × 2.6730 + 0.20 × 7.3601 + 4 × 0.83962 + 5 × 0.55839
46.2264
=
8.0501
= 5.742 years

(b) Let V denote the present value of the new portfolio per unit nominal.

1
V =
i
Volatility of new portfolio per unit nominal is

d
(V )
− di
V

79
Financial Mathematics  Exercises School of Risk and Actuarial Studies  UNSW

d 1
( )
= − di 1 i
i
1
=
i
Duration of new portfolio is:

1+i 1.06
Volatility × (1 + i) = =
i 0.06
= 17.67 years

(c) Present value of existing bonds = $8.0501 bn


Let the nominal amount of new bonds issues =X
0.05X
PV of new bonds = 0.05Xa∞ =
0.06

0.05X
= 0.8 × 8.05013
0.06
X = $7.728 bn

Exercise 3.20 [dur3]

vi∗ + 2 × vi2∗ + 3 × vi3∗ + . . . + 40 × vi40



D= 2 3 40
vi∗ + vi∗ + vi∗ + . . . + vi∗
i∗
(Ia)40
= ∗
ai40
Where
1.038835
vi∗ =
1.07
1.07
i∗ = − 1 = 0.03
1.038835
Hence

i ∗
(Ia)40
D= ∗
ai40
384.8647
=
23.1148
= 16.65 years

Exercise 3.21 [dur4]

(a) Working in 000's,

P VL = 200a20 + 300v 15
= 200 × 9.818147 + 300 × 0.315242
= 2058.20199
Present value is $2, 058, 201.99

80
Financial Mathematics  Exercises School of Risk and Actuarial Studies  UNSW

(b)

200v + 200 × 2v 2 + 200 × 3v 3 + . . . + 200 × 20v 20 + 300 × 15v 15


Duration =
200a20 + 300v 15
200(Ia)20 + 300 × 15v 15
=
2058.20199
200 × 78.9079 + 300 × 15 × 0.31524
=
2058.20199
= 8.3569 years

(c) Two conditions are:

P VL = P V A
DL = DA

Let the nominal amount in securities A and B be X and Y respectively.


Equate the present values

P VL = P VA
2, 058, 201.99 = X(0.09a12 + v 12 ) + Y (0.04a30 + v 30 )
= X(0.09 × 7.5361 + 0.39711) + Y (0.04 × 11.2578 + 0.09938)
= 1.075361X + 0.549689Y
2, 058, 201.99 − 0.549689Y
⇒X=
1.075361

Equate the durations

DL = DA
X(0.09(Ia)12 + 12v 12 ) + Y (0.04(Ia)30 + 30v 30 )
8.3569 =
2, 058, 201.99
⇒ 17, 200, 175 = X(0.09 × 42.17 + 12 × 0.39711) + Y (0.04 × 114.7136 + 30 × 0.09938)
= 8.56066X + 7.56986Y
Substitute for X

2, 058, 201.99 − 0.549689Y


17, 200, 175 = 8.56066 × + 7.56984Y
1.075361
⇒ Y = 255, 287
⇒ X = 1, 783, 470

Hence company should purchase $1,783,470 nominal of security A and $255,287


nominal of security B for Redington's rst two conditions to be satised.

(d) Redington's third condition is that the convexity of the asset cash ow series
is greater than the convexity of the liability cash ow series. Therefore the
convexities of the asset cash ows and the liability cash ows will need to be
calculated and compared.

81
Financial Mathematics  Exercises School of Risk and Actuarial Studies  UNSW

Exercise 3.22 [dur5]

(a)

10a40 = 10 × 19.7928
= $197.928m

(b) Call 10 year security security A and ve year security security B. We need
to calculate the PV of $100 nominal for each of security A and security B,
PV of $100 nominal of A is:

5a10 + 100v 10 = 5 × 8.1109 + 67.556


= 108.1105
PV of $100 nominal of B is:

10a5 + 100v 5 = 10 × 4.4518 + 100 × 0.82193


= $126.711
$98.964m is invested in each security

98, 964, 000


× 100 = 91, 539, 674
108.1105
$91,539,674 nominal of A is bought

98, 964, 000


× 100 = 78, 102, 138
126.711
$78,102,138 nominal of B is bought

(c) Duration is

10(Ia)40
10a40
10 × 306.3231
=
197.928
= 15.48 years

(d) Duration is:

(5(Ia)10 + 10 × 100v 10 ) × 915, 396.74 + (10(Ia)5 + 5 × 100v 5 ) × 781, 021.38


(10m)a40
using:

(Ia)10 = 41.9922
v 10 = 0.67556
(Ia)5 = 13.0065
v5 = 0.82193
we get

1, 223, 157, 000


Duration =
197, 928, 000
= 6.23 years

82
Financial Mathematics  Exercises School of Risk and Actuarial Studies  UNSW

(e) The duration (and therefore the volatility) is greater for the liabilities than
for the assets. As a result, when interest rates fall, the present value of the
liabilities will rise by more than the present value of the assets and so a loss
will be made.

Exercise 3.23 [dur6]

(a) The accumulation is

(12) (12) (12)


1200s̈20 (1.06)20 + 2300s̈20 + 100(Iä)20 (1.06)20
i
= (12) 1200s20 (1.06)20 + 2300s20 + 100(Ia)20 (1.06)20

d
= 1.032211(1200 × 36.7856 × 3.20714 + 2300 × 36.7856
+ 100 × 98.7004 × 3.20714)
= 266, 138

(b) Let half-yearly payment =X

Xa40 = 266, 138 at 2.5%


266, 138
⇒X=
25.1028
= 10, 601.94

Therefore, annual rate of payment = $21, 203.88

(c) Work in half-years. Duration is:

10, 601.94(v + 2v 2 + . . . + 40v 40 )


266, 138
10, 601.94(Ia)40
=
266, 138
10, 601.94 × 433.3248
=
266, 138
4, 584, 075
=
266, 138
= 17.26 half years
= 8.63 years

Exercise 3.24 [irr8]

(a) The present value of assets (5-year and 15-year bonds) and liabilities (10-year
ZCB) are:

VA (i) = M5 v 5 + M15 v 15
VL (i) = 1000v 10

where v = (1 + i)−1 .

83
Financial Mathematics  Exercises School of Risk and Actuarial Studies  UNSW

(b) A zero cost portfolio that provides a positive (non negative) prot with zero
chance of loss.

(c) To ensure that VA (i) = VL (i), we require:

M5 + M15 v 10 = 1000v 5

The duration of the assets and liabilities are:

tCt v t 5M5 v 5 + 15M15 v 15


P
DA (i) = Pt t
=
t Ct v M5 v 5 + M15 v 15
10 × 1000v 10
DL (i) = = 10
1000v 10
Therefore, to ensure that DA (i) = DL (i), we require:

5M5 v 5 + 15M15 v 15
= 10
M5 v 5 + M15 v 15
which simplies to:
M5 = M15 v 10
Solving these simultaneous equations, we get M5 = 340.29 and M15 = 734.66.
We can check that this is an arbitrage opportunity by looking at VA (i) − VL (i)
for each parallel shift in the at yield curve:

i VA (i) − VL (i)
7% 0.55
8% 0
9% 0.45

This shows that the surplus will be positive regardless of the direction in which
interest rates shift (as long as we have a parallel shift in a at yield curve).

Exercise 3.25 [irr9]

See Sherris p. 87.

84
Financial Mathematics  Exercises School of Risk and Actuarial Studies  UNSW

Exercise 3.26 [irr10]

At i = 4.5%, we have (for the liabilities):

X
VL = Ct v t
t
= 3v + 4v 2 + 3v 3 + 2v 4
= 10.84
tCt v t
P
DL = t
VL
3v + 8v 2 + 9v 3 + 8v 4
=
10.84
= 2.29
t(t + 1)Ct v t+2
P
CL = t
VL
6v + 24v 4 + 36v 5 + 40v 6
3
=
10.84
= 7.84

For the assets:

VA = M0.5 v 0.5 + M5 v 5
0.5M0.5 v 0.5 + 5M5 v 5
DA =
VA
Ensuring that VA = VL and DA = DL , we get:

M0.5 v 0.5 + M5 v 5 = 10.84


0.5M0.5 v 0.5 + 5M5 v 5 = 24.79

These can be solved to obtain M0.5 = 6.6803 and M5 = 5.3647.


The eect of a change in interest rates on the surplus is determined in Excel Spread-
sheet irr10.xlsx. The results are shown in the following table:

Rate VA VL S
6.5% 10.39 10.38 0.01
4.5% 10.84 10.84 0.00
2.5% 11.34 11.33 0.01
Twist (d) 10.32 10.67 0.35
Twist (e) 10.86 10.89 0.03

The negative surplus for the twist scenarios illustrate the failure of immunisation to
protect against non-parallel shifts in the yield curve.

Exercise 3.27 [new12]

(a) Refer to Excel Spreadsheet new12.xlsx (Worksheet 1) to nd a plot similar to


the one below.

85
Financial Mathematics  Exercises School of Risk and Actuarial Studies  UNSW

Spot Curve

0.058
0.056
Spot Rate

0.054
0.052
0.050

0 5 10 15 20

Term

(b) Fisher-Weil duration relaxes the assumption that the yield curve is at. In
practice, yield curves are rarely at. Therefore, dierent interest rates must
be used when present valuing cash ows of dierent maturities. By splitting
the cash ows into ZCBs of dierent maturities, we can value the cash ows by
valuing the ZCBs (which is done using spot rates). The time-weighted value
(duration) is also calculated using these principles.

(c) The immunisation strategy is found using the Fisher-Weil duration (since the
yield curve is no longer at):

X
VL = Ct (1 + st )−t
t
= 10.609
tCt (1 + st )−t
P
DL = t
VL
= 2.275
−(t+2)
P
t t(t + 1)Ct (1 + st )
CL =
VL
= 7.572

For the assets:

VA = M0.5 (1 + s0.5 )−0.5 + M5 (1 + s5 )−5


0.5M0.5 (1 + s0.5 )−0.5 + 5M5 (1 + s5 )−5
DA =
VA

86
Financial Mathematics  Exercises School of Risk and Actuarial Studies  UNSW

Ensuring that VA = VL and DA = DL , we get:

M0.5 (1 + s0.5 )−0.5 + M5 (1 + s5 )−5 = 10.609


0.5M0.5 (1 + s0.5 )−0.5 + 5M5 (1 + s5 )−5 = 24.134

These can be solved (with Solver in Excel - see spreadsheet new12.xlsx) to


obtain M0.5 = 6.588 and M5 = 5.516.

(d) See Excel Spreadsheet new12.xlsx for the simulations and histogram similar
to that of below.

Histogram of Surplus
150
100
Frequency

50
0

−0.02 −0.01 0.00 0.01 0.02

Surplus

As shown from the results and the graph, the portfolio is not fully immunised
as there is a chance we can make a loss.

Exercise 3.28 [irr11]

At i = 4%, we have:

VL = 10.96
DL = 2.29
CL = 7.94

87
Financial Mathematics  Exercises School of Risk and Actuarial Studies  UNSW

Letting P1 be the price of the 0.75 year bond, and P2 be the price of the 8 year
bond, we have (assuming annual coupons and $100 face value):

P1 = 100.99
D1 = 0.75
C1 = 1.21
P2 = 126.93
D2 = 6.43
C2 = 49.29

Let x1 be the number of units of the 0.75 yr bond, and x2 be the units of the 8 yr
bond. Therefore, we have:

V A = x1 P 1 + x2 P 2
x1 P1 D1 + x2 P2 D2
DA =
VA
Ensuring that:

VA = VL
DA = DL

we obtain two simultaneous equations which can be solved to give x1 = 0.07905


and x2 = 0.02345. These correspond to investing 7.983 and 2.977 in the two bonds
respectively.

A check of convexity shows that CA − CL = 7.49 > 0, so the portfolio is immunised.

Exercise 3.29 [irr12]

(a) At i = 6%, we have:

VL = 1.68
DL = 3
CL = 10.68

To form an immunised portfolio of bonds, it is clear that we need to use 2 or


more bonds with maturities on either side of 3 (as the portfolio duration would
be an average). Furthermore, to maximise the asset convexity (as immunisa-
tion suggests) we would wish to use the 1 year and 5 year bonds (Barbell
strategy). Using the 1 year and 5 year bonds, and solving using methods sim-
ilar to previous exercises, we obtain an investment strategy of $0.8396m into
both bonds (ie. face values of 0.8900 and 1.1236). The convexity of the assets
would be 14.240 > 10.680.

(b) Cashow matching involves nding assets that match the liability CF perfectly
without considering the costs. Since we have ZCBs available, we would simply
invest in a 3 year ZCB with a face value of $2m.

88
Financial Mathematics  Exercises School of Risk and Actuarial Studies  UNSW

(c) If interest rates increase to 7%, then for the immunised portfolio:

VA − VL = 0.8900v + 1.1236v 5 − 2v 3 = 0.0003

For the CF matching, we have:

VA − VL = 2v 3 − 2v 3 = 0

Therefore, both methods have surpluses of approximately zero (exact for CF


matching).

(d) If interest rates twist, then the surplus of the immunised portfolio is:

   5  3
1 1 1
0.8900 + 1.1236 −2 = −0.0588
1.04 1.08 1.06

The CF matching surplus is still exactly zero.

Exercise 3.30 [imm1]

(a) PV of assets is
5 20
5.5088v8% + 13.7969v8% = 6.7093
PV of liabilities is

8 15
6v8% + 11v8% = 6.7093 = PV of assets

Hence rst condition is satised.

Duration of assets is

5 20
5 × 5.5088v8% + 20 × 13.7969v8%
= 11.618
6.7093
Duration of liabilities is

8 15
8 × 6v8% + 15 × 11v8%
= 11.618 = Duration of assets
6.7093
Thus second condition is also satised

(b) Yes. The insurer is immunised against small changes in interest rate.

Exercise 3.31 [imm2]

(a) Present value of liabilities (in $m)

= 10a10 at 4%
= 10 × 8.1109
= 81.109

89
Financial Mathematics  Exercises School of Risk and Actuarial Studies  UNSW

(b) Duration is equal to

10(Ia)10
at 4%
10a10
41.9922
=
8.1109
= 5.1773 years

(c) Let the amounts to be invested in the two zero coupon bonds be X and Y.
Xv 3 + Y v 12 = 81.109
3Xv 3 + 12Y v 12 = 419.922
Solving simultaneously gives:

Y = $31.415m
X = $69.164m

Exercise 3.32 [imm3]

(a) The present value of the assets is equal to the present value of the liabilities.
The duration of the assets is equal to the duration of the liabilities. The spread
of the asset terms around the duration is greater than that for the liability
terms (or, equivalently, convexity of assets is greater).

(b) One of the problems of immunisation is that there is a need to continually


adjust portfolios. Once there is a change in interest rate, the entire portfolio
has to be restructured to immunize it again. This is resource intensive.
Another issue is that it may be dicult nding the right type of assets with
the necessary cashow structures that would allow a liability to be immunized.

6.4 Module 4

Exercise 4.1 [der1]

The theoretical futures index is determined as follows (accumulated value of spot


index and storage costs):

ft0 = St er(T −t) − ds̄T −t


= 293.3e0.07(210/365) − (0.035)(293.3)s̄210/365 0.07
= 305.353447 − (0.035)(293.3)(0.587085)
= 299.326724
This is close to the observed index of 299.0 (only 0.11% dierence).

Aside: The dierence between the actual and theoretical futures price is usually
quite small (less than 1%), except for major events (eg. 1987 nancial crisis). The
0
dierence (ft − St ) is called the futures to cash spread or basis, whereas (ft − St )
is the theoretical basis.

90
Financial Mathematics  Exercises School of Risk and Actuarial Studies  UNSW

Exercise 4.2 [der2]

The forward price will be the accumulated cost (purchase and funding) less coupons
received.

The price of the bond today is:

3.25a16 i + 100vi16 = 97.214171


0.0696
where i= 2
= 0.0348 is the semi-annual eective rate.

This price (purchasing cost) is accumulated 2 years at the funding costs:


" 12  12 #
0.065 0.07
97.214171 1+ 1+ = 111.223057
12 12

The accumulated value of coupons received at the funding costs is:

3.25s2 i1 (1 + i2 )2 + 3.25s2 i2 = 13.700113


0.065 6 0.07 6
 
where i1 = 1 + 12
−1 and i2 = 1 + 12
−1 are the semi-annual eective
funding costs.

The forward price is therefore 111.223057 − 13.700113 = 97.522944


The forward yield is the yield on the (6-year) bond which is locked in by the purchase
of the forward contract:

97.522944 = 3.25a12 j + 100vj12

which results in j = 3.50% per half-year, equivalent to an eective 7.013% p.a.

Exercise 4.3 [der3]

The current 90-day forward price implied by the market is (accumulated spot price
and costs):
 
90 90
f90 = 420 1 + (0.0975) + (0.025) (420) = 432.69
365 365
However, our short contract is for a forward price of $450, ie. we have an agreement
to sell gold for $450 in 90 days time. Thus, based on today's market conditions, we
should be able to make a prot of 450 − 432.69 = 17.31 in 90 days time. Today, this
prot is worth:
17.31
90 = $16.90
1 + (0.0975) 365

Exercise 4.4 [der4]

(Also see Example 6.9 in Sherris, p. 112) The forward price is the accumulated
costs less receipts (dividends). The cost of purchasing the shares now (including
transaction costs) is:
1.02 (10000 · 10) = 102000

91
Financial Mathematics  Exercises School of Risk and Actuarial Studies  UNSW

This is accumulated to account for funding costs over the next 6 months:

 6
0.06
102000 1 + = 105098.506
12

The accumulated dividends are:


 3
0.06
0.4 (10000) 1 + = 4060.3005
12

Therefore the forward price is 105098.506 − 4060.3005 = $101038.21

Exercise 4.5 [der5]

The cost of carry formula is based on no-arbitrage arguments. If it is violated, for


r(T −t)
example if Ft,T > St e − der(T −t1 ) , then one could short a forward, purchase the
asset with funds borrowed at the risk-free rate, invest the dividend at the risk-free
rate (at time t1 ), and exercise the forward contract at time T. This would lock in a
positive prot (at time T) with certainty at zero cost.

See lecture notes for further details. It may be helpful to draw a diagram.

Exercise 4.6 [der6]

For the 1st contract, the forward rate for days 30120 is 7.3%, whereas the 120-day
spot rate is 7.4%. Therefore, for no-arbitrage, we require a repo rate r given by:

    
30 90 120
1+ r 1+ · 0.073 = 1 + · 0.074
365 365 365

∴ r = 0.075639
However, the 30-day spot rate is 7.5% (< 7.5639%). Therefore, we could:

Action (t = 0) (t = 30) (t = 120)


Borrow $1m at 30-day spot (7.5%) 1,000,000 1,006,164 0
Short forward for $1,006,164 (7.3%) 0 1,006,164 1,024,275
Invest $999,948 in 120-day spot (7.4%) 999,948 0 1,024,275
Total 52 0 0

This results in a guaranteed prot of $52 at zero cost.

For the 2nd contract, the forward rate for days 90180 is 7.0%, whereas the 180-day
spot rate is 7.4%. Similarly, we have:

    
90 90 180
1+ r 1+ · 0.070 = 1 + · 0.074
365 365 365

∴ r = 0.076677
The 90-day spot rate is 7.4% (< 7.6677%). Therefore:

92
Financial Mathematics  Exercises School of Risk and Actuarial Studies  UNSW

Action (t = 0) (t = 30) (t = 120)


Borrow $1m at 90-day spot (7.4%) 1,000,000 1,018,247 0
Short forward for $1,018,247 (7.0%) 0 1,018,247 1,035,822
Invest $999,352 in 180-day spot (7.4%) 999,352 0 1,035,822
Total 648 0 0

This results in a guaranteed prot of $648 at zero cost.

Exercise 4.7 [der7]

See details in Sherris Example 7.1 (p. 133)

Exercise 4.8 [der8]

See details in Sherris Example 7.4 (p. 138)

Exercise 4.9 [der9]

See details in Sherris Example 7.10 (p. 149)

Exercise 4.10 [der10]

Let the random future commodity price be Sti . The swap provides for party A at
time ti :
Sti − X
The swap allows to get the dierence between the spot price at time ti and X
without actually exchanging the commodity. The swap price X is set such that the
initial value of the swap is 0.

To value the swap, consider what happens if party A enters into a series of short
forward positions (which cost nothing to enter in) to replicate the swap. In order to
settle the forward contract (buy the commodity and sell it to the long party at the
pre-specied price at the same time), the payo is

F0,ti − Sti
where F0,ti represents the forward price set at time 0 (now) for settlement at time
ti . The initial value of this forward is 0. Hence for times t1 , t2 , . . . , tM the net eect
of the swap and forward must be equal to 0:
M
X
0= e−rti [(Sti − X) + (F0,ti − Sti )]
i=1
M
X
F0,ti e−rti − Xe−rti

=
i=1
XM
S0 − Xe−rti

=
i=1
M
X
= M S0 − X e−rti
i=1

93
Financial Mathematics  Exercises School of Risk and Actuarial Studies  UNSW

M S0
∴ X = PM
−rti
i=1 e

Exercise 4.11 [swp1]

X 1.4% advantage over Y in the xed rate market, while the advantage in the
has a
oating rate market is only 0.5%. Thus X has the comparative advantage in the
xed rate market and Y the comparative advantage in the oating market.
Since the nancial institution takes 0.1% and the remainder is shared equally be-
tween the two companies, the benet to each company will be

1
(1.4% − 0.5% − 0.1%) = 0.4%
2
X will borrow on the xed market at 5% and Y will borrow on the oating market
at LIBOR +0.6%.
The nancial institution must replicate what X does against the market and borrow
from X at 5% xed.
In return, X borrows from the nancial institution at the rate adjusted by the
benet it receives. In this case, X borrows from the institution at the rate:

(LIBOR + 0.1%) − 0.4% = LIBOR − 0.3%

Similar steps need to be taken with Y. The nancial institution borrows from Y at
LIBOR +0.6%.
Y borrows from the nancial institution at:

6.4% − 0.4% = 6.0%

It can easily be seen that X and Y have both beneted 0.4% each from this arrange-
ment.
The bank has borrowed:

• 5% xed (from X)

• LIBOR +0.6% oating (from Y)

And it has lent:

• LIBOR −0.3% (to X)

• 6.0% xed (to Y)

The institution will prot 1.0% from its xed lending and borrowing whilst losing
0.9% from lending and borrowing on the oating market. This leaves them with a
0.1% net prot as required.

94
Financial Mathematics  Exercises School of Risk and Actuarial Studies  UNSW

Exercise 4.12 [for1]

(a) Assuming no arbitrage, the present value of dividends, I, is:

I = 0.5(e−0.05×(1/12) + e−0.05×(7/12) ) = 0.98355

Hence, forward price F = (10 − 0.98355)e0.05(9/12) = $9.3610

(b) The expected price of the share does not have to be taken into account because,
using the no-arbitrage assumption, the purchaser of the forward is simply able
to use the current price of the share (and the value of the dividends) given
that the forward is simply an alternative way of exposing the investor to the
same set of cash ows.
The expected future price of the share will be taken into account by investors
when determining the price they wish to pay for the share and therefore the
current share price.

Exercise 4.13 [for2]

(a) An investor can make a deal that would give her or him an immediate prot,
with no risk of future loss. Or an investor can make a deal that has zero initial
cost, no risk of future loss, and a non-zero probability of a future prot.

(b) The theoretical price per share of the forward contract is

9
$6e(0.09−0.035)× 12 = $6.2527

(c) In this case the actual forward price is too expensive in relation to the stock.

9
The investor should borrow $6e−0.035× 12 and use this to buy stock. The in-
vestor will also go short in one forward contract. The continuous dividends
are reinvested in the stock.

9 9
After nine months, the investor will have e0.035× 12 × e−0.035× 12 = 1 unit of
stock that can be sold under the terms of the forward contract for $6.30. The
9 9
−0.035× 12
investor will also have to repay cash of $e × e0.09× 12 = $6.2527

Exercise 4.14 [for3]

(a) Assuming no arbitrage, buying the share is the same as buying the forward
except that the cash does not have to be paid today and a dividend will be
payable from the share.
Therefore, price of forward is:

1 5 5
700(1.05) 12 (1.03) 12 − 20(1.03) 12

= 711.562 − 20.248 = 691.314

(b) The no arbitrage assumption means that we can compare the forward with
the asset from which the forward is derived and for which we know the market

95
Financial Mathematics  Exercises School of Risk and Actuarial Studies  UNSW

price. As such we can calculate the price of the forward from this, without
knowing the expected price at the time of settlement. The market price of the
underlying asset does, of course, already incorporate expectations.

(c) If it was not known with certainty that the dividend would be received we
could not use a risk-free interest rate to link the cash ows involved with the
purchase of the forward with all the cash ows from the underlying asset.

Exercise 4.15 [for4]

Forward price of the contract is K0 = (S0 − I)eδT = (68 − I)e0.14×1


8
where I is the present value of income during the term of the contract = 2.5e−0.14× 12
 8

⇒ K0 = 68 − 2.5e−0.14× 12 e0.14 = 75.59919

Forward price of a new contract issued at time r (3 months) is

9
Kt = (St − I ∗ )eδ(T −t) = (71 − I ∗ )e0.12× 12
5
I ∗ = 2.5e−0.12× 12 ⇒ K0.25 = 71 − 2.5e−0.05 e0.09 = 75.08435


(where I∗ is the present value of income during the term of the contract)

Value of original contract = (Kt − K0 )e−δ(T −t)


9
= (75.08435 − 75.59919)e−0.12× 12
= −0.47053

Exercise 4.16 [der11]

(a) Let (hC , BC ) be the units of stocks and bonds we hold in our portfolio to
replicate the payo of the call.

For the call option with exercise price 275, the payo function is :
(
max (285 − 275, 0) = 10 if stock price goes up
C1 =
max (250 − 275, 0) = 0 if stock price goes down

Hence we want to nd a portfolio such that the value at time 1 is:
(
hC 285 + BC e0.05 = 10 if stock price goes up
C10 =
hC 250 + BC e0.05 = 0 if stock price goes down

Solving these equations:

10
hC =
285 − 250  
−10
BC = e−0.05 (250)
285 − 250

96
Financial Mathematics  Exercises School of Risk and Actuarial Studies  UNSW

Since this portfolio pays the same amount as the option at time 1 regardless of
the stock price, to have an arbitrage free market this portfolio must be worth
the same as the option at time 0. Hence:

C0 = hC 260 + BC = 6.34

(b) For the put option with exercise price 275, the payo function is

(
max (275 − 285, 0) = 0 if stock price goes up
P1 =
max (275 − 250, 0) = 25 if stock price goes down

Hence we want to nd a portfolio such that the value at time 1 is:

(
hP 285 + BP e0.05 = 0 if stock price goes up
P10 =
hP 250 + BP e0.05 = 25 if stock price goes down

Solving these equations:

−25
hC =
285 − 250  
25
BC = e−0.05 (285)
285 − 250

Since this portfolio pays the same amount as the option at time 1 regardless of
the stock price, to have an arbitrage free market this portfolio must be worth
the same as the option at time 0. Hence:

P0 = hP 260 + BP = 7.93

(c) The put-call parity relationship states that (in words):

Call Value + Discounted Strike Price = Put value + Share Price

ie.
C0 + Xe−rt = P0 + S0
This is satised, since (for t = 1) :

C0 + Xe−rt = 6.34 + 275e−0.05 = 267.93


P0 + S0 = 7.93 + 260 = 267.93

Exercise 4.17 [der12]

For the call option with exercise price 50, the payo function is

(
max (55 − 50) = 5 if stock price goes up
C1 =
max (45 − 50, 0) = 0 if stock price goes down

97
Financial Mathematics  Exercises School of Risk and Actuarial Studies  UNSW

Hence we want to nd a portfolio holding (h, B) units of the stock and bond, such
that the value at time 1 is:
(
1

0 h55 + B 1 + 0.05 12 =5 if stock price goes up
C1 = 1

h45 + B 1 + 0.05 12 = 0 if stock price goes down

Solving these equations:

5
h=
55 − 45  
(45) −5
B= 1

1 + 0.05 12 55 − 45
Therefore:
C0 = h50 + B = 2.59

Exercise 4.18 [der13]

The pricing is done by ensuring that there are no arbitrage opportunities, rather than
through any particular probability assessments (except to decide where the stock
can go to, eg. up to 55 or down to 45). In terms of pricing, there is no dierence
between a model where the stock price can go up to 55 with probability 99% and
a model where the stock price can go up to 55 with probability 2%. An important
observation is that these contracts are priced as a function of the current underlying
asset price, that is, they are relative prices. The probability of movements in the
asset price do impact the underlying asset price but once this is known, these other
contracts are determined by no-arbitrage based on the current underlying asset price.

Exercise 4.19 [der14]

This option (which is no longer a simple call or put option) pays:


(
max (2852 − 2652 , 0) = 11000 if stock price goes up
X1 =
max (2502 − 2652 , 0) = 0 if stock price goes down

Hence we want to nd a portfolio holding (h, B) units of the stock and bond such
that the value at time 1 is:
(
0 h285 + Be0.04 = 11000 if stock price goes up
X1 =
h250 + Be0.04 = 0 if stock price goes down

Solving these equations:

11000
h=
285 − 250  
−0.04 −11000
B=e (250)
285 − 250
Since this portfolio pays the same amount as the option at time 1, to have an
arbitrage free market this portfolio must be worth the same as the option at time
0. Hence:
X0 = h260 + B = 6223.81

98
Financial Mathematics  Exercises School of Risk and Actuarial Studies  UNSW

Exercise 4.20 [der15]

Put call parity does not hold since:

K 22
C0 + = 1.2245 + = 22.1769
1+i 1.05
P0 + S0 = 20 + 2.5 = 22.5

Therefore we want to buy low (call and bond of face value 22 and maturity 1) and
sell high (put and share). The cost of this transaction today is:

22.1769 − 22.5 < 0

This is a prot, since it is a negative cost.

At time 1, the cash ows of the put, stock, call and bond will all cancel out.

Exercise 4.21 [der16]

The stock is assumed to either go up to su or down to sd . The bond starts o as 1


and rises toert at time t, where r is the risk free rate.
Suppose we want to price this derivative that pays xu or xd . For a call option, we
have:

xu = (su − K)+
xd = (sd − K)+

Consider a portfolio that holds φ stocks and ψ bonds. The value of this portfolio
today is:
V (0) = φs0 + ψ1
The value of this portfolio at time T is:

φ (su + D) + ψerT if the stock goes up

φ (sd + D) + ψerT if the stock goes down

Therefore, we need φ and ψ such that the payos are identical (so that this portfolio
will have the same value as our derivative):

φ (su + D) + ψerT = xu
φ (sd + D) + ψerT = xd

Solving these equations:

xu − xd
φ=
su − sd
   
−rT xu − xd
ψ=e xu − (su + D)
su − sd
The value of the portfolio with these φ and ψ will be the same as the value of our
derivative.

99
Financial Mathematics  Exercises School of Risk and Actuarial Studies  UNSW

Exercise 4.22 [der17]

Consider two investment portfolios:

• Portfolio A: one call plus cash of (K − c) e−rT


• Portfolio B: one put plus one unit of gold

The value of portfolio A at expiry is given by:


(
GT − K + K − c = GT − c if GT > K (i.e. option is exercised)

0+K −c=K −c if GT ≤ K (i.e. option is worthless)

The value of portfolio B at expiry is given by:


(
0 + GT − c = GT − c if GT > K (i.e. option is worthless)

K − GT + GT − c = K − c if GT ≤ K (i.e. option is exercised)

Therefore, both portfolios have a payo at expiry of:

max {K, GT } − c
Because the two portfolios have equal payo at expiry and the options cannot be
exercised before expiry, the portfolios must also have equal value for any time t < T.
In particular, at issue (t = 0), we must have:

c0 + (K − c)e−rT = p0 + S0
or:
c0 + Ke−rT = p0 + S0 + ce−rT

Exercise 4.23 [opt1]

(a) −37 + (495 − 480) = −22, a loss of $22 per contract purchased

(b) −12 + (180 − 150) = 18, a prot of $18 per contract purchased

(c) 22 + 0 = 22, a prot of $22 per contract purchased. Here, you collected the
premium and the option has expired worthless.

Exercise 4.24 [opt2]

(a) Let S denote the share price at expiry so that S ∼ U (90, 110). Its density is
therefore
1 1
fS (s) = = for 90 ≤ s ≤ 110
110 − 90 20
For the call option, the payo function is max(s − 105, 0). Its expected payo
therefore is Z 110
1 5
(s − 105) ds = = 0.625.
105 20 8
For the put option, the payo function is max(105 − s, 0). Its expected payo
therefore is Z 105
1 45
(105 − s) ds = = 5.625
90 20 8

100
Financial Mathematics  Exercises School of Risk and Actuarial Studies  UNSW

(b) Repeating these calculations assuming this time that S ∼ U (80, 120). Its
density is
1 1
fS (s) = = for 80 ≤ s ≤ 120
120 − 80 40
For the call option, the payo function is max(s − 105, 0). Its expected payo
is therefore Z 120
1 45
(s − 105) ds = = 2.8125
105 40 16
For the put option, the payo function is max(105 − s, 0). Its expected payo
is therefore Z 105
1 125
(105 − s) ds = = 7.8125
80 40 16
(c) Both distributions used for the share price have a mean of 100, but the volatil-
ity (i.e. standard deviation) has been doubled in part (b). Comparing the
answers, we see that the expected payos have increased for both options.
Their corresponding option values will also increase. This is exactly what we
would expect it to be if the volatility increased.

(d) In each case, the dierence between the expected payos is 5. This is ex-
actly consistent with the put-call parity relationship because we are eectively
nding the value of these options assuming they are about to expire. So the
dierence between the value of the put and the call should equal the dierence
between the strike price (equal to 105) and the average share price at expiry
(equal to 100).

Exercise 4.25 [opt3]

(a) The put-call parity theorem states that the values of the call and the put
options are related as follows:

ct + Ke−r(T −t) = pt + St
Simply substituting the values we get:

42 + 500e−0.06(0.25) = pt + 480
Solving for pt gives:
pt = 45.56

(b) This calculation assumes that the markets in which the share and the options
are traded are arbitrage-free and that no dividends are payable.

Exercise 4.26 [opt4]

(a) A European put option gives you the option to sell a share on the expiry date
for a specied price. If the current share price fell, you would expect future
share prices to be lower as well. Your option to sell the share at the specied
price would now become more valuable and the value of the put would therefore
increase.

101
Financial Mathematics  Exercises School of Risk and Actuarial Studies  UNSW

(b) If there was a sudden increase in the risk-free rate of interest, owning the put
option (which you could sell and convert to cash) becomes relatively more
expensive since you are losing more interest on the cash you eectively have
tied up in it. So the value of the put option would go down.

Exercise 4.27 [opt5]

(a) Consider two portfolios:

A: one call plus cash of Ke−r(T −t) = c0 + 10.15e−0.04×2


B: one put plus one share = p0 + S0

Both portfolios have a payo at time of expiry (t = 2) of max(K, ST ) =


max(10.15, S2 )
Since they have the same value at expiry and since the options cannot be
exercised before then they should have the same value at any time t < T, by
no-arbitrage: that is
ct + Ke−r(T −t) = pt + St
⇒ c0 + 10.15e−0.04×2 = p0 + S0

(b) Using above relationship and substituting for price of put and call options,
S0 = $12.80

Exercise 4.28 [opt6]

(a) • a purchased call option with strike price K1 and

• a written call option with strike price K2 , where K1 < K2

(b) • a purchased put with strike price K1 and

• a written put with strike price K2 , with K1 < K2

(c) • a written call with strike K1 and

• a purchased call with strike K2 , with K1 < K2

(d) • a written put with strike K1 and

• a purchased put with strike K2 , with K1 < K2

(e) A call and a put option purchased or written with the same strike price.

102
Financial Mathematics  Exercises School of Risk and Actuarial Studies  UNSW

6.5 Module 5

Exercise 5.1 [sto1]

(a) The mean accumulation is:

E(1000S10 ) = 1000E [(1 + i1 ) . . . (1 + i10 )]


= 1000E(1 + i1 ) . . . E(1 + i10 ) (by independence)

= 1000 [E(1 + i)]10 (identically distributed)

= 1000(1.06)10
= 1790.85

(b) The variance of the accumulation is:

= 10002 E S10
2
− [E (S10 )]2
  
Var(1000S10 )

where:

2
= E (1 + i1 )2 . . . (1 + i10 )2
  
E S10
10
= E (1 + i)2

10
= 0.4(1.04)2 + 0.2(1.06)2 + 0.4(1.08)2


= 1.1239210
Therefore:
Var(1000S10 ) = 9145.60
and:
σ(1000S10 ) = 95.63

(c) (i) The mean accumulation depends only on the mean interest rate, which
is not changed. However, the variance of the accumulation will be lower,
as the variance in interest rates is lower.

(ii) The mean will be larger as we are accumulating over a longer period. The
standard deviation will also be larger, as investing in a longer term will
result in a greater spread of possible accumulated amounts.

Exercise 5.2 [sto2]

(a) The mean accumulation is:

E(Sn ) = E [(1 + i1 ) · · · (1 + in )]
= E(1 + i1 ) · · · E(1 + in ) (by independence)
= [E(1 + i)]n (identically distributed)
= (1 + j)n
The variance of the accumulation is:

= E Sn2 − [E (Sn )]2



Var(Sn )

103
Financial Mathematics  Exercises School of Risk and Actuarial Studies  UNSW

where:

E Sn2 = E (1 + i1 )2 · · · (1 + in )2
  
n
= E (1 + i)2

n
= E 1 + 2i + i2

n
= 1 + 2j + j 2 + s2
noting that E(X 2 ) = Var(X) + [E(X)]2 for a random variable X. Therefore:

2 n
= 1 + 2j + j 2 + s − (1 + j)2n

Var(Sn )

(b) E(S8 ) = 1.59385



(c) σ(S8 ) = 2.65844 − 2.54035 = 0.34364

Exercise 5.3 [sto3]

(a) The mean accumulation is:

E(10000S2 ) = 10000E [(1 + i1 )(1 + i2 )]


= 10000E [1 + i] E [1 + i2 ] (iid)
 
1 1 1
= 10000 1.03 + 1.04 + 1.06 (0.7 · 1.05 + 0.3 · 1.04)
3 3 3
= 10923.70

(b) The variance of the accumulation is:

= 100002 E S22 − [E (S2 )]2


  
Var(10000S2 )

where:

E S22 = E (1 + i1 )2 (1 + i2 )2
  

= E (1 + i1 )2 E (1 + i2 )2
   
 
1 2 1 2 1 2
0.7 · 1.052 + 0.3 · 1.042

= 1.03 + 1.04 + 1.06
3 3 3
= 1.193465
Therefore:
Var(10000S2 ) = 19278

Exercise 5.4 [sto4]

(a) The mean and variance are:

i1 + i2
E(i) =
2
2 2
Var(i) = E(i ) − [E(i)]

1 1
= (i21 + i22 ) − (i1 + i2 )2
2 4
1
= (i1 − i2 )2
4
104
Financial Mathematics  Exercises School of Risk and Actuarial Studies  UNSW

(b) The mean and variance of the accumulated value can be determined using the
formulae in Exercise 5.2:

E (Sn ) = (1 + j)n
n
Var(Sn ) = 1 + 2j + j 2 + s2 − (1 + j)2n

Therefore:

5.5 = (1 + j)25 ⇒ j = 0.0705686


25
0.52 = 1 + 2j + j 2 + s2 − (1 + j)50 ⇒ s2 = 0.000377389

Solving for i1 and i2 :


i1 + i2
= j = 0.0705686
2
∴ i1 + i2 = 0.1411372
1
(i1 − i2 )2 = s2 = 0.000377389
4
∴ i1 − i2 = 0.0388530
Therefore, we obtain i1 = 0.089995 and i2 = 0.051142.

Exercise 5.5 [sto5]

(a) The single premium X is given by:

X [1 + E(i)]10 = 10000

∴ X [0.3(1.07) + 0.5(1.08) + 0.2(1.10)]10 = 10000


∴ X = 4589.26

(b) Expected prot is:

E(π) = E X(1 + i)10 − 10000


 

= X 0.3(1.07)10 + 0.5(1.08)10 + 0.2(1.10)10 − 10000




= 42.94

Exercise 5.6 [iid1]

(a) Let S20 be the accumulation of the unit investments after 20 years:

E(S20 ) = E[(1 + i1 )(1 + i2 ) . . . (1 + i20 )]


= E[1 + i1 ]E[1 + i2 ] . . . E[1 + i20 ] as it are independent

E[it ] = j ∴ E[S20 ] = (1 + j)20 = 2


1
⇒ j = 2 20 − 1 = 3.5265%

105
Financial Mathematics  Exercises School of Risk and Actuarial Studies  UNSW

(b) The variance of the eective rate of return per annum is s2 where

20
V ar[Sn ] = (1 + j)2 + s2 − (1 + j)40
= 0.62
h 2 i 201
s2 = 0.62 + (1 + j)20 − (1 + j)2
1 1
= (0.62 + 22 ) 20 − 2 10
= 0.004628

Exercise 5.7 [iid2]

(a) Value of the annuity is

(12)
20, 000ä1 (1 + 1.03v + 1.032 v 2 + . . . + 1.0319 v 19 )
1.03 20

1 − 1.07
= 20, 000 × 1.037525 × 0.93458 ×
1 − 1.03
1.07
= 19, 393.4173 × 14.26488
= $276, 645

(b) Let S5 = Accumulation of $1 after 5 years and let it = investment return for
year t.

5
" #
Y
E[S5 ] = E (1 + it )
t=1
5
Y
= E[1 + it ] using independence
t=1
Y5
= (1 + E[it ])
t=1

Now E[it ] = 0.6 × 0.04 + 0.4 × 0.07 = 0.052

⇒ Expected accumulation = 200, 000E[S5 ]


= 200, 000 × (1.052)5
= 200, 000 × 1.288483
= 257, 696.60

(c) The variance of the accumulation is

200, 0002 × E S52 − E [S5 ]2


  

106
Financial Mathematics  Exercises School of Risk and Actuarial Studies  UNSW

Where:
" 5 #
 2 Y
E S5 = E (1 + it )2
t=1
" 5
#
Y
1 + 2it + i2t

=E
t=1
5
Y
1 + 2E[it ] + E[i2t ]

= from independence
t=1
Now, E[i2t ] = 0.6 × 0.042 + 0.4 × 0.072
= 0.00292
Hence, E[S5 ] = (1 + 2 × 0.052 + 0.00292)5
2

= 1.661809

Variance is 200, 0002 × (1.661809 2


√ − 1.288483 ) = 64, 822, 348.44
Hence, standard deviation is 64, 822, 348.44 = $8, 051.23

(d) Consider under which circumstances the accumulation is higher than the price
of the annuity.
Note that if there is one period of 4% return, accumulation is

200, 000 × (1.07)4 (1.04) = 272, 645.57 < 276, 639

thus insucient funds.


With all 5 periods yielding a 7% return, however, the accumulation is

200, 000 × (1.07)5 = 280, 510.35 > 276, 639

and so funds are sucient.

Hence the individual requires annual returns of 7% for every single year. The
probability of this is
(0.4)5 = 0.01024 = 1.024%
Be careful not to assume log-normal returns.

Exercise 5.8 [iid3]

(a) The expected annual interest rate in the rst ten years is 0.4×0.04+0.7×0.06 =
0.054. The expected interest rate in the second ten years is clearly 5.5%
If the premium is calculated on the basis of these interest rates, then the
premium will be P such that:

20, 000 = P (1.054)10 (1.055)10


⇒ 20, 000 = 2.89022P
⇒ P = 6, 919.89

107
Financial Mathematics  Exercises School of Risk and Actuarial Studies  UNSW

(b) The expected accumulation factor in the rst ten years is

0.3 × (1.04)10 + 0.7 × (1.06)10 = 1.69767

The expected accumulation factor in the second ten years is:

0.5 × (1.05)10 + 0.5 × (1.06)10 = 1.70987

As they are independent, we can multiply them to get an expected accumula-


tion of
6919.89 × 1.69767 × 1.70987 = 20, 087.04
The expected prot is $87.04

(c) The prot is a function of random interest rates. Mathematically speaking,


the expected value of a function of a random variable is not necessarily equal
to the function of the expected value of the random variable.

E[Prot(interest rates)] 6= Prot(E[interest rates]) =0

(d) 1. The highest possible outcome is

(1.06)10 (1.06)10 = 3.20714 with probability 0.7 × 0.5 = 0.35

The lowest possible outcome is

(1.04)10 (1.05)10 = 2.41116 with probability 0.3 × 0.5 = 0.15

The range is therefore: 6, 919.89(3.20714 − 2.41116) = 5, 508.05


2. The other two possible outcomes are

(1.06)10 × (1.05)10 = 2.91710 with probability 0.7 × 0.5 = 0.35


(1.04)10 × (1.06)10 = 2.65089 with probability 0.3 × 0.5 = 0.15

The mean accumulation factor is 2.90280

The variance of the accumulation is:

0.35(3.20714 − 2.90280)2 + 0.15(2.41116 − 2.90280)2


+ 0.35(2.91710 − 2.90280)2 + 0.15(2.65089 − 2.90280)2
= 0.03241 + 0.03626 + 0.00007 + 0.00952
= 0.07826

Standard deviation is 0.07826 = 0.27976
Standard deviation of the accumulation of the whole premium is

6, 919.89 × 0.27976 = $1, 935.88

Which is also the standard deviation of the prot.

108
Financial Mathematics  Exercises School of Risk and Actuarial Studies  UNSW

Exercise 5.9 [sto6]

(a) The mean of accumulated premiums is:

E(P ) = 425000(1.03)5 + 425000E [(1 + i1 ) · · · (1 + i5 )]


= 425000(1.03)5 + 425000 [E(1 + i)]5 (since 1 + it iid)

= 425000(1.03)5 + 425000(1.035)5
= 997458

The standard deviation is found as follows:

5
!
Y
Var(P ) = Var 425000(1.03)5 + 425000 (1 + ik )
k=1
5
!
Y
= 4250002 Var (1 + ik )
k=1
5
!
Y
2
= 425000 E (1 + ik ) 2
− 4250002 (1.035)10
k=1
5
!
Y 5
(1 + ik )2 = E (1 + i)2

E
k=1
5
= E 1 + 2i + i2

5
= 1 + 2E(i) + [E(i)]2 + Var(i)

5
= 1 + 2(0.035) + (0.035)2 + (0.03)2
= 1.416534
p
∴ σ(P ) = Var(P ) = 32743.21

(b) Investing all premiums in the risky assets is likely to be more risky because
although there may be a higher probability of the assets accumulating to more
than $1 million, the standard deviation would be twice as high so the proba-
bility of a large loss would also be greater.

Exercise 5.10 [sto7]

(a) The mean accumulation is:

E(1000S10 ) = 1000E [(1 + i1 ) · · · (1 + i10 )]


= 1000E(1 + i1 ) · · · E(1 + i10 ) (by independence)
10
= 1000 [E(1 + i)] (identically distributed)

= 1000(1.07)10
= 1967.15

The variance of the accumulation is:

= 10002 E S10
2
− [E (S10 )]2
  
Var(1000S10 )

109
Financial Mathematics  Exercises School of Risk and Actuarial Studies  UNSW

where:

2
= E (1 + i1 )2 · · · (1 + i10 )2
  
E S10
10
= E (1 + i)2

10
= 1 + 2E(i) + [E(i)]2 + Var(i)

10
= 1 + 2(0.07) + (0.07)2 + (0.09)2


= 1.115310

Therefore, we have:
p
σ(1000S10 ) = Var(1000S10 ) = 531.65

(b) We want to nd PrQ(1000S10 < 0.5(1967.15)). The distribution of the accumu-
lation term S10 = 10k=1 (1 + ik ) is determined as follows:

(1 + ik ) ∼ LN (µ, σ 2 )
∴ ln(1 + ik ) ∼ N (µ, σ 2 )
10
X
∴ ln(1 + ik ) ∼ N (10µ, 10σ 2 )
k=1
10
! 10
X Y
∴ exp ln(1 + ik ) = (1 + ik ) ∼ LN (10µ, 10σ 2 )
k=1 k=1

where:

1 2
E(1 + i) = 1.07 = eµ+ 2 σ
2
 2 
Var(1 + i) = 0.092 = e2µ+σ eσ − 1
 2 
∴ 0.092 = (1.07)2 eσ − 1
∴ σ 2 = 0.007050
1
∴ µ = ln 1.07 − (0.007050) = 0.064134
2
Therefore:

Pr (1000S10 < 0.5(1967.15)) = Pr (S10 < 0.983575)


 
ln 0.983575 − 10µ
= Pr Z < √
10σ
where Z ∼ N (0, 1)

= Pr (Z < −2.4778)
= 0.00661

110
Financial Mathematics  Exercises School of Risk and Actuarial Studies  UNSW

(c) The probability required is:

 
7
Pr (1200S10 < 1400) = Pr S10 <
6
ln 76 − 10µ
 
= Pr Z < √
10σ
= Pr (Z < −1.8349)
= 0.03326

Exercise 5.11 [sto8]

(a) The mean accumulation is:

E(S10 ) = E [(1 + i1 ) · · · (1 + i10 )]


= E(1 + i1 ) · · · E(1 + i10 ) (by independence)
10
= [E(1 + i)] (identically distributed)

= (1.06)10

Therefore:

E (2, 000, 000 S10 ) = 2, 000, 000 (1.06)10 = 3, 581, 695

(b) The distribution of S10 is lognormal:

(1 + ik ) ∼ LN (µ, σ 2 )
∴ ln(1 + ik ) ∼ N (µ, σ 2 )
10
X
∴ ln(1 + ik ) ∼ N (10µ, 10σ 2 )
k=1
10 10
!
Y X
∴ S10 = (1 + ik ) = exp ln(1 + ik ) ∼ LN (10µ, 10σ 2 )
k=1 k=1

where:

1 2
E(1 + i) = 1.06 = eµ+ 2 σ
2
 2 
Var(1 + i) = 0.082 = e2µ+σ eσ − 1
 2 
∴ 0.082 = (1.06)2 eσ − 1
∴ σ 2 = 0.0056798
1
∴ µ = ln 1.06 − (0.0056798) = 0.055429
2

111
Financial Mathematics  Exercises School of Risk and Actuarial Studies  UNSW

Therefore, the required probability is:

Pr (S10 < 0.8 E(S10 )) = Pr S10 < 0.8(1.06)10




= Pr (S10 < 1.4327)


 
ln 1.4327 − 10µ
= Pr Z < √
10σ
= Pr (Z < −0.8171)
= 0.207

Exercise 5.12 [sto9]

We have (1 + i) ∼ LN (µ, σ), where:


1 2
E(1 + i) = eµ+ 2 σ = 1.001
 2 
2µ+σ 2
Var(1 + i) = e eσ − 1 = 0.0022

These can be solved simultaneously to obtain µ and σ2:


1 2 2
   2 
Var(1 + i) = eµ+ 2 σ eσ − 1
 2 
2 2 σ
∴ 0.002 = (1.001) e − 1
∴ σ 2 = 0.000003992
1
∴ µ = ln 1.001 − (0.000003992) = 0.0009975
2
Therefore, we can nd j as follows:

0.05 = Pr(i < j)


= Pr(1 + i < 1 + j)
= Pr(ln(1 + i) < ln(1 + j))
 
ln(1 + j) − µ
= Pr Z <
σ
ln(1 + j) − µ
∴ = −1.645
σ
∴ ln(1 + j) = −0.00228921
∴ j = e−0.00228921 − 1 = −0.2287%

Exercise 5.13 [sto10]

Note: This question is similar to Exercise 5.11.

(a) The mean accumulation is:

E(S10 ) = E [(1 + i1 ) · · · (1 + i10 )]


= E[1 + i1 ] · · · E[1 + i10 ] (by independence)
10
= [E(1 + i)] (identically distributed)

= (1.06)10

112
Financial Mathematics  Exercises School of Risk and Actuarial Studies  UNSW

Therefore:

E (1, 000, 000 S10 ) = 1, 000, 000 (1.06)10 = 1, 790, 848

(b) The distribution of S10 is lognormal:

(1 + ik ) ∼ LN (µ, σ 2 )
∴ ln(1 + ik ) ∼ N (µ, σ 2 )
10
X
∴ ln(1 + ik ) ∼ N (10µ, 10σ 2 )
k=1
10 10
!
Y X
∴ S10 = (1 + ik ) = exp ln(1 + ik ) ∼ LN (10µ, 10σ 2 )
k=1 k=1

where:
1 2
E(1 + i) = 1.06 = eµ+ 2 σ
2
 2 
Var(1 + i) = 0.082 = e2µ+σ eσ − 1
 2 
∴ 0.082 = (1.06)2 eσ − 1
∴ σ 2 = 0.0056798
1
∴ µ = ln 1.06 − (0.0056798) = 0.055429
2
Therefore, the required probability is:

Pr (S10 < 0.9 E(S10 )) = Pr S10 < 0.9(1.06)10




= Pr (S10 < 1.61172)


 
ln 1.61172 − 10µ
= Pr Z < √
10σ
= Pr (Z < −0.32304)
= 0.373

Exercise 5.14 [log1]

(a)
1 2
E(1 + i) = eµ+ 2 σ
1
= e0.05+ 2 ×0.004
= 1.0533757
∴ E[i] = 0.0533757
Let A be the accumulation at the end of 25 years.

Then E[A] = 3000s̈25 at rate j = 0.0533757


((1 + j)25 − 1)
= 3000 × (1 + j)
j
= $158, 036.43

113
Financial Mathematics  Exercises School of Risk and Actuarial Studies  UNSW

(b) Let the accumulation be S20


S20 has a log normal distribution with parameters 20µ and 20σ 2
1 2
∴ E[S20 ] = e20µ+ 2 ×20σ
= e20×0.05+10×0.004
= e1.04
= 2.829217

ln(S20 ) ∼ N (1, 0.08)

P r(S20 > 2.829217) = P r(ln(S20 ) > ln(2.829217))


 
ln(2.829217) − 1
= Pr Z > √ where Z ∼ N (0, 1)
0.08
= P r(Z > 0.14)
= 1 − Φ(0.14)
= 0.444

Exercise 5.15 [log2]

(a)

E(1 + it ) = 1.06
V ar(1 + it ) = 0.032 = 0.0009

1 2
⇒ 1.06 = eµ+ 2 σ
2
 2 
0.0009 = e2µ+σ eσ − 2
1 2 2
   2 
0.0009 = eµ+ 2 σ eσ − 2
 2  1 2
2 σ
0.0009 = 1.06 e − 2 by substitution eµ+ 2 σ = 1.06

Solving for σ2 gives


σ 2 = 0.000800676
0.000800676
⇒ 1.06 = e(µ+ 2 )

⇒ µ = 0.0578686

(b) 1. Assets would accumulate to 14 × 1.04 = 14.56 < 15 therefore 100%

2. The guaranteed portion of the fund would accumulate to

0.25 × 14 × 1.04 = 3.64

∴ non-guaranteed portion of the fund needs to accumulate to 15 − 3.64 =


11.36

114
Financial Mathematics  Exercises School of Risk and Actuarial Studies  UNSW

We need to nd the probability:

P r((0.75 × 14)(1 + it ) < 11.36)


= P r((1 + it ) < 1.081905)
= P r(ln(1 + it ) < ln(1.081905))
 
ln(1 + it ) − µ ln(1.081905) − µ
= Pr <
σ σ
 
ln(1.081905) − 0.0578686
= Pr Z < Where Z ∼ N (0, 1)
0.0282962
= Φ(0.7370169)
= 0.77

(c) 1. Return is xed so variance is 0

2. Return from portfolio = 0.25 × 0.04 + 0.75it


∴ Variance of return = 0.752 V ar(it )
0.752 × 0.0009 = 0.00050625
The variance of return is

($14m)2 × 0.00050625 = $2 99, 225m


Which is the equivalent of a standard deviation of $315, 000

Exercise 5.16 [log3]

(a)
1 2
E[1 + i] = eµ+ 2 σ
1
= e0.05+ 2 ×0.004
= 1.0533757

∴ E[i] = 0.0533757 Let A be the accumulation of $5000 at the end of 20 years.


E[A] = 5000s̈20 at rate j = 0.0533757
((1 + j)20 − 1)
= 5000 × (1 + j)
j
(1.053375720 − 1)
= 5000 × 1.0533757
0.0533757
= $180, 499

(b) Let the accumulation be S20


S20 has a log normal distribution with parameters 20µ and 20σ 2
1 2
∴ E[S20 ] = e20µ+ 2 (20σ )
= exp(20 × 0.05 + 10 × 0.0004)
= e1.04
= 2.829217

115
Financial Mathematics  Exercises School of Risk and Actuarial Studies  UNSW

ln S20 ∼ N (20µ, 20σ 2 )


ln S20 ∼ N (1, 0.08)

P r S20 > e1.04 = P r (ln S20 > 1.04)



 
1.04 − 1
= Pr Z > √ where Z ∼ N (0, 1)
0.08
= P r(Z > 0.14)
= 1 − Φ(0.14)
= 1 − 0.56
= 0.44

Exercise 5.17 [log4]

(a)
1 + it ∼ LogN ormal(µ, σ 2 )

12
Y
S12 = (1 + it )
1
12
X
⇒ ln(S12 ) = ln(1 + it ) ∼ N (12µ, 12σ 2 )
1

E[1 + it ] = 1.08 = exp(µ + σ 2 /2)


V ar[1 + it ] = 0.052 = exp(2µ + σ 2 )(exp(σ 2 ) − 1)
= 1.082 exp(σ 2 ) − 1


2 0.052
⇒ eσ = 1 +
1.082
⇒ σ 2 = 0.002141053
1
µ = ln(1.08) − σ 2
2
= 0.075890514

Hence S12 has LogNormal distribution with parameters 0.910686 and 0.025692636

(b) PV of annuity at time 12:

(12) (4) (4)


P V = 4000 ä4 +5000 ä2 v 4 +5, 000 ä2 |{z}
v 6 +6000 ā4 v 2 |{z}
v6
|{z} | {z } |{z} 7% |{z}
7% 9%
7% 7% 9%
= 1000(4 × 1.037525 × 3.3872 + 5 × 1.043380 × 1.8080 × 0.76290
+ 5 × 1.055644 × 1.7591 × 0.66634
+ 6 × 1.044354 × 3.2397 × 0.84168 × 0.66634)
= 1000 × (14.057219 + 7.195791 + 6.186911 + 11.385358)
= 38, 825.28

116
Financial Mathematics  Exercises School of Risk and Actuarial Studies  UNSW

Hence

P rob(18, 000S12 ≥ 38, 825.28) = P rob(S12 ≥ 2.15696)


 
ln(2.15696) − 0.910686
= P rob Z ≥ √
0.025692636
= P rob(Z ≥ −0.8858)
= Φ(0.89)
= 0.81

Exercise 5.18 [new3]

(a) The accumulated value based on a principal of $1 is:

S10 = (0.05 − i1 )(1 + i2 ) · · · (1 + i10 ) + · · · + (0.05 − i10 )

The mean of S10 is:

E(S10 ) = E[(0.05 − i1 )(1 + i2 ) · · · (1 + i10 ) + · · · + (0.05 − i10 )]


= E[0.05 − i1 ]E[1 + i2 ] · · · E[1 + i10 ] + · · · + E[0.05 − i10 ]
(by independence)

= 0.01(1.04)9 + 0.01(1.04)8 + · · · + 0.01(1.04) + 0.01


= 0.01s10 0.04
= 0.120061

(b) The distribution of the interest rate in each year is:

1 + i ∼ LN (µ, σ 2 )

where:

1 2
E(1 + i) = 1.04 = eµ+ 2 σ
2
 2

=⇒ Var(1 + i) = 0.022 = e2µ+σ eσ − 1
 2 
2 2 σ
=⇒ 0.02 = (1.04) e − 1
0.022
 
2
=⇒ σ = ln 1 +
1.042
1
=⇒ µ = ln 1.04 − σ 2
2
The simulation can be found in Spreadsheet new3.xlsx.

117
Financial Mathematics  Exercises School of Risk and Actuarial Studies  UNSW

Simulated Density of S

5
4
3
Density

2
1
0

−0.2 −0.1 0.0 0.1 0.2 0.3 0.4

(c) A company who has an exposure to a oating rate may want to hedge this
interest rate risk by using a xed-for-oating swap. For example, if the com-
pany has a liability to repay a loan at a oating rate, they could enter into a
swap to pay a xed rate and receive a oating rate. The payment and receipt
of the oating rate will net to zero, allowing the company to pay a net rate
which is xed.

Exercise 5.19 [new5]

(a) The mean is:

E(S1 ) = E(1 + y1 )
= E(1 + µ + β(y0 − µ))
= 1.05 + 0.4(y0 − 0.05)

Therefore E(S1 ) = 1.046 and E(S1 ) = 1.054 for y0 = 0.04 and y0 = 0.06
respectively.

(b) The variance is:

Var(S1 ) = Var(1 + y1 )
= Var(1 + µ + β(y0 − µ) + σε1 )
= σ2
= 0.012

118
Financial Mathematics  Exercises School of Risk and Actuarial Studies  UNSW

(c) The probability is:

Pr(S1 < 1.04) = Pr(1 + y1 < 1.04)


= Pr(µ + β(y0 − µ) + σε1 < 0.04)
 
−0.01 − 0.4(y0 − 0.05)
= Pr ε1 <
0.01
where ε1 ∼ N (0, 1)

Therefore Pr(S1 < 1.04) = 0.2743 and Pr(S1 < 1.04) = 0.0808 for y0 = 0.04
and y0 = 0.06 respectively.

Exercise 5.20 [new6]

The simulation can be found in Spreadsheet new6.xlsx. The histograms should be


similar to the ones below.

Histogram of S (4%)
250
200
150
Frequency

100
50
0

1.4 1.5 1.6 1.7 1.8 1.9

119
Financial Mathematics  Exercises School of Risk and Actuarial Studies  UNSW

Histogram of S (6%)

250
200
150
Frequency

100
50
0

1.4 1.5 1.6 1.7 1.8 1.9

Exercise 5.21 [new9]

(a) Denoting the mean and standard deviation of it as j and s respectively (ie.
j = 0.04 and s = 0.02), the formulae can be derived as in the lecture notes:

s̈n = (1 + y1 ) · · · (1 + yn ) + (1 + y2 ) · · · (1 + yn ) + · · · + (1 + yn )
= (1 + yn ) [(1 + y1 ) · · · (1 + yn−1 ) + · · · + (1 + yn−1 ) + 1]
= (1 + yn )(s̈n−1 + 1)
∴ E(s̈n−1 ) = E[(1 + yn )(s̈n + 1)]
= E(1 + yn )E(s̈n−1 + 1) by independence
= (1 + j)[E(s̈n−1 ) + 1]
E s̈2n = E (1 + yn )2 (s̈n−1 + 1)2
  

= E (1 + yn )2 E (s̈n−1 + 1)2
   
by independence
2 2
 
= E 1 + 2yn + yn E s̈n−1 + 2sn−1 + 1
= 1 + 2j + j 2 + s2 E s̈2n−1 + 2E (sn−1 ) + 1
  

The variance can then be determined using:

= E s̈2n−1 − [E (s̈n−1 )]2



Var (s̈n−1 )

(b) The recursion can be found in Excel spreadsheet new9.xlsx. We see that
E (s̈30 ) = 58.33 and V ar (s̈30 ) = 17.16.
(c) The distribution of the interest rate in each year is:

1 + i ∼ LN (µ, σ 2 )

120
Financial Mathematics  Exercises School of Risk and Actuarial Studies  UNSW

where:

1 2
E(1 + i) = 1.04 = eµ+ 2 σ
2
 2

=⇒ Var(1 + i) = 0.022 = e2µ+σ eσ − 1
 2 
=⇒ 0.022 = (1.04)2 eσ − 1
0.022
 
2
=⇒ σ = ln 1 +
1.042
1
=⇒ µ = ln 1.04 − σ 2
2
The simulation can be found in spreadsheet new9.xlsx.

Simulated Density of Annuity


0.08
0.06
Density

0.04
0.02
0.00

45 50 55 60 65 70 75

s30

121

You might also like